Вы находитесь на странице: 1из 196

M. L. Krasnov G.I.

MaKarenKo
A.I. Kiseliov
;
CALCULO
VARIACIONAL
(ejemplos y problemas)
Editorial MIR
Loe autores de este libro &un
Mijafl i<tasnov
1
Grigori Makn
candidatos a Doctores en
Ciencias fisicomateuyticas y do-
centes Instituto
de Mosc, y Alexandr Kiseliov ,
colaborador cienHlicu superior
del Instituto de In
vestigaefones Nucleares de Ja
ciudad de Duhna.
Este compendio contiene pro
blemas y ejercicios dedicadM "
ilustrar los diferentes principi<lS
de la teor(a y los mtodos de
resolucin de las ecuaciones por
el clculo de variaciones.
M. L. Krasnov, G. l. Makarenko,
A. l. Kiseliov
CLCULO
V ARIACIONAL
(ejemplos y problemas)
Traducido del ruso por Carlos Vega j
candidato a doctor en ciencias fsico-matemticas
e Editorial MIR
l.S.B.N.: 84-(>()4..1605-4
D.L: M-5299-1992
@F.di torial MIR- 1992
Imprcsi6n: Grficas Zenit. Madrid
Prefacio a l a edi cin espaol a . . . . . . . . . .
Observaci ones prelimi nares .. .... .. .
Capitulo l. Extremo de funciones de varias variables
l. Extremo incondic ionado
2. Extremo c.ondicionado . ....... .
Captulo l J. Extremo de funcional es
3. Funcional. Vari acin de una [unc ional 'i sus p ropie
dad es ..... . . .. . .. .. ..... .
4. Pro blema element al dt>I Cl culo variacrona l. E<:: ua-
cin de Eulcr . . . . . . . . . . . . . . . . .
5. Generalizaci ones del problema elemental del Clculo
vari acional . . . . . . . . . . . . . . . . . .
6. l nvariancia de l a ecuacin de Euler .
7. Campo de ext remal es . .... .. .... . . .
~ 8. Condiciones suficientes de extremo de una funcional
9. Ext remo condicionado .... ... ... .. .

10. Problemas var acionales con fronterns mviles .. .


l l. Problemas discontinuos. Var iaciones unilat eral es
12. Teor a de H amilton- J acobi. Principios variacionales
de la Mecnica . . . . . . . . . . . . . . . . .
Capt ul o 1 J l. Mtodos directos en el Clculo var iacional
13. Mtodo de diferencias fini tas de Euter ..
14. Mtodo de R. ilz, Mtodo de Kantorv1ch .
15. Mtodos v ari acionales para la determi nacin de los
v l o r e ~ y de l as funciones propios . . . . . .
Res pues tas e indicaciones . . . . . . . . . . . . . . .
7
9
11
HI
26
50
68
77
80
i
106
121
134
143
15B
160
167
180
BIBLIOGRAFJA
1. Courant .R . y D. Hilbert , .Methoden der Mathematischen Physi k
(Mtodos de la Fsica Matemtica), vol. 11, Spr inger, Berlin, 1937.
2. L. Elsgoltz, Ecuaciones difer enciales y clc:ulo variacional, Edito-
rial MIR, Mosc, 1969.
3. 11 . M . I'e111J</JaHO u C. B. c/JOA!Uk, BapHal(KOHHoe JtC'lHCJieHHe ,
Cl>l:laMaTrH3, 1961 (/. M. Ouelfand y S. V. Fom n. Clculo varia-
cional).
4. E. Goursat, Cours d'anal yse mathma tique <Curso de Anl isis
Matemtico), vol. IJI, 5 ed .. GauthierVillars. Pars. 1942.
5. M. A . JlaepeHmt.ee u .ll. A. JliocmepHUK, Kypc eapHaUHOHHoro
HC'UICJl('fflUI, rocTeXH:JAaT, 1950 (M. A . Laurientiev y L. A . lus-
ternik, Curso de clculo v ariac ional ).
6. J . Rey Pastor, P. Pi Calleja, y A. C. Tre;o, An lisis matemt ico,
vol. 111, 211 ed. , K.apelusz, Buenos Aires , 1961.
7. L. C. Y oung, Lectures on the calculus of variat1011s and op t i ma!
control theory (Lecciones sobre el clculo variaclonal y l a teori a
de control ptimo) , Phil adelp hia, London, Toronto, 1909.
PREFACIO A LA EDIC1N ESPANOLA
Hoy da todo ingeniero tropieza con
problemas que requieren buenos conocimientos matemticos
y perici a en la aplicacin de distintos mtodos matemticos.
Se puede afirmar que la elevacin de la cultura matemtica
de los ingenieros contribuye a nuevos logros en la Tcnica.
El Clculo variacional es uno de los captulos del Anl s ic;
Matemtico clsico ms importante para las aplicaciones.
Actualmente, en varios Institutos politcnicos e1 Clculo
variacional se incluye en el programa obligatorio del curso
de Matemticas superiores. Existen valiosos libros sobre el
Clculo variacional como, por ejemplo, los libros de L. Els-
goltz [2}, de E. Goursat [4), de L. C. Young [7] , etc. En cuanto
a los problemas, muchos de ellos aparecen diseminados en
numerosos textos o artculos cientficos especi ales dedicados
a este tema. Pero, por lo que conocemos, no exist.e en la
literatura correspondiente ningn libro de probl emas ded i-
cado especialmente al Clculo variacional. Los autores se
han planteado la tarea de preparar un cierto rnnim0 de
problemas referentes a los captulos pri ncipales del Clculo
variacional cl sico (sin tocar las cuestiones relacionadas. con
la Teora de direccin ptima).
El libro est escrito de forma que al principio de cada
pargrafo se dan los elementos tericos ind ispensables {defi -
niciones, teoremas y frmul as) y se anal izan deta lladamente
ejemplos t picos.
El libro contiene ms de 100 ejemplos y 230
problemas para eJ trabaj o individual. Al final se dan las
respuestas a todos problemas y, en algunos casos, las indica-
ciones correspondientes. Por eso, el libro puede servir tan to
para estudiar asignatura individualmente como para proftm-
dizar en el material que se expone en las lecciones.
8
PREFACIO
Consideramos deber nuestro agradecer al traductor Carlos
Vega, candidato a doctor en ciencias fsico-matemticas, por
el gran trabajo que se ha realizado al revisar los problemas
y por las tiles observaciones que ha hecho contribuyendo
al mejoramiento del libro.
30 de Octubre de 1974.
M. L. Krasnov
O. /. Makarenko
A. 1. J( iseliov
OBSERVACIONES PRELIMINARES
l. Si A es un conjunto cual quiera de elementos, la proposici n
cel elemento a pertenece al conjunto A se representa simblicamente as:
a E A.
Si se escribe a E A (o bien a E A). ello significa que el elemento a
no pertenece al conjunto A.
Siendo A y 8 dos conjuntos, la proposicin A es un subconjunto
del conjunto 8'1 (A e B, s imblicament e) significa que todo el emento x
del conjunto A tambin pertenece al conjunto B.
2. La unin y 1 a interseccin de dos conjuntos A y B se defi nen
del modo siguiente:
la unin A U B = {x 1 x E A o x E 8} es l a totalidad de los
elementos x que pertenecen por lo menos a uno de los conjuntos A o B:
la interseccin A n B = {x 1 X E A. X E B} es la totali dad de los
elementos x que pertenecen tanto a A como a 8 .
3. Si A es un conj unto formado por nmeros real es, se denomi na
cota superior (cota superior exacta) de A el menor nmero real M tal
que a ~ M para todo a E A. En otras palabras, M es la cota superior
de A si para lodo a E A es a ~ M y si para cualquier s > O, por peque
o que sea, existe como minimo un elemento b E A tal que M - e < b.
SI no existe tal nmero, convendremos en decir que la cola superior
de A es +oo.
En ambos casos designaremos la cot a superior del conjunto A
por sup A.
Anloga mente se define la cota inferior del conjunto A que se re-
presen ta por inf A.
4. Se denomina espacio lineal t odo conjunto R de el ementos
x, y, z, ... de naturaleza arbitrarla para los cuales estn definidas dos
operaciones, de adicin y de multiplicacin por nmeros, que cumpl en
los axiomas siguientes:
~ X + !1 = y + x;
2 (x + y) + z = x + (y + z);
3 existe un elemento O (elemento nul o) tal que x + O= x para
todo x E R:
4) para todo x E R existe un elemento - x (el emento opuesto)
tal que x + (-x) = O;
5) l X = x;
6) a (Jlx) = ( a ~ ) x;
7) (a + ) x = ax + x ;
8) a. {X+ V) = l;tX + ag.
JO
OBSEHVACJONES P RELlMI NARF.S
5. Vil espacio lineal R se ll ama normado si a todo elemento x E R
le corresponde un nmero real no negativo U x \l. llamado norma de
este <.>lement o, con la particulari dad de que
1) ll x U = O slo s i x = O;
2) Ct X IJ = 1 et. l 11 X 11 ;
3) x + y 11 11x11 + 11 y11 (axioma triangular para l as nor
ma s):
6. Un conjunto M de elementos x, y , z, . . . de naturaleza arbitra
ria se denomina espacio mtrico si a todo par de elementos x, y de M
le corresponde un nmero r eal no negativo p (x, y) de modo que
1) p (x, y) = O si, y slo si, x = y (axi oma de i dentidad):
2) p (x, y) = p (J, x) (axioma de simetr a);
3) (l (x, y) + p (y, z) ;.. p (x, z) (axioma triangular).
El nmero p (x, y) ll eva el nombre de distancia entre los elemL11los
X e y.
Toao espado lineal normado es mt rico: basta tomar p (x, y) =
= 11 X - IJ 11
7. El espacio e [a, b] es<.>! espacio formado por todas las funciones
,e (x) continuas en {a., b] donde
11 !I ll e = mx l y (x [.

El espacio C
1
[a , b] es el espacio formado por todas l as funciones
y (x) que, a parte de ser continuas, tienen derivada primera cont inua
en [a, bl donde
11 Y llc
1
= mx l Y (x) 1 + mfix 1 y' (x) 1.
a:% x <b
El espa.cio Cn ta, b] es el espacio formado por todas las runclones
y (x) que, a parte de ser continuas. ti enen en (a, b] derivadas continuas
has ta de orden n-simo incl usive (fl es un nmero natural fij o) donde
1\
11 Y /l en= mx 1 ch.) (x) I
k = O
A veces la norma del ele.mento y (x) en Cn [a, b) se define as:
11 Y llcn = mx { 1 Y (x) J, 1 y' (x) , . . . , 1 y<n> (x) 1 }.

Captulo I
EXTREMO DE FUNCIONES DE VAR.IAS VARIABLES
l. Extremo inccndicionado
Sea f (x
1
, x
2
, , Xn ). o brevemente f (x), una funcin definida
en un recint o D del espacio eucldeo En de n dimensiones.
Diremos que la funcin f (x) alcanza su valor mximo (mnimo)
en el punto x
0
E D si
f (x) f (xo) (f (x) f (xo))
cualquiera que sea el punto x E D.
TEOREMA DE BOLZANO-WEIERSTRASS. Toda funcin continua en
un recinto acotado cerrado alcanza en l sus valores mximo y mnimo.
DEF'INICION 1. Sea f(.x) una funcin de[inida en un recinto De E<m.
Diremos que el punto .x<
0
> = (x? .... E D es un punto de mximo
estricto (un punto de mnimo estricto, respectivamente) de l a funcin
f (x) si existe una vecindad U (x<>) del punto x 1J tal que la desigualdad
f (x) < f (x<
0
>) (la desigualdad f {x) > f (xt
0
>), se
cumple para t odos los puntos X E g (x<>) n D. X =t= x<>. Es decir, lo
que caracteriza el punto de mximo estricto (el punto de mnimo es-
tricto, respecti vament e) es que
f = f {x) - f (x<
0
>) < O (t./ > O, respectivamente)
para todo X E Q n D, X#= x<
0
1.
En cambio, si para el punto x<> existe una vecindad Q (x<
0
>)
tal que para todos los puntos X E Q (x<l) n D se cumple la desigual dad
f (x) f (x<>) (l a desigualdad f (x) ;;;.: f (x<
0
>) , respectivamente). se
dice simplemente que el punto x<> es un punto de mximo (un punto
de mnimo, respectivamente).
DEFINIClN 2. Los puntos de mximo y de mnimo da la funcin
f (x) se denominan punt<Js de extremo de la misma.
J. Basndose en la definicin, hallar Jos puntos de extre-
mo de las funciones
a) f (x,, x
2
) = +
{
xr +Xi Si Xt + Xi=#= 0,
b) f(x .. x
2
)= 1 si xi+x2=0
l 2 '
e) f (x,, x
2
) = x-
en el recinto D {x: + 1 }.
12
CAP. l. EXTRf>MO DE FUNCIONES
1 (condicin necesaria de extremo). S ea j (x), x =
= {XI> Xi , . . . , Xn). un.a funcin definida en una vecnda del punto
x'> = xg, . . . , xg). Si este punto es un punto de extremo de la
funcin f (x) y .si en l existen las derivadas o' (i = 1. '?, .. ., n), todas
Xj
ellas son iguales a cero:
of (xiO>) = 0 ( . 1 2 )
" 1 = ' ' , n.
VXj
Sl la funcin f (x) es diferenciable en el punto de extremo x<
0
>, su
diferencial en. este punto es igual a cero: df (xi >) = O.
EJEMPLO 1. Hallar los puntos de exti:emo de la runcin z = x
2
+ y'!..
SOLUCION. Los puntos de extremo estn entre los puntos para los
cuales dz = O. En nuestro caso, dz = 2x dx + 2y dy. La condicin
dz = O se cumple en el punto x = O, y = O solamente. En erecto, si
x = y = O, tenemos dt = O. Recprocamente, sea dz = O; basndonos
en que dx y dy son arbitrarios, tomemos dy = O de modo que O = dz =
= 2x dx de donde, puesto que dx es ar bitrar io, resulta que x = O;
anlogamente encontramos que tambin y = O. En el punto (0, 0)
tenemos z = O; en todos los dems puntos tenemos z = :<'
2
+ yZ > O.
Por eso, el punto (0, O) es un punto de minimo estricto de l a funcin
z = xz + y2.
Si se ampla la clase de funciones en la que se busca el extremo
incluyendo en ella las funciones no dilerenciables en algunos puntos,
se llega a la siguiente condicin necesaria de extremo.
Si x<
0
1 es un punto de extremo de la funcin f (x
1
x2 Xn)
cada una de las derivadas parciales :1 (i = 1, 2, .. ., n} es igual a cero
vX
o no existe en dcho punto.
EJEMPLO 2. Consideremos la parte superior z ;:;;.- O del cono z
2
=
= xll + .yi. Es obvio que la funcin z tiene. mnimo en el punto (O, O).
Pero las derivadas :; y :: no existen en est.e punto.
DEP!NIClN 3. Los puntos en los que se cumple la condicin necesa
ria de extremo de la funcin f (x) se denominan puntos crticos de la
misma.
Los puntos x<
0
1 en los que df (x>) = O se denominan puntos esta
cionarios de la funci n f (x).
La condicin df (x<>) = O es equivalente a la condicin
of (x<Ol )
ax
O (i=I, 2, ... , n).
La existencia de punto crtco no garantiza an la existencia del
extremo de una funcin. Por ejemplo, el punto (O, O) es un punto es ta-
cionarlo de la uncin z = x - y'J. y, sin embargo, la funcin z no
tiene extremo en l: en cualquier vecindad del punto (O. O), por pequea
que sea, la funcin toma valores tanto positivos como negativos.
1 EXTREMO I NCONDICION1\DO
1 . Condiciones suficientes de extremo est ricto
DEFINJCION 4 . Se dice que la forma cuadrl ca
n
A (x) = A (xi. X2, . . , Xn) = _; OXX ;
i, ;- t
a ""' a; i, = 1, 2, . . . , n:
13
es definida positiva (definida negativa, si A (x) >O
(A (x) < o respectivamente) para todo punto X E en, X -:::fo o, y se
anul a slo para x = O, o sea, para x
1
= x
9
= . . . = x
11
= O.
La for ma cuadrt ica se denomina no negativa si jams toma valores
negativos. Por ejemplo, las formas
son ambas no negat ivas. La primera es definida positiva ya que se
anula slo para x
1
= x
2
= ... = Xn = O; en camhio la segunda no lo
es ya que se anula, por ejemplo, para x
1
= l. x
2
= - 1, x
1
= x
4
=
= ... = Xn = 0.
Una forma cuadrtica definida positiva o definida negativa se
denomina forma cuadrtica defi nida.
Una forma cuadrtica que toma valores tanto positivos como
nega tivos se denomina indefinida.
TEOREMA 2. (condiciones . suficien tes de; ext remo estricto). Sea
f (x) un.a funci11 de{ in ida en una uecindad del punto x'' =
= (xy, xa ... , xft) en la que son continuas sus segundas deriuadas y sea
x<
0
> un punto estacionario de la funcin f (x) . Si la forma cuadrtir.a
( 1)
o sea , la segunda diferencial de la funcin f en el punto xi
0
>, es def inida
positiva (definida 11egativa, respeclivamenle). el punto x t
0
> es punto de
mnimo estrcto (punto Je mdximo tstriclo, respectivamente) : sl la forma
cuadrtica (1) es indef inida, no hay extremo en. el punto x
0
>.
CRITERIO DE S'\'LVESTER DE FORMAS CUADRAT ICAS DEFINIDAS
POSITIVAS. Condi cin necesaria y suficie11te para que la. forma cuadrtica
n
A (x) = 11 (Xi> X2, , x
11
) = a;XXj
' 1-= l
(2)
14
C.Ar l. bX l IH'.MO DE FUNCION!.':-
cott au = J;; i, ; =- 1, 2, . . . , u; sea definida pusiliva es que se cumpla
11 12 a13 I
21 a22 a23 > O,
a31 a32 :l3
au a12 1n
.... ' >0-
Condici6n necesaria y suficiente para que la forma cuadrtica (2)
sea de{ inida nega/illa es que se cumpla
11 12 a13
a21 022 a23 <O,
a31 a32 a33
au 12 - tn
a21 a2z . zn
a,.1 an2 a,.u
CASO n2- Sea f (x, y) una funci n definida en una vecindad del
punto (x
0
, y
0
) en la que son continuas sus derivadas parciales de segundo
orden y sea (x
0
, y
0
) un punto estacionar io, es decir, sea
(xo. Yo) = f (xo. Yo) = O.
Entonces, si en el punto (x
0
, y
0
)
l'!cxf'
11
- {/';
11
)
9
> O,
hay extremo en este punto; a saber, mximo si en l
y mnimo si en l
/';ex < O

< O)
> O (f'jy > 0).
Si en el punto (x
0
, Yo)
r;x1'
11
- < o,
no hay ext remo en el punto (x
0
, y
0
). Por ltimo, si en el punto (x
0
, Yo)
f';.:itf;y - (/';.,11)
2
=
i 1
EXTREMO 1 NCONDICIONADO
15
en dicho punto puede haber extremo y puede no babe.rlo; este caso
requiere un estudio complementario.
. EJEMPLO 3. Consideremos 1 as funciones z = x' + y4, z = -x' - y4
y z = x4 - y4. El punto (O, O) es un punto est acionario de las tres y
en l se tiene - (z;
11
)
11
= O para cada una de las funciones.
Es fcil ver que el punto (0, O) es un punt o de mnimo de la pri
mera funcin. un punto de. mximo de la segunda y no es punto de
extremo de la tercera. Efectivamente, en Jos tres casos tenemos
2 (O, O)= O; si n embargo, en cualquier vecindad del punto (O, O),
a excepcin del propio punto, )os valores de la funcin son positivos
en el primer caso y negativos en el segundo mi entras que en el tercer
caso la funcin z = x' - {/' toma, en cualquier vecindad del origen
di :::oorde11adas, valores tanto positivos (por ejemplo, si x =FO e y = O)
como negativos (por ejemplo, si x = O e y =FO).
t::JEMPLO 4, Hall ar el ext remo de la funcin de tres variables
f = x + y
2
+ :z - xy + x - 2z.
SOLUCIN. Determinamos los puntos estacionarios de la funcin f.
Consideremos con este fin e\ sistema de ecuaciones
= 2x-y+ 1 = 0, )i
of
<)y = 2y-x= O,
of 1
oz = 2Z-2=0; J
2
resolvindolo, x
0
= -
3
,
Consideremos lii forma cuadrtica
--} , 1). Tenemos
1
Yo=-3
(1) en el
y Zo =: l.
punto P
0
( - E.- ,
.)
r;.x= 2,
- 1,
f ;x= O,
f;u = -1, f;z=O,
fJ11 = 2, fj'l. = 0,
!;11= O. / ;z=2.
En el punto P
0
encont ramos
au = 2,
au = -1,
111. = - 1,
a,,= 2,
a
13
= O,
023 = o,
= 2,
de motlo que
a1 = O,
'=o,
11 >o,
1
au Gz 1-1 2 - 1 1-3 --... 0
21 22 - l 2 -
2 -1 o
- 1 2 o = 6> 0.
o o 2
16
G,\P. l. EXTREMO DE FUNCIONES
Basndose en el criterio Sylvester, llegamos a la conclus in de que
la (or ma cuadrtica es definida positiva; por lo t anto, en virtud del
teore ma 2, el punto P
0
es un punto de mnimo estricto s iendo f (P
11
) ==
4
=- 3
EJEMPLO s. Ha ll ar el ex t remo de l a funci n de dos variabl es
z=x
3
y2 (6-x- y).I
SOl.UCl N. Determinamos los puntos estacionarios;
,_, 18x
2
y
2
- 4xy
2
- 3x
2
y
3
= O, }
Zy = 12x'.ly - 2x'y - 3x3y2 = 0,
de donde x
1
= O, y
1
-: O y x
2
= 3, yz = 2. Hemos obtenido dos puntos
estacionarlos P
1
(O, O) y P
2
(3, 2).
Ca lculc:>mos l as segundas d!!ri Vadas de la funcin:
z;;.x = 36xy
2
- l 2x
2
y:i - 6xy3,
ZJy = - 2xt - 6x
3
y,
i;y = 3Gx
2
y - 8x
3
y - 9x:ly
2

En el punt o P
1
tenemos z;..x = z"yu = z;.
11
= O de modo que


- (z;
11
)
2
=O y queda pendiente e problema sobre l a existencia
de ex tremo en es le punto; para resolverlo habr que recurrir a las
derivadas superiores.
En el punto P
2
tenemosz;x = -144, zyy = -162 y z;y = - 108.
Queda daro que -

>O y como z"xx < O, en el punto
P
2
l3, 2) hay mximo siendo zmx = 108.
Hall ar los mximos y los mnimos de las funciones:
2. f = (x - l )
2
- 2y
2

3. f = x
4
+ y
4
- 2x
2
+ 4xy - 2y
2

4. = (X2 + y2) e-<x2+112>.
f
1 +x-y
s. =V l +x2+y2
y2 z2 2
6. f=x+4X+y-+-z(x>O, y>O. z>O}.
7. f = x
2
- xy + y
2
- 2x + y.
8. f = sen x sen y (x + y) (O x n, O y :i)
9. f -= x
1
x; ... (l - X
1
- 2x2 - . . . - nxn)
(X1 :>O, X2 >O, . . , X
11
>O).
l. EXTREMO INCONDICIONADO
17
10. Demostrar que la funcin z = (1 + e11) cos x - ye"'
tiene una cantdatl infinita de mximos y no tiene mnimos.
11. Ser condicin suficiente para que la funcin f (x, y)
tenga mnimo en el punto M
0
(x
0
, 9
0
) el que esta funcin
tenga mnimo a lo largo de cualquier recta que pase por el
punto M
0
? Considerar la funcin f (x, y) = (x - y
2
) x
X (2x -y2) .
12. Demostrar que (a diferencia de las funciones de una
variable) incluso para l as funci ones de dos variables la exis-
tencia de un extremo nico - mximo o mnimo - en un
recinto D no significa aun que este extremo represente el
valor mximo o minimo de la funcin respecto a todo el
recinto. Considerar los ejemplos:
a) z = x'.! - y'.! + 2e-.x2,
-oo <x<+ oo, -oo <Y<+ oo;
b) z = x
3
- 4xll + 2xy - y",
D {-5 x 5;
13. Sea f (x) una funcin peridica con perodo 2n. Entre
todos Jos polinomios trigonomtricos de grado n
n
; + (a.h cos kx + sen kx)
h=I
determinar, escogiendo convenientemente los coeficientes a.1;
y aquel que ofrece el valor mnimo para el error cuadrti-
co definido por la igualdad
n n
6!=

) [t (x)- i - J2 dx.
-n
2. Mtodo del gradiente. Suponga mos que es preciso hall ar e l
mfnimo de la funcin f (x), donde x = (x
1
, x
2
. , xm) Tomemos
un punto xO = (xf, xU ... , xi.> y calculemos el gradiente de la uncin
f (x) en este punto
m
grad f (x) = ei.
l
i- 1 .
donde ei. e2 . , en es una base ortonormal del es pac o R.
11

Si se tiene gr ad f (xi>) :: O, ponemos
xi = x - h
1
(grad f (xO), e1i) (k = 1, 2, ... , rn) ,
2-01387
18
GAP. l. EXTREMO DE FUNCIONES
donde h
1
> O es suficientemente pequeo. Si se tiene grad f (xl) =f=. O,
ponemos
en general, s i se ll ene grad f (x
11
-
1
) =f=. O, ponemos
x ~ = x ~ -
1
- hn {gr ad f (xtt-
1
), e,.)
(k = 1, 2 , . . , m; hn >O).
As obtenemos. si se cumplen determinadas condiciones, una su
cesin montona decreciente {/ (x")}. SI x" - x y X' es un punto de
ml nl mo de la funci n f (x), se tiene g_rad f (x
11
) - O cuando n-+- oo.
EJEMPLO 6. Hall ar el punto de mnimo de l a funcin f (x) = x.
SOLUCIN. Tomemos, por ejemplo, el punto x:O = l. Tenemos
grad f (x"} = 2x"I = 21 =FO.
Por eso, ponemos
x1 = x;O -h2=1 -211. donde h >O.
Tenemos ahora
grad f (x
1
) = 2 (1 - 211) l .
Si h =F ! , es grad f (x
1
) =I= O y ponemos
x
2
= x
1
- 2h (l - 2h) = ( 1 - 2h)2.
Conti nuando este proceso, encontramos
xn = (1 - 2h)
11

Es claro que, s iendo O < Ti< \, se tiene xn - O para 11- oo. El
punto x = O es el punto de mnimo de la funcin f (x) = xi. Si /1 = ~
es x
1
= O y grad f (x') = O y obt enemos la sucesin estacionaria {O}
cuyo li mite es el cero.
EJ.EMPLO 7. Hallar el punto de mlni mo de la funcin f (x, y) =
= r + yz.
SOLUCION. Tomemos, por ejemplo, el punto (1, 1 ), o sea, tomemos
x
0
= 1 e IJ = l. Tenemos
grad f (1, 1) = 2 + 2j.
Pues to que grad f (!, 1) =/=O, ponemos
x
1
= x
0
- 2xll = 1 - 2h,
Tenemos
y
1
= y - 2yh = 1 - 2h.
.
(h >O)
grad f(xt, y
1
) = 2(1-2h)i+2(1- 2h) j 4: O
y, por eso, lomamos
xi= xl- 2xlh = ( 1--2h)2,
yi= y1-2y l / = (l -2h)Z.
2.
EXTREMO CONDICIONADO
Continuando este proceso, encontramos
xn = (l - 2h)n,
yn = (1 - 2h)n,
19
de modo que para O < h < 1 obtenemos una sucesin de puntos
Mn (x"', y
11
) convergente al punto M (O, O) de mlni mo de la funci n
considerada. Es obvio que
gr ad f (xn, yn) = 2 (l - . 2h)n l + 2 (l - 2h)"' j-+ O cuando n-.. oo.
Es decir, el punto (O, O) es el punto de mnimo de la funcin f (x, y) =
=xi+ y'l..
Empleando el mtodo de gradiente, hallar el punto de mni mo
de la funcin
z = x2 + y'J. - 2x + 4y + 5.
2. Extremo condicionado
Sea z = f (x
1
, x
2
, . , Xn) una (uncin de n variables definida
un recinto D del espacio En.
Supongamos, adems, que las variables x
1
, x2> .. .-e
71
estn
ligadas por (m < n) condiciones complementarias
: .. (I)
q>m (x,, X2, , Xn) = O,
que se denominan ecuaciones de enlace.
Sea x<
0
l = (xy, ... , un punto interior del recinto D.
Se dice que f (x
1
, x
9
, , xn) tiene mdximo condicionado (mnimo
condicionado, respectivamente) en el punto tj, .... xYi) si la de
sigualdad
f (xi. X2, , xn) f M, xK ....
{la desigualdad f (x
1
, x
2
. , xn) > f xi, . ... respectivamen
te) se cumple en una vecindad del punto ... , siempre que
los puntos (x
1
, x
2
, , Xn) y .... xi) verifiquen las ecuaciones
de enlace (1 ).
EJEMPLO 1. La funci n z = x
3
+ y"' tiene mfnimo incondidonado,
igual a cero, en el punto (O, O). Agreguemos la ecuacin de enlace
x + y - 1 = O; se trata entonces de deter minar el mlnimo de las
Z-coordenadas de los puntos de Ja superficie z = x' + y'l. considerando
slo aquellos valores de x y de y que satisfacen la ecuacin x + y - 1 =
= O. Este mnimo condicionado no se puede alcanzar en el punto
(O, O) pues este ltimo no satisface la ecuacin de enlace. Resolviendo
la ecuacin de enlace x + y - 1 = O respecto a y e introduciendo la
expresin obtenida y = \ - x en la ecuacin de la superficie, encon-
tramos z = xt. + (1 - x)t., o sea, obtenemos una funcin de una
variable. Extremndola, encontramos Xcr = y zm1a = A partir

20 CAP. l. EXTREMO DF. FUNCI ONES
de la ecuacin de enlace, determinamos Ycr =;. El punto ( ; , ; )
es el vrtice de la par bola que corresponde a Ja interseccin del para-
boloide z = + y'I. con el plano x + y - 1 = O.
Anlogamente se puede proceder en situaciones ms gc.>nerales.
Supongamos que se busca el extremo condi cionado de la funcin
z = f (x, y) siendo qi (x, y) = O ta ecuacin de enl ace. Supongamos que
para los valores considerados de x y de y la ecuacin q> (x. y) = O deter-
mi na y como una funcin unvoca diferenciable y = 'I' (x) . Sustituyen
do y por 'i> lx) en la funcin f lx, y), obtenemos una funci n de una
variable x; z = f (x, '!> (x)) = F (x). El extremo (incondicionado) de
la funcin F (x) sera el extremo condicionado buscado de la funcin
f (x, y) con Ja condicin de enla ce qi (x, y) = O. En la prctica este
mtodo resulta poco cmodo ya que para a plicarlo es preciso resolver
la ecuacin <p (x, y) = O respecto a una de las vari abl es.
Para hallar los extremos de la funcin z = f (x
1
t
2
, ... , Xn) con
las condiciones tle. enlace (i) se emplea el mtodo de los multiplicadores
de Lagrange.
MeTODO DE LOS M.ULl'IPLICADORES DE LAGR.l\NO.E. Supongamos que
1) las der ivadas pardales de primer orden de l as runciones
f (Xt. x2, . ... Xn) y cp, (x1. x2, .. . , Xn) (i = ! , 2, . . . , m) son continuas
en el recinto D;
2) m < n, :mmdo el rango de Ja matr iz { i:; ) (i = t , 2, ... , m;
= I, 2, . . . , n) igual a m en todo punto del recinto D .
Consideramos una funcin nueva (fuucin de Lagrange)
m
. '-.:'
<l> -=- 1+ L..J h lp,
'-'
donde /..
1
son factores constan tes indeterminados.
Despus analizamos d extremo incondicionado de la funcin
<P (XJ, x
2
, .. . , x,.), o sea, formamos el sistema de ecuaciones
o<I> o'11 c<I>
OX1 =O, ax; =O, ' ., OXn = O (
2
)
y, a partir de este sistema y de 1 as m ecuaciones de enl ace
tp = O, <jl 2 = O, . , 'Pm = O,
determinamos los val ores de los parmetros /.
1
A.
2
, , A.m y las coor-
denadas (x
1
, x
2
, ., xn) de l os posibles puntos de extremo.
Las condiciones (2) son condiciones necesarias de extremo tant o
para la funcin de Lagrange como par a la funcin inicial z =
= f (X, X2
1
, Xn}.
Si el pu nt o (x?, .. . , es un punto de extremo condicionado
de la luncin l (xI> x
2
, . , Xn ) , ser a la vez un pun lo estacionario de
1 a r uncin de Lsgr auge, o :,(!a, !!11 este punto = O (i = 1, 2, . ..
. . , ri) . Para atwltzar el punto estaci onari o (xY. ... x?
1
) en tanto
2. EXTREMO
21
que ex tremo condiciona do de la funcin de Lagrange <ll(x
1
, x
2
, , x,,)
habr que considerar la for ma cuadrtica
n m
B (d;ci, dx2 . . , d,'TI-m} = /J11 dx; dxj.
f, ;-1
o sea, la segunda diferenci:il
cue nb las condiciones
de la funcin de Lagrang<', teniendo en
a,., oq
. ..
v .\ t vX2
OfP
\ -i)- dx" =- O (i = 1, 2, ... , m).
Xn
Si la forma cuadrlic11 (3) es definida, en el punto xg, ... , x?i)
tendremos extremo condicionado es tricto; a saber, mximo condicio-
nado estricto si la forma cuadrtica (3) es definida negati va, y ml nmo
condicionado estricto s i la forma cuadrtica l3) es defi nida posi tiva.
En cambio si la forma cuadrtica (3) es indefinida, en el punto
... , no habr extremo condicionado.
Por consiguiente, la existencia en el punto (x'l, . .. , de
mxi mo (mnimo} incondici onado de la funcin de l.agrange (tomada
con los valores encontrados para ;>...
11
1.
3
, . . , ,m) implica la
de este punto mximo (m nimo) condicionado de 111 funcin z =
= f (x
1
, x
2
, , Xn) con las de enlace
Ql (x
1
, x2 .... x,, ) = O (l - 1, 2, .. . , m) .
La ausencia de extremo incondicionado de la funcin de
<t> (x
1
, x
2
.. , .xn) no significa an la ausencia de ex tremo condicionado
de la funcin f (xlP X2, ... , Xn):
EJEMPLO .2. Hall ar el extremo de la funcin z = xy con la con-
dicin !J - X = 0.
SOLUCIN. Formamos la (uncin de Lagrange
<D (.x, y) = xy + '- (y - x}
y el sistema correspondiente para determinar ). 'J lss coordenadas de
los posibles puntos de ext remo
= y- ). = 0, }
a<D
Jy = x+ "' =O,
y-x=O.
La primera ecuacin da A= y. Tenindolo en cuenta, encontramos la
segunda ecuacin x + y = O. Es deci r,
x+y= O, }
y - x= O.
de donde .x = y = O y, adems, A. = O. Por lo tanto, la funcin corres-
pondiente de Lagrange es <D (x, y) = xy. La funcin <D (x, y) no tiene
extremo Incondicionado en el punto (O, O).
22
CAP. J. EXT'REMO DE PUNCIONES
Sin embargo, exste el extremo condicionado de la funcin z = xy
con la condicin y = x: en efecto, tenemos en este caso z = x', de
donde resulta que hay mfnlmo condicionado en el punto (O, O).
EJEMPLO 3. Hall ar el extremo condicionado de la funcin
f (x, y, z) = xyz
con las condiciones
<p
1
(x, y, z)=x+y-z-3=0, }
<f'2 (x, y, z)=x-y- z- 8= 0. (
4
)
SOLUCIN. Formamos la funcin de Lagrange
t!> (x, y, z) = xyz + J..
1
(x + y - z - 3) + /..
9
(x - y - z - 8)
y el s istema de ecuaciones para determinar los parmetros Ai y i..
1
y 1 as coordenadas de los posibles puntos de extremo
: )
E - ... -. 1
1
x+u-1-3= 0,
x-y-z-8=0 . .
Resolviendo el sistema de ecuaciones (5), obtenemos
11 231 11 5
A.1 = 32, x=4' Y=-2 y
11
Z= - 4
La segunda dierencial de la fllncin <I> (x, y, z) es ig..ial a
aiai a2m aZ<D
d2<I> =--dxz+ --dyz+--azz +
axz ayz iJz2
(5)
82(,I> (}Z$
, 2 ax oy dx dy -f-2 iJy iJz dx dz+ 2 dy dz.
F.n nuestro caso,
d9<1> = 2z dx dy + 2y dx dz + 2x dy dz.
De las ecuaciones de enlace (4) encontramos
dx+ dy-dz=O, }
dx-dy- dz = O,
(6)
de donde dx = dz, dy = O. Introduciendo estas expresiones en (6)
1
obtenemos
8 (dx) = 2y dil.
En el punto estacionario se tiene B = -5 dx2 <O, o sea, en el punto
(
11 5 11 ) 605
T , - 2 , - 4 hay mximo, siendo /mu=
32
.
2. EXTREMO CONDICIONADO
EJEMPLO 4. Hall ar el extremo de 13 funcin z = c0s
2
x + cos
2

con la condici n
n
y-x=T
SOLt:ClN. Formamos l a [uncin de
<D(x, y)=cos2.x+ cos2y+J., (u-x-
y el sistema de ecuaci ones para deter minar el piira melro /, y 1.:oor
dl!nadas de los posibles puntos de ex tremo
es decir,
1
- 2 cos y sen y+ J...= O, i
n . J
y- x--;-=0,
sen 2x= -'A.,
sen 2y=i.,
n
y-x=4,
De las ecuacionc!l (7) y (8) tenemos sen 2x + S<'ll 'ly =
2sen (x + y) cos (y - x) = O.
l7)
(8)
(9)
o, o sea,
(1 0)
Debido a (9). tenemos cos (y - x) = O y, or eso, <le (10)
resulta sen (x + y) = O, es deci r,
X + y kn, k = 0, J, 2, (l !)
Resolviendo las ecuaciones (9) y (11 ), tendremos
Determinamos las sei,:u11 das derivadas de la fundn tJJ (x, y):
cJ2<D )2(D
i)xZ = -2 cos 2x, i)x ()y =0,
CJ2<D
i}y2. = - 2 cos 2y.
(
kn n kn n )
En los puntos P-ii,
2
--
8
, z +s se tiene
24
CAP. J. EXTPEMO DE FUNCIONES
V - (tl>;,i)2 = 4 COS ( k!r. - : ) COS ( kn + ) =
= 2cos 2kn = 2> O.
Por consiguiente, hay condicionado en los puntos P11 - Adems,
para k = 2n es
a2a> 1 = - V 2 <o
iJxZ Pzn
y, por l!so, en los puntos Pin se tiene mxi mo condicionado siendo
.. -1+ -V2 .
"max- -2-
para X= 2n+ 1 es
o sea, en los puntos Psn+
1
tenemos mlnlmo condlclonado siendo
-V2
Zmt n= 1--2- .
En los problemas que siguen haHar el extremo condicio-
nado.
14. f = xy s iendo xi+ yt. = l.
15. f =x
2
+!f siendo + =l.
16. f = xyz siendo x +y + z = 5 y xy + yz + zx = 8.
17. f = ex
11
s iendo x +y= a.
18. f = 6 - 4x - 3y siendo x
2
+ y
2
= l.
19. f = x - 2y + 2z siendo x
2
+ y
2
+ z
1
= 9.
20. f = sen x sen y sen z siendo x + y + z = ; , x > O,
y> O y z> O.
21. Demostrar la desigualdad
l , e
2. EXTREMO CONDICIONADO
25
22. Hallar el valor mximo del producto xyzt de cuatro
nmeros no negativos x, y, z y t si la suma de los mismos
permanece constante: x + y + z + t = 4c.
23. Hallar la distancia mfnima del punto M (1 , O) a la
elipse 4x
2
+ 9y
2
= 36.
24. Hallar la distancia de la parbola y = xz a la recta
x -y = 5.
25. Hallar los lados del rectngulo de rea mxima
inscrito en Ja circunferencia x
2
+y"' = R'J.
26. Inscribir en la esfera de radio R el cilindro de mxima
superficie total.
Captulo 11
EXTREMO DE FUNCIONALES
3. Funcional. Variacin de una funcional y sus
propiedades
1. Definicin de funcional. Proximi dad de curvas. Sea M una
clase de funciones y (x). Si a toda f uncin y (x) E M le corresponde,
segn una regll, un nmt-ro determinado J se dice que en la clase M
est definida la funcional J y se escribe J = J {y (x)].
La clase M de funciones y (x) en la que es t defini da la funcional
J f y (x)] se denomi n3 e.ampo de definicin de la funcional.
EJEMPLO l. St>a M = e ro. IJ el conjunto dt' todas las [unciones
continuas y (x) definidas en el segmento [O, I] y sea
1
J (y (x}l = J y (x} dx I>. ( 1)
o
Entonces J (y (x)} es una funcional de y (x): a toda funcin y (x) E
E C [O, 1 J le corresponde un valor determinado J (y (x)]. Tomando en
(1) funciones concretas en lugar de y (x), obtendremos los valores corres
pon dientes de J 1 y}. Por ejemplo, si y (x) = 1, tenemos
si y (x) =ex, tenemos
1
J[l)= J l dx=I ;
o
1
J{ex= J
o
s y (x) = cos nx, tenemos
1
J(cosnx)= J cosnxdx= O.
o
1
) En adelante, al considerar funcionales in escribiremos
en el ntegrando y en de y (x), y' en l ugar de y' (x). y etc.
3.
FUNCIONAL. DB UNA FUNCIONAL
27
EJEMPLO 2. Sea M = C
1
[a, b] la clase de funciones y (x) que
tienen derivada continua en el segmento [a, bl y sea
J [y (x)l = y' {x
0
), donde x
0
E [a, bJ.
Queda cl aro que J [y (x)l es una fu ncional definida en la clase de
funciones seal ada: a toda funcin de esta clase le corresponde un
nmero determinado, el valor de la derivada de esta funcin en el
punto rij o x
0

Siendo. por ejemplo, a = 1, b = 3 y x
0
= 2, tenemos para
y (x) = x"'
J \x
2
] = 2x lx=:i = 4;
para y (x} = x
2
+ l encontr amos J {xi+ l I = 4 y par a y =
1 1 1
= In (1 + x) tendremos J {In (l + x)] = l + x x - Z = 3
EJEMPLO 3. Sea M = e l-1, 1j la clase de funci ones y (x) con
tinuas en el segmento [-1, l] y sea q> (x, y) una funcin definida y
continua para todos los -1 ::;;; x::;;; l y para todos los val ores reales
de y. Entonces,
1
J [y (x)] = j q> y) dx
- 1
ser una funcional def inida de la clase de funciones indicada. Por
ej emplo, si q> (x, y} =
1
para la funcin /1 (x) = x tendremos
1
X dx
J txJ = J i+x
2
=0 y para y(x)=l + x
-1
tendremos
1
r xdx
J [l -1- x)= J l +{l +x)
1
-1
ln Y5- arctg 2.
EJEMPLO . Sea M = e, [a, b) la clase de funciones y (x) que tienen
derivadas conti nua y' (x) en el segmento {a , bJ. Entonces
b
J fy(x})= f Vl +y'Zdx (2)
a
ser una funcional defi nida en esta clase de funciones. Desde el punto
de vista geomtrico, la funcional (2) representa la longitud del arco
de la curva y = y (x) cuyos extremos son los puntos A (a, y (a)) y
B (b, y (b)).
Se denomina variacin o incremento oy (x) del argumento y (x)
de la funcional J [y (x)l la diferencia entre dos funciones y (x) e Yo {x)
28
CAP. 11. EXTREMO DE FUNCIONALES
pertenecientes a clase considerada M de funciones:
6y (x) = y (x) - Yo (x)
1
) .
Para la clase de funciones k veces difercn riables tenemos
(6y) <ll> = 6y<li> (x).
Di remos que las curvas y = y (x) e y= y
1
(x) definidas en el
segmento la, bl san cercanas en el de proximidad de orden nulo
si es pequea rn la, b} la magni tud 1 y (x) - y
1
(x) 1- Desde el punto
de v isla esto significa qu(' son prximas las ordenadas
de dichas curvas en !a, bj,
Diremos la:> curvas y = y (x) e y= y
1
(x) defi nidas en el
segment o Ja, bl son cercanas en el sentido de proximidad de primer orden
s i son peq ueas en la. bl las magnitudes I y (x) - y
1
(x) 1 y
1 y' (x) - Y (x) 1 Desde el pun to de vi sta geomt rico, esto significa
que en la, bl son prximas t an t o las ordenadas de dichas curvas como
las direccione:> de sus tangentes en los puntos correspondientes.
Las curvas y = 11 (x) e y = y
1
(x) son cercanas en el sentido de
proximidad de k -smo orden si son pequeas en (a, b] las miignitudes
1 y (x) - !/1 (x) l. 1 y; (x) - Yi (x) l. . . .. 1 y<"-> (x) - yp
1
1 (x) 1.
Si las curvas son cercanas en el sentido de proximidad de k-simo
orden, con mayor razn lo sern en el sentido de proximidad de cual
qui er inferior.
L (
)
sen n'x r . t l d
EJEMPLO s a curva y x = n con n su 1c1en emen e gran e
y la cur\'a y
1
(X) r.. O son cercanas en (O, n ) en el sentido de proximidad
de orden nulo ya que
!
senn2x 1 1
lu (x)-yi(x) I = n
o sea, el valor al>soluto de esta diferencia es pequeo en todo el segmen
to [O, ni si n es su[ idenlemente grande.
No hay proximidad de primer orden ya que
1 y' (x) - Y (x) 1 = n 1 cos n'x 1
y, por ejemplo, l'll Jos r unlos x = tendremos 1 y' (x) - Y (x) 1 ""''
= 11, o sea, 1 y' (x) - y; {x) 1 puede resultar tan granlle como se quiera
si n es suidentement<.> grande.
EJEMPLO 6 . La curva y (x) =

con n s ufici entemente grande


y la curva y
1
(x) -:= O son cercanas en (O, nl en el sentido de proximidad
de primer orden ya que. tanto

sen nx l
I !I (x)-Yt (x) 1 =
112
<" /j2
1
) Para abreviar, en lo que sigue escribiremos simplemente 611
en lugar de By (x).
'a.
FUNCIONAL. VARIACIN DE UNA FUNCIONAL
29
como
, . ICOStlX I 1
1 y (.c} -yi(x) 1 = - n- < n
son pequeos.
Determinar el orden de prox imadad de las curvas en los
problemas que siguen.
27. y (x) =
cos nx
Yt (x) iii! O (0, 2nl.

e en
28. y(x) =
sen X
n
e y
1
(x) =O en [O, n] .
X
Yt (x) =O
(0, 1 ).
29. y(x)=sen- e en
11
Se denomina distancia entre las curvo,s y::: y (x) e y= y
1
(.x)
(a x b). donde y (x) e !Ji (x) son funciones cont inuas en [a, bJ,
el nmero no negativo p igual
al mximo del mdulo 1 Y1 (x)-
- 11 (x) 1 en el segmento
Q b:
P= P IYl (x), Y (x)] =
= mx lyi(x)-u(x)j.

y
M(l,I)
EJEMPLO 1 . Hallar l a ds-
tancia p entre las curvas
y (x) = x e !/l (x) = x
2
en el
segmen to [O, 11 (fig. 1). O )t
SOLUCIN. Segn la defini-
cin p = mh t r-x 1. osea,
Fig. 1
p = mx (.x - x'). La fun-
l
cln y= x - x' se anul a en los extremos del segmento {O, I].
Determinemos el mximo de la funcin y= x - x
2
en el segmento
[O, 1}.
Tenemos
de modo que
1
y' = l-2x e Y'=O para x=
2
En los problemas que siguen hallar fa distancia entre
las curvas en los segmentos indicados.
30
CAP. 11. EXTRF.MO DE PUNCIONAl, F.$
30. !J (x) = xrx e
Yt (x) == O en
[0, 21.
3 L y (x) = sen 2x e !Ji (x) =sen x en
[o.
32. y (x) = X e !11 (x) = In x en fe-
1
, eJ.
Suponga mQS que las curvas y= y (x) e y = y
1
(x) ti enen en el
segmento la, bl deri vadas conti nuas de orden n.
Se llama distancia de n-si mo orden entre las curvas y = y (x)
e y = YI (x) el mayor de los mximos de las expresiones
1 IJ1 (X) - y (x) , . 1 Yi (x} - y' (x) 1, .. ., l y\nl (x) - 11"" (x) 1
en el segmento [a, b). Representemos esta distancia as
Pn = Pn \ y t (x), IJ (x) ) = mx mx 1 y
11
) (x) - y01) (x) 1.

Desde este punto de vista se puede interpretar la distancia definida en
la pg. 29 como distancia de orden nulo.
EJF.MPLO 8. Hall ar la distancia de primer orden entre las curvas
y (x) = x' e y
1
(x) = x3 en el seg mento O x < 1.
SOl.UC.l\)N. Ca!cu!C?mos las derivad as de las funciones dadas:
y' (x) = 2x e Y. (x) = 3x
2
y consideremos las funciones y, (x) =
= x
1
- x3 e IJJ (x) = 2x - 3x
11
Determinamos sus mximos valores
en el segmento [O, lj. Tenemos y = 2x - 3x
2

y
Fig. 2
lgua lan<l o deri va da a cero, encont ramos los puntos estacio
narios de la funcin y
2
(x): x
1
= O y Xz = . !\ hora bien, Ys l.x,,.o =O,
y
1
I
2
= ..!. y el valor dl' y
1
(x) en el extremo de la derecha es
27
3
3. F UNCIONAi.. VARIACIN OE UNA fllNCIONAI.,
31
Ya (1) = O. Por eso,
Determinamos ahora la distancia Po de orden nulo entre las derivadas
y' (x) = 2x e Yi (.x:) = 3.x:':
Po= mx 1 Y3(x)1 = mx l 2x-3x
2
.

Consideremos el grfico de la funcin y,= 1 2x - 3x
1
1 {fig. 2). Puede
verse de l que Po = 1. Por consiguiente, la distancia p
1
de primer
orden entre las cur vas y (x) = .x:
1
e y
1
(x) = x:' se.r igul a
Pl = mx <Pe. Po)= l.
33. Hallar la distanc.ia de primer orden entre las curvas
y (x) = In X e Yi (x) = X en el segmento re-
1
, el.
34. Hallar la distancia de segundo orden entre las curvas
y (x) = x e y
1
(x) = -cos x en el segmento [O, J
35. Hallar la distancia de 1001-simo orden entre las
curvas y (x) = ex e y
1
(x) = x en el segmento [O, l J.
Se llama e-vecindad de n-simo orden de la curva IJ = y (x)
x b) el conjunto de las curvas y = y
1
(x) cuyas distancias de
ns imo orden a la cur va y = y (x) son menores que e:
Pn = Pn [y (x), Y1 (x)) < 6.
La 6-vecindad de orden nulo se denomina e-vecindad fuerte de la
funcin y = y (x}.
La e-\ecindad fuerte de la curva y = y (x) est formada por todas
las curvas comprendidas en la franja de 2e de anchura construida a
partir de la curva y= y {x).
La &-vecindad de primer orden se denomina e-vecindad dbil de
la funcin. !I = y (x).
2. Continuidad de una funciona l. Una funcional J (y (.x)) definida
en la clase M de funciones y (x) se ll ama conJnua en y = Ye (x) en
el sentido de proximidad de n-simo orden si cualquiera que sea el
nmero e > O existe un nmero TJ > O t-al que la desigualdad
1 J [y {.x)J - J (y
0
(x)J 1 < & se cumple para todas las funciones admi-
sibles y = y (X), o s(!a, para todas 1 as funciones que satisfacen las
condiciones
1 Y (x) - !lo (x) 1 < 11 1 y' (x) - Y {x) 1 < TJ, .
.. 1 y<ni {x) - ut"> (x) 1 < 'l
En otras parabras, si se tiene 1 J [y (x)l - J l!le (x)} 1 < e siempre que
rr1 (y (x), !lo (x)l < TJ
32
CAP. TI . EXTREMO oe FUNCIONALES
Toda runcional que no sea continua en el sentido de proximidad
de nsimo orden se denominar disconlintla e.n este seJ1tldo de proxi
mldad. Poniendo
y<I>.> (x) = {x) + ct(l)<k> (x) (k = O, 1, 2, ... , n),
donde a es un parmetro y w (x) es una funcn cualquiera de la clase
M, podemos persuadirnos de que
lm y ' h} (x) = (x) (k = O, 1, 2, .. , n)
a. ... o.,
y, por eso, podemos definir la continuidad lle Ja uncional J fy (x)]
en !J = Yo (x) de la forma siguiente
lm J (Yo (x} + CXCI) (x}) = J {Yo (x)).
ci ... o
EJEMPLO 9. Demostrar que la funci onal
t
I'
J {y (x)l = (!I +2y') dx
o
considerada en el espacio C
1
{O, 1 (es continua en la funcian !lo (x) = x
en el sentido de proximidad de primer orden.
sotucrON Tomemos un nmero cualquiera e >O y demostremos
que existe un nmero TI > O tal que 1 J ly (x}l - J (x) < e siempre
que 1 /1 (x) - x 1 < 'l y 1 y' (x) - l 1 < 1}. Tenemos
1
IJ [y(x)J- J [xJl =I J
o
t 1
J Jg-xJdx+2 J ly'-1 ldx.
o o
e
Tomemos Tl=T Entonces, para todas las funciones y (x)E CdO, ll
tales que
e e
)y(x)-x)<T e l!l'(x)-11<3
tendremos
1 J [y (x)) - J [x) <s.
Es decir, para todo s >O existe un nmero 11 > O (por ejemplo,
TI = ; ) tal que l J {y (x)J - J [x] l < e siempre que Pt fy (x), xj <
<TI Pero, segn la definicin, esto significa precisamente que nuestra
funcional es continua en la funcin !lo {x) = x en el sentido de proxl
1 8. FUNCIONAL. VARIACIN DE UNA FUNCI ONAL
mldad de primer orden. Es fcil ver que esta funcional es continua e
el sentido de proximidad de primer orden en cual quier curva y (x)
E e, (O, l).
EJEMPLO 10. Consideremos la funcional
I [y (x)I = y' (x
0
),
donde las funciones !I (x) E e, [a, b] y Xo E [a, b).
Esta funcional es discontinua en el sentido de proximidad de
orden nula en cualquier funcin y (x). Efectivamente, escogemos cp (x)
de modo que q>' (x
0
) == 1 y que 1 <p (x) 1 < 11 en el segmento (a, b }.
Consideremos la funcin y (x) = Yo (x) + <p (x), donde !Jo (x) E
E e, la, b). Entonces tendremos y' (xo) = !I (Xo) + 1. Es obvio que
p (y (x), Yo (x)} < t, o sea, que las curvas y (x) e !lo (x) son cercanas en
el sentido de proximidad de orden nulo. Al mismo tiempo J (y (x)l -
- J [y
0
(x)) = 1, es decir, Jos valores de la funcional no son prximos
por cercanos que sean, en el se.nlido de proximidad de orden nula, los
argumentos y (x) e Yo (x).
Hablando con ms precisin, existe un e > O (por ejemplo, cua l
quier a < 1) tal que para cualquier 'l >O existirn funciones y (x)
para las cuales
Po [y (x), Yo (x)J < 11 y, sin embargo, 1 J [y (x)} - J IYo (x}) 1 s.
Esto significa precisamente que la funcional J (y (x)) es discontinua
en el sentido de proximidad de orden nulo.
Demostremos que esta funcional es continua en el sentido de
proximidad de primer orde t.
Tomemos un e > O cualquiera. Tendremos
1 J (y (x)] - J IYo (x)) = l 11' (xo) y ~ (Xo) J.
Queda <:laro que lomando 11 = e tendremos
1 J [u (x)J - J IYo (x)J 1 < e
siempre que Ps [y (x), y
0
(x) J < 'l que es lo que se querla demostrar.
Este ejemplo permite ver que de la continuidad de la funcional en el
sentido de proximidad de n-simo orden no implica, hablando en tr
minos generales, su continuidad en el sentido de proximidad de orden
Inferior.
EJEMPLO 11. Consideremos 1 a funcional
R
J (g(x)] = J ''J.dx
o
definida en el espacio C
1
{0, n). Demostremos que es discontinua en la
funcin Yo (x) == O en el sentido de proximidad de orden nulo.
. sennx
En efecto, sea !lo (z);;: O en [O, n] y sea !Ir> (xJ = --. Enton-
n
l
ces, Po (Yo (.x), !In (x}) =- de modo que Po ~ O cuando n ~ oo.
n
3-()1387
34
CAP. 11. E XTREMO DE FUNCIONALES
Por ofro lado, la dHerencia
11
\ cos2nx d n
J lYn (,")\ -J (Yo (x)]= J n x=2
o
no depende de n. Es decir, J Yn (x)) no tiende hacia J [g
0
(x)) = O
cuando n ..... oo y, por consigui ente, nuestra luncional es discontinua
en l a funcin Yo (x) = O en el senti do de proximidad de orden nulo.
Proponemos al lector demostrar que esta runcional es continua
en la funci n y
0
(x) = O en el sentido efe proximidad de primer orden ..
Analizar la continuidad de las funcionales sguientes
36. J ly (x)J =y (xo). donde y (x) E e [a, b] y Xo E [a, b],
en el sentido de proximidad de orden nulo
37. J ly (.x)f = mx 1 y (x) I, donde y (x) son funciones
continuas en el segmento la, b], en el sentido de proximidad
de orden nulo.
38.
{
tl si y (x) toma al menos un valor negatvo,
J (y (x)] = + si y (x) ==O,
1 si y x ) ~ O e y(x) O ,
en el sentido de proximidad de orden nulo
1
39. J (y (x)] = ) 1y'1 dx, donde las funciones y (x)
o
tienen primera derivada continua en el segmento [O, 11:
a) en el sentido de proximidad de orden nulo;
b) en el sentido de proximidad de primer orden.
11
40. J lY (x)J = ) V l + y'
2
dx en la funcin Yo (x) == O,
o
donde y (x) E C
1
[O, 11]: O
a) en el sentido de proximidad de orden nulo;
b) en el sentido de proximidad de primer orden
re
4 t. J fy (x)l = J (1 + 2y'
2
) dx en la funcin Yo (x) e= O,
o
FUNCIONAL. VARIACIN DE UNA FUNCIONAL
35
donde y (x) E Ct. (0, n), en el sentido de proximidad de
primer orden.
EJEMPLO 12. Demostrar que la funcional
l
J[g(x}]= J xVl+y2dx
o
definida en el conjunto de las funciones y (x) E C [O, l] es continua
en la funcin y
0
(x) = x2 en el sentido de proximidad de orden nulo.
SOLUCJON. Pongamos y (x} = x2 + CU] (x), donde T) (x) E e (O, l]
y a. es tan pequeo como se quiera, tenemos
1
J [y (x)] =1 [x2 + art (x)J = J x3 Vl + (x2+ar)2dx =
o
1
= J x3 V 1+x' + 2ax2r +a.2f12 dx.
o
Pasando al limite cuando o:-+ O, obtenemos de esta igualdad
t
lfm Jig(x)J= J xV l+x'dx=l[x2J
a-o.O O
lo que equivale a la continuidad de la funcional en la funcin
Yo (X)= x.
DE.PlNICiN. Sea M un espacio lineal normado formado por las
funciones y (x).
La funcional L [y (x)) definida en el espacio M se denomina
lineal si satisface las condiciones
1) l {cy (x)} = el (y (x)J,
donde e es una constante cualquiera y
2)
L IY1 (x)J + y" (x)J = L l.!lt (x)J + L (y
2
(x)J,
donde Y1 (x) E M e !Jz (x) E M.
Por ejemplo, la funcional
b
L [y(x))= J (U' + y)dx
a
definida en el espacio C
1
fa, bJ es, obviamente, lineal.
AP. lf . EXTREMO DE FUNCIONALES
Existe otra definicin de funcional lineal:
La funcional L [y (x) 1 se denomina lineal si 1) es continua y 2) satis
lace la condicin
L YJ (x) + Yz {.%) ) = L (Y1 (x)] + L [y, (x)]
cualquiera que sean
1
(x) E M e !/t (x) E M.
42. Demostrar la equivalencia de las dos definiciones de
funcional lineal.
43. Demostrar que la funcional L [y (x) J = y (x
0
) es lineal.
44. Sea L [(y (x)J una funcional l ineal. Demostrar que si
, l (y(x)J . r 1 -
la razon llY(x)JI-+ O cuando 11 y (x) 11 ~ O es L y (x) =O.
3. Variacin de una funcional . Sea J [y (x)J una funcional defin
da en el conjunto M de funciones y (x). La magnitud
ll J = 61 (y (x)) = J [y (x) + By) - J fy (x)]
(6y = y(x) - y (x), donde 1J (x) E M e y (x) E M)
se denomina incremento de la funcional J {y (x)) correspondiente 111
incremento 6y del argumento.
EJEMPLO u . Hallar el incremento de l a funcional
t
J [y {x)) =: J yy' dx
o
definida en el espacio C
1
[a, bj si y (x)' = x e !h (x) = x'.
SOLUCIN. Tenemos
t t t
M=J(x2)-J[x]= J x2'2xdx- J xl dx=) (2xS -x) dx=O.
o o o
45. Hallar el incremento de la funcional del ejemplo 13
siendo y (x) ~ e Y1 (x) = l.
DEFJNIClN. Si el incremento de l a funcional J [y (x)]
l:!.J = J [y (x) + 6yJ - J [y (x}]
se puede representar en la forma
M = L [y (x), c5y) + (y {x), 6y) 11 By 11.
donde L {y (x), 6yl es una funcional lineal respecto a 6y y ~ (y (x), 6y)-
- O cuando U 6y U - O, entonces la parte del incremento lineal res
pecto a 6y, o sea, L [y (x), 6yJ, se llama fJariacin de la funcional y se
FUNCIONA L. VARIACJON DE UNA FUNCIONAL
37
representa por 6J. Se dice en este caso que la funcional J [y (x)) es
di f erenciabfe en el punto y (x).
46. Demustrese que la variacin &J de la funcional
J [y (x)l se determina unvocamente (si es que existe).
EJEMPLO 14. Demustrese que la funcional
b
J [y (x)] = ) !I d:z
a
definida en el espacio C la. b] es difercnciable en todo punto !I (x)
de este espacio.
SOLUCION. Tenemos
6.J=J lv <x>+oyJ - / fy(.rlJ=
b b b
= J v + ~ . v d:r - r yd.x= J av ci.r.
a a o
b
Es decir, !11 = J f>y dx. Pero sta es una funcional lineal res pecto
a
a 6y. Todo el incremento se ha reducido en nuestro caso a una funcional
lineal respecto a 6y. La funcional consderada es diferenciable en todo
I>
punto y (x) y su variacin es 6J = J 6y dx.
a
47. Demostrar que toda funcional J ly (x)l lineal continua
es siempre d iferenciable.
EJEMPI.O 1s. Demostrar que la funcional
b
J tu {x)l =) y'l.dx
a
definida en el espacio C [a, bJ es dilerenciable en todo punto y (x).
SOLUCIN. Tenemos
b b b b
61= J (y+au)dx-) udx= J 2y0ydx+ J {Oy)idx. (3)
o a a a
En el (lltlmo miembro de (3) la primera integral representa la funclo
nal lineal respecto a 6y cual quiera que sea la funcin fija y (x). Estl
38
CAP. 11. EXTREMO DE FUNCIONALES
memos la segunda integral de este miembro. Tenemos
b I>
J (y)'l. dx = J 1 g 2 dx
a a
b
mx Jaul)z dx=(b-a)(l611 IF=((b-a)ll6yllJlf6y1J.
J
a
Si 11 6y 11 - O, la magnitud
(b - a) 11 6g 11 -i- O.
Es decir, hemos logrado representar el incremento !lJ de la funcional
como la suma de L !y {x), oyl y de una magnitud infinitslma de
segundo orden con respecto a 11 6y U. Segun nuestra definicin, la
funcional considerada es dferenciable en el punto g (x) y su variacin es
b
61=2) y(>ydx.
a
t
48. En la funcional J (y (x)J = J y'l.dx tomar y = 2x
o
y 8y = axz.; comparar 8J y l:!i.J para a = 1; - 0, l y 0,01.
1 .
49. En la funcional J [y (x)J = J x!/dx tomar y = tr y
o
y = ax; comparar 6/ y 6J para a = 1; 0, 1 y 0,0 l.
50. Analizar si son o no diferenciables las funcionales
siguientes:
1) J [y (x)l = y (a) en el espacio e la, bl.
2) J ly (x)J =y (a) en el espacio c. ra, b}.
3) J [y (x)l = V 1 + y''I. (a) en el espacio C
1
[a, bl.
4) J {y (x}J = 1 y (a) 1 en el espacio e [a, bJ.
51. Demostrar que la funcional J [y (x)J es diferenciable
si to es J [y (x)]. Hallar la variacin de /
2
[(y (x)J.
52. Sea F (x, y) una funcin continua de sus argumentos
con derivadas parciales continuas hasta de segundo orden
inclusive en el recinto x b, - oo <Y<+ oo. Demos
trar que la [uncional
b
J y(x)l= J F(x, y) dx
o
3. FUNCIONAL VARI ACIN DE UNA F UNCIONAL
39
definida en el espacio C (a, lr] es diferenci able y que su varia-
cin es
b
<'J = r y) oydx.
V y
a
EJEMPLO 16. Consideramos l a funciona!
b
J (y (x)J = J F (x, y, y') dx
a
definida en el espacio C
1
[a, b) de funciones y (x) que son continuas
en el segmento [a, b) y (Ue t ienen en l derivada continua de primer
orden. La funci n F y, y' ) es conti nua respecto a todos sus argu
mentos y tiene derivadas parciales continuas hasta de segundo orden
inclusive en el recinto
x b, -oo <y< +oo, -oo < y' < +oo.
Determinamos el incremento AJ de l a funcional correspondiente al
Incremento y del argumento siendo y E C
1
ta, bl. Tenemos
b
AJ (y (x))= 5 (F(x, y+6y, y'+y'}-F(x, y, y'))dx. (4)
e
Segn la frmula de Taylor
f(x, g+y, t/+/)- F(x, y, y')='
= oy+ 6y' -1- R (x, y, y', y, tly'), (5)
donde R (x, y, y'. 6y, y') es el trmino complementario de la frmul a
de Taylor. Introduciendo (5) en (4), obtenemos
b b
liJ {!! (x)] = J ( hu+;:, o/) dx+ J R (x, y, y', hy') dx. (6)
a a
El pri mer sumando en el segundo miembro de {6) es l ineal respeclo a
6yly y'. Suponft.'amos que las segundas deri vadas parciales de la
fundn F
1
.(x, y, y ) respecto a y e y' no pasan, en valor absoluto, de una
const ante M >O en un recinto acotado respecto a y e y'. Tendremos
entonces
b b
J J R (x, y, y', 6y , 6y') / dx < 2M J /J y 1/
2
dx= ZM (b-a) 11 y /1
2

o a
40
CAP. 11 . EXT'Rl:MO DE FUNCJONALES
donde 1\ 8y 1\ = i;i;t (1 oy 1. 1 6y' 1 ). Por consiguiente, el segundo
a . ~ x . ~ b
sumando del segundo miembro de (6) es una infinitslma de segundo
orden respecto a 11 Oy 11- Es decir, en v irtud de la definicin, \a funcio-
nal J [y (x)J es dlferenciable en el espacio C
1
(a, b) y su variacin es
b
OJ = J ( ~ Oy+ : Oy' ) dx
o
EJE'-'PLO 17. H ali ar la variacin de la funcional
1
J (y (x)] = J (y'ev+xy
2
) dx.
-t
(7)
SOLUCION La funcin F {x, y, y')= y'e11 + xy
2
es, evidentemente,
conti nua respecto a todas las derivables x, y e y' en conjunto y sus
derivadas parciales de cualquier orden respecto a y e y' son acotados
en cual quier recinto acotado de variacin de y e y' . Por esto, la funcio-
nal considerada es diferenciable en C
1
(-1, 1) y, segn la frmula (7),
su vari acin es

IH = ) ((y' ell + 2xy) Oy + e116y' 1 dx.
-t
53. En la funcional
'
J [y (x}J = j (y' y + xy'
2
) dx
t
k(x-'l)
tomar y = In x y f>y = --;=:--- ; comparar /J.J y BJ para
k= 1; 0,1 y 0,01.
54. En la funcional
1
J [y (x)] = J (x
2
g'
2
- !f) dx
o
tomar y = x?. y 6y = kx
8
; comparar J y fJJ para k = 1;
0, 1 y 0,01.
n
55. En la funcional J [y (.t) I = 5 yi sen x dx tomr y =
o
=sen x y fJy = k cos x; comparar J y 6J para k = -1;
0,3' y 0,03.
'3
FUNCIONAL. VARIACION DE UNA PUNCIONAL
41
56. Las derivadas de segundo orden de la fun
cin F (x, z
1
, z
2
, ... , zm+
1
) respecto a todos los argumentos
son continuas en el recinto a =::;;;; x =::;;;; b y - oo < Z1t < + oo
(k = 1, 2, .. ., m + 1). Demostrar que la funcional
b
J l!I (x)] = J F (x, y, t', .. , !/"'') dx
Q
es diferenciable en el espacio Cm la, b] y que su variacin es
b
6J = J [ :: 6y + ::. 6y' + ... +
0
;,:> oym> J dx.
<J
4". Segunda definicin de la variaci n de: una funciona!. Se llama
variacin de la funclonal J [y (x)] en el punto y = y (x) el valor que
toma en a= O la derivada de la funcional J (y (x) + ciayJ (considera
da en tanto que funcin de ci) respecto al parmetro :
(J
6J = <Jtz J lY (x)+a6yJ 111<-o.
Si existe la variacin de la funcional en tanto que parte principal
lineal de su incremento (o sea, si existe la variaci n en el sentido de
la primera definicin), tambin existe In variacin en tanto que valor
en a = O de. la derivada respecto al parmetro ci y ambas variaciones
coinciden.
EJEMPLO 18. Empleando Ja segunda definicin, ballar la variacin
de la funciona\
b
J [g (x)J = J Ji'- dx.
ll
SOLUCICN. La variacin de esta lunconal en el sentido de la pri
mera definicin es
b
fJJ = 2 J 11 6y dx
a
(vase el ejemplo 15). Determinemos la variacin de la funcional
J [y (x)) basndonos en la segunda dellnicin. Tenemos
b
J(v(x)+a.6y)= J (y+a.By)2dx.
o
42 CAP. JI. EXTREMO DE FUNCIONALES
Por eso,
b
J fy(x)-t-croy)=2) (y+ay)gdx
o
de modo que
b
J =

J {y (x) + la- o= 2 J yy dx,


Q
Las variaciones de la funcional en el sentido de la prime.ra y de
la segunda definiciones coinciden.
Para las funcionales que siguen hallar, en los espacios
correspondientes, sus variaciones en el sentido de la segunda
definicin.
b
57. J [ y (x)l = J (x+ y) dx.
b
58. Jty(x)]= J (y
2
-y'
2
)dx.
Q
1
59. J (y (x)] = y2 (O)+ j (xy+ y'
2
) dx.
o
lt
60. J [y (x)l = ) y' sen ydx.
o
b
6t. J[!/11 !Jz, ., Ynl= \ F(x, Y1 Yz, !1111 y;,
" a
.. , dx,
donde F es una funcin continua de sus argumentos y sus
deri vadas parciales respecto a todos los argumentos son
cont inuas en un recinto acotado O de variacin de los mismos.
OBSERVAC!N. La segunda definicin de la variacin de una fun
cional es en cierto sentido ms amplia que la primera pues existen
lunclonales que tienen variacin en el sentido de la segunda definicin
aun cuando no se pueda despejar la parte prindpal lineal en el ncre
mento de las mismas. Para explicarlQ rec1mlremos a las funciones;
3. FUNCIONAL. VARIACIN DE UNA FUNCIONAL
43
en este caso nuestra afirmacin equivale a que Ja existencia de las
derivadas en cualquier direcci n no basta para la existencia de la dife
renclal de la funcin.
Sea
f (x, Y)
xy
donde p y cp son l as coordenadas polares del punto (x, y). Las derivadas
parciales existen en todo punto y son iguales a cero en el ori gen
de coordenadas; sin embargo, no existe la diferencial df en el ori gen
de coordenadas. Efectivamente, supongamos que df existe. En este
caso el gradiente de la funcin f seria igual a cero en el origen de coor
denadas y, por eso, Ja derivada df O) en cualquier direccin tambin
seria igual a cero. Pero es fcil persuadirse de que
df (O, O) 1
2
dl = 2 sen cp
lo que, en general, es diferente de cero. Agul cp es el ngulo entre el
vector l y el eje Ox.
5. Segunda variacin de una funcional. Una funcional J [x, y)
dependiente de los elementos x e y (que pertenecen ambos a un espacio
Ji rieaJ) se denomina bilineal S es Una funci onal lineal en r para X
fij o y una funcional lineal en x para y fijo. O sea, l a funciona J [x, y)
es bili neal si
J (a1xt + Gt2JCs, y) = a
1
J [X, y] + a,) [x2. y],
l (x, + P2flal = j}/ [x, y] + [x, Y2l
Poniendo en la funci onal bilineal y = x obtenemos la expresin
J {x, x] llamada funcional cua drtica.
Toda funcional bllineal definida en un espacio de dimensin
finita se denomina forma bilineal.
Una Suncional cuadrtica J (x, x) se denomina defini da positiva
si J [x, x] > O cual qui era que sea el elemen to no nulo x.
Por ejemplo,
1) la expresin
,,
Jfx, y]= J A{l)x(l )g(i)dt,
(1
donde A (/) es una funcin continua fija, representa una funcional
b
bilineal en el espacio C la, b] mientras que la expresi n 1 A (t) x (t) dt
(l
representa, en este mismo espacio, una funcional cuadrtica que resu lta
definida positiva si A (t) > O para todo t E (a, b);
CAP. ll, EXTREMO DE FUNCfONALES
2) l ' expresin
b
J (x, x) = J (A (/) x2 (/} +B (t) x (f) x' (/)+e (t) x'2 (t)] dt
a
es un ejemplo de una funcional cuadrtica definida para todas l as
funciones pertenecientes al espacio C
1
[a, bJ;
3) la expresin
,, b
J [x, Yl = } ) K (s, t) x (s) y (t) ds dt,
a a
donde K (s,t) es una funci n fija de dos variables, representa una tun-
cional bilineal en e la. b).
DEFINICION. Sea J fy (x)) una funcional definida en un espado
lineal normado. Diremos que la funcional J [y (x)l tiene segunda
variacin si su incremento 6J = J (y (x) + 5y] - J (y (x)) puede
ser representado en la forma
1
t..J =La


donde L
1
(6yJ es una funcional lineal, L
2
[6y] es una funcional cuadr
tica y -+ O cuando \1 6y \1 -+ O.
La funcional cuadrtica L
2
[8y) se denomina segunda variacwn
{o segunda diferencial) de la uncional J [y (x)] y se designa por 6'-J.
La segunda va riacin de una funcional se determina unlvocamente
(si es que existe).
EJEMPLO 19. Hallar Ja segunda variacin de la funcional
1
J IY (x)) = (xy2+y') dx
definida en el espacio C
1
[O, 11 de las funciones y (x).
SOLUCIN Tenemos
6J = J (y (x) + 6yJ- J [y (x}] -=-
1
= J [x (11+6g)2+(.y' +6g')'-xy2-y'3J dx=
o
1
=) [2xy6y+x (6y)2+3g'20/ +3y'
o
t 1 '
= J (2xy6y+3y'26y'}dx+ J (.x{6y)2+3y'(6y')2Jdx+) (8)
o o o
FUNCIONAL- VARIACIN DE UNA FUNCIONAL
45
Fijemos y (x). El primer sumando del ltimo miembro de (8) ser
entonces una funcional lineal respecto a 6y; el segundo sumando de
este miembro ser una funcional cuadrtica; finalmente, el ltimo.
tercer, sumando de este miembro puede ser estimado asi
1 1 1
1 J (6y')Sdx 1)
2
J J 6y' 1 dx ( J l 6y' 1 dx) ll 6y 11
2
o o o
(la norma se toma en el sent ido del espacio C
1
(0, 1)), o sea, podr ser
representado en la forma 11 (jy 11
11
donde - o cuando 11 &y n - o.
Por definicin, para nuestra funcional existe la segunda variacin
6
2
J igual a
1
6?.J=2 J (x(liy)2+Sy'(6y')2)dx.
o
62. Ha!Jar la segunda variacin de una funcional cuadr-
tica.
63. Hallar la segunda variacin de la funcional eF(11i
siendo F (y) una funcin dos veces diferenciable.
64. Demostrar que las funcionales de t ipo
b
J [y(x)] = J F (x, y, y') dx
o
definidas en el espacio C
1
[a, b) son dos veces diferenciables
si la funcin integrando F tiene derivadas continuas hasta
de tercer orden inclusive. Hallar la frmula para la segunda
variacin.
Consideremos la funcin CD (a.) = J [y (x) + aOy]. La segunda
variacin 6't.J de la funcional J (y (x)] se define tambin mediante el
valor de la segunda derivada de la funcin <D (a.) en el punto a.= O:
a2<D (a.) 1
da.2 ti-O.
Para las funcionales de tipo integral (que predominarn en nuestras
consideraciones) ambas deriniciones coinciden.
Hallar la segunda variacin.
b
65. J [y (x)J = F (x, y, y', ... , y1m>) dx.
el
46
CAP. 11. l!XTRP.M.0 DE FUNCIONALES
66. J (z (x, y) 1 = J f P (x. y, z, Zx, zu) dx dy.
G
I>
67. J lY Y21 , !/nl =) F(x, Yt . . . ' !lr11 y;,
a
... , dx.
6. Ext rtmo de una lunc:lonal. Condicin necesar ia de extremo.
Diremos que la funci onal J (y (x)] alcanza su mzlmo en la curva
y = y
0
(x) si los valores que toma la funci onal J [y (x)l en cualesquiera
curvas prximas a y = Yo (x) no son mayores que J [y
0
(x)). o sea , si
M = J [y (x)} - J (Yo (x)l O.
Si 6.J O y .J = O slo para y (x) = y
0
(x), di remos que se
alcanza mximo estricto en la curva y = y
0
(x).
Anlogamente se define la curva y = y
0
(x) en l a que se alcanza
un mltilmo. En este caso se tiene /;.J ;;;i, O para todas las curvas prxi
mas a l a curva y = Yo (x).
EJEMPLO 20. Demostrar que la funcional
1
J (y (x)I = J (xz+ y2) dx
o
alcanza mini mo estricto en la curva y (x) e O.
SOLUCION. Cualquiera que sea la funcin y (x}. continua en [O, I],
tenemos
1 t t
M = J (y (x)] - J [O]= J (x2 + y2) dx - J xz dx = J y2 dx :;i.. O;
o o o
adems, el signo de Igualdad se da slo para y {x) = O.
FUERTE y D2BIL. Diremos que la funcional J (y (x)J
alcanza su mdxlmo relativo fuerte en l a curva 11 = y, (x) si
J ly (x)] J i!le (x)}
para todas las curvas admisibl es y = y (x) pertenecientes a una e-vecin-
dad de orden nulo de la curva y = y
0
(x). Anlogamente se define el
minimo relativo fuerte de una funcional.
Diremos que la funcional J [y (.x)} alcanza su mdximo relativo dbil
en la curva y = Yo (x) si
J [y (x)) J [Yo (x)l
para todas l as curvas admisibles y = y (x) pertenecient es a una e-ve
cindad de primer orden de la curva y = y
0
(x). Anlogamente se define
el nnimo relativo dib!l ck una funcional.
3. FUNCIONAL. VARIACIN DE l iNA FUNCIONAL
47
Los mximos y mnimos (fuer tes y dl>iles) de la funcional
J (y (x)l se denominan extremos relat ivos.
Todo extremo fuerte es al mismo tiempo ext remo dbi l pero no
viceversa.
El extremo de la funcional J (y (x)l referente a la totalidad de las
funciones en las que est deinida la funcional se denomina extremo
absoluto.
Todo extremo absoluto es al mismo tiempo ex tremo relat ivo fuerte
y dbil pero no todo extremo relativo ser extremo absolut o.
EJF.MPLO 21. Cons i cJeremos 1 a funciona 1
fl
J (y (x)I = ) yi (1 - y'2) dx
o
en el espacio de funciones y {x) E C
1
[O, n) que satisfacen la condicin
y (O) = y (n) = O. En el segmento (O, n} del eje Ox hay minimo dbil
de J. En electo, tenemos J = O si y = O; por otra parte, para las curvas
perte.necientes a una e-vetindad de primer orden .de este segmento,
donde e es cual quier nmero positivo menor que l , se tiene 1 y' 1 < J
de modo que el integrando es positivo para y =FO y, por consiguiente,
la funcional se anula slo si y = O. Es decir, la lundonal alcanza
mni mo dbil en la curva y= O.
Mnimo fuerte no hay. Basta tomar
En este caso
1
y (x) = -V sen nr
n
J l y(x)) = J se11211x(l-ncosZnx) d.x=
o
= _!_ r sen2nxdx - .!.. r
n. J 4 J 2n 8 '
o o
o sea, J <O en estas curvas si n es suficlcnlemente grande. Por otra
f
arte, siendo 11 suficientemente grande, todas estas cur vas se encuen-
ran en una vecindad tan pequea como se quiera de orden nulo de la
curva y = O. P or consiguiente, no se alcanza mlnimo fuerte en y = O.
EJEMPLO 22. (Weierstrass). Consideremos la funcion al
1
J[y{x))= J x'-y'2dx; y (-1)=- I , y ! l) =I.
- 1
En el segmento [ - 1, 1) tenemos J [y (x)] y, adems, J [y (x)} = O
slo si y' (x) = O, o sea, si y (x) = C = const. La funci n y (x) = e
pertenece a \a clase C
1
l - 1, 1] de las funciones que tienen primera deri-
vada continua en t>I segm('nto (-1, I], pero 110 satisface l as condiciones
de frontera da das. Por consiguien te, J [y (x)] > O parH todas l as
(unciones y (x) E C
1
[-l, 1 l que satisfacen las condiciones y ( -1) = -1
48
CAP. J) . oe PUNCIONAt. BS
e !I (J) = J. En otras palabras, la funcional tlene cota inferior pero
sta no se alcanza en las curvas y (x) E C
1
(-1, 11. Efectivamente,
consideremos la familia monoparamtrica de curvas
X
arctg-
a.
Ya. (x) = l ' a. > O.
arctg-
n
Todas ellas satisfacen las condiciones de frontera: Ya (-1) = -1 e
Ya. (1) = 1. Pasando al lmite para a. - O, obtenemos la Funcin
{
-1, si -1
y(x) = O, si x = O,
+ l , si 0 < X 4' l.
es decir, y (x) = sg x (flg. 3). Esta funcin pertenece a la clase de
funciones diferenciables a trozos en el segmento (-1, 1).
-t
Tenemos
t
J (!la. (X)}= J
-t
y
Flg. 3
cr.x2dx
1=
(a.2 + .x2) arctg2 -
a
1
2a. r
i J
arctg2 -
0
a:
Y""YoJx)
2

1
( 1-a. arctg .
arctg2 -
a.
3. PUNClONAL. VARIACIN DE UNA F UNCIONAL
49
Queda cl aro que J (x)J -+ O cuando a. -+ O. En la funcin l mite
y(x), que satisface tas condiciones de frontera!/'(-!)= -1 e y (1 ) :c..,,
= 1, el valor de l a funcional J [y (x)/ es igual a cero: J Y (x)} = O.
Por consiguiente, la funcional J IY (x)) alcanza su val or mni mo
en la cur \'a y (x) = sg x que pertenece. a la clase de funciones dieren
d ables a trozos en el segmento (-1, IJ pero no pertenece a la cl ase
C1 (-1, l].
(condicin necesar ia de ext remo de la funcional). Si la
funcional diferenciable J [y (x)l alcanza su valor extremo en la curva
y= y
0
(x), donde y
0
(x) es un punto interior del campo de defnicin
de la funcional, entonces en y = Yo (x) se tiene
f>J [Yo (x)] = O. (9)
Las funciones para las cuales 6J = O se deno minarn f unciones
estacionarias.
Hallar las ecuaciones funcionales para las funciones esta-
cionarias de las funcionales que siguen, empleando la condi-
cin necesaria de extremo (9) y los lemas fundamentales del
Clculo variacional.
, b
68. J [qi (s)J = J J K (s, t) q> (s) cp (t) dsdt +
a a
b b
+ J cp
2
(s)ds-2 J 'fl(s)/ (s)ds,
Q Q
donde K (s, t) es una funcin continua simtrica de s y t en
{
a< s < b}
el recinto D a .::;;. t b , f (s) es una funcin continua
en fa, b] y cp (s) es el argumento funcional continuo incgnito.
+oo
69. J [ q> (x) J = J [p (x) q>'
2
(x) + 2qi (x + l) X
-oo
X q> (x- 1 )- cp
2
(x)-2cp (x) f (x)J dx,
donde el argumento funcional q> (x); es continuo y tiene deri-
vadas continuas a trozos en todo el intervalo - oo < x <
< + oo, p (.x) tiene derivada continua y f (x) es continua.
70. J [ cp (x)J = r [p ( x) q>'
2
(x) + q (x) q
2
(x) -2cp (x) f (x) 1 dx,
4-01387
50
CAP, 11. DEiPUNCIONALES
donde p (x) tiene derivada continua, q (x) y f (x) son conti-
nuas y el argumento funcional q> (x) tiene dos derivadas
continuas.
4 Problema elemental del Clculo variacional.
Ecuacin de Euler
Supongamos que la funcin F (x, y, y') tiene derivadas parciales
continuas hasta de segundo orden inclusive respecto a todos sus argu
mentos.
El problema elemental del Clculo variacional es el sigui ente:
entre todas las funciones y (x) que tienen derivada continua y que
satisfacen las condiciones de frontera
1J (a) = A e 11 (b) = B
hallar la funcin que ofrece extremo dbil a la runcional
b
J 111 (x)) = J F (x, y, 11') dx.
(1)
(2)
En otras palabras, el problema elemental del Clculo variacional
consiste en hallar el extremo dbil de la funcional de tipo (2) en el
conjunto de todas las curvas suaves que unen dos puntos fijos P
1
(a, A)
y P,. (b, B).
TEOREMA 1. Condicin necesaria
1
) para que Ja fu ncional (2), defi
nida en el conurito de todas lat. funcioties y = (x) que tienen derivada
continua y que satisfacen las condiciones de frontera (1), alcance su valor
extremo en la f unctn y (x) es que e.sfa funcin verifique la ecuacin
de Euler
(3)
Las curvas integrales de la ecuacin de Euler se denominan extre-
males (o curvas de Lagrnnge).
En forma desarrollada la ecuacin de Euler da
v" (x} F
11
'v' -1- y' (x) Fyy + F:r.
11
,- Fy = O (F
11
,
11
, .::6:0)
y representa una ecuac;n <lHerencial de segundo orden de modo que
su solucin ' general comprender dos constan tes arbi trarias cuyos
val ores se determinan. hablando en trminos generales, de las condi
ciones de frontera (1) .
La funcional (2) puede alcanzar extremo slo en las cxtrcmales que
satisfacen las condi ciones (l ).
1
) Esta condicin es necesaria para el extremo dbil. Como quiera
que todo extremo fuerte P.S al mismo ti empo un extremo db i l, cual-
qu ier condicin necesaria para el extremo dbil t arnbin ser nt:ccsarla
el extremo fue r te.
s 4. PROBLEMA ELEMENTAL
51
El problema de contorno
d }
Fv-- F . = 0
dx 11
y(a) =A, y(bl=B
no siempre tiene sol ucin y si la solucin existe, puede no ser nica.
EJEMPLO 1. 1.En qu curvas puede alcanzar su extremo la funcional
2
J[y{x))= J (y'2 -2xy)dx; y(l}=O, y{2) ~ - 1 ?
1
SOLUCION. Aqu tenemos F (x, y, y') = y"J. - 2xy de modo que
la ecuacin de Euler da y+ x = O. Su solucin general es
x3
JJ = - 6 +C1x+C2.
Utilizando las condiciones de frontera, obtenemos para C
1
y C'l el
seguiente sistema de ecuaciones lineales:
C1+C2= ~ }
2c, +c2 =
6
.
De aqu resulta C
1
= ! y C
2
=0. Por consiguienie, el ext remo puede
alcanzarse slo en la curva
X
Y=5{I -x2).
EJEMPLO 2. Ha ll ar las extremales de la funcional
3
J [y (x)) = J (3x - y) .1/ cix
l
que satisfagan las condiciones de fron tera y (1 ) = 1 e y (3) = 4 ~ .
SOLUCJN. La ecuacin de Euler es 3x - 2y= O, de donde y = ; x.
La extrema! IJ = ~ x no satisface la condkin y (I} = 1 y, por eso,
nuestro problema variacional no tiene solucin.
EJEMPLO 3. Hallar las extremales de la funcional
2it
Jly(x)]= i (Y''-y2)dx
o
que satisfagan las condiciones de front era y (O) = l e IJ (2n} = l.
4
52
CAP. 11 . EXTREMO DE FUNCIONALES
sOLUClON. La ecuacin de Euler tiene la forma y+ g = O; su
solucin general es
y = C
1
cos x + C
9
sen x.
Utilizando las condiciones de frontera, encontramos
y = COS X + C sen X,
donde Ces una constante arbitraria. Es decir, el problema variaci onal
considerado tiene un conjunto tnlinito de soluciones.
Hallar las extrernales de las funcionales siguien-tes.
o
71. J [g(x)] = I (12xy-y'2)dx; y(-1) = 1, y(O) = O.
-1
2
72. J{y(x)l = ) (y'
2
+ 2yy'+y
2
)dx; y(l)= 1, y(2)=0.
1
1
73. J[y(x)l= jYY(l + y'z)dx; y(O) = y(l)=y
2

o
1
74. J[y(x)l=) yy'
2
dx; y(O)= l, y(l)=:-14.
o
n
75. J(y(x)l=) (4ycosx+y'
2
-y
2
)dx;
o
y (O)= O, y (n) =O.
1
76. J{y(x)= J (y'
2
-yi-y)e'l/rdx; y(O)=O, y(l)=e-
1

o
1
77. J[y(x)] = J (y'
2
- 2x.y)dx; y(- 1)= -1, y(l)=l.
-1
o
78. J (y (x)] = J (y'
2
-2xy) dx; y ( -1) =O, y (0) = 2.
- 1
e
,.
79. J [y (x)} = j (xy'
2
+ yy') dx; y(I) =O, y (e)= J.
1
Pl OBLEMA ELEMENThL
53
La ecuacin de Euler (3) para la funcional (2) es una ecuacin
diferencial de segundo orden y, por eso. la solucin y = y (x) de 13
ecuacin de Eul er debe tener segunda derivada y" (x). Sin embargo,
se dan c.asos en que 1 a (uncin que o[rec.e el extremo a la funcional
b
J {y (x)) = F (x, y, y') dx no tiene segunda derivada.
a
EJEMPLO -1. La funci onal
1
J ly\:c))= J y2 (\-y')2 dx
- 1
con las condiciones de frontera
y (-1) = O e y( \)= 1
alcanza su valor m nimo, igual a cero, en la funcin
( )
_ { O si x O,
V X - , O
X SI x> .
Aun cuando l a runcin u (x) no tiene segunda derivada, satis[ac.e la
ecuacin de Eul cr correspondiente.
Efectivamente, t enernos F (x, y, y') = y
2
(1 - y' )2 y, poniendo
y = v (x), obtenemos la ecuacin de Euler
2u(l - v' )2+ !, (2u2 (t - v')] = 0. (4)
Pero, seg n la definicin de la funcin u (x), en (-1, 1) t enemos
P , = -2u
11
(1 - u') = O y, por consiguiente, tambin dd F , = O;
,, " ,,
o sea, a pesar de que l a ecuacin de Eul er (4) es ormalme.ntc de segundo
orden y a pesar de que u (x) no ex is le, la ecuacin de Euler se convierte
en identidad al sustituir en ella v (x).
TEOREMA 2. Sea y= y (x) solucin de la ecuacin c Euler
d
Fy-dXF
11
. = 0.
Si la funcin F (x, y, y') tiene derivadas parciales continuas hasta de
segundo orden inc/usiue, entonces la funcin y = y (x) tene segunda
derivada continua en todos los puntes (x, y) para los cuales
Fyy [x, y (x), y' (x)] :6=0.
COROLARIO. La extremal y = y (x) puede tener puntos angulares
slo en aquellos puntos en los que F yu = O.
Asi, en el ejemplo 4 tenemos que FY'Y' = 2y
2
se anula en los
puntos del eje Ox; la extrema} t iene punto. angular en x = O.
TEOREMA 3. (Bernst en). Supongamos que en la ecuacin
u = f (x, y, y')
54
C,\P. 11. E.'n.REMO DE FUNCIONALF. S
las funcones f, f 11 y f , son continuas en lodo punto finito (x, y) para
cualquier valor fi nito de y' y supongamos que exisle una constante k > O
y unas fu nciones
a = a (x, y) > O y ~ = ~ (x, y) > O.
acotadas en cualquier porcin finita del plano, tales que
f y (x, y, y' ) > k y 1 f (x, y, y') 1 ~ a..y'
2
+ (6)
Entonces, por dos cualesquiera puntos del plano (a, A) y (b, 8) de abscisas
distintas (a ;:f= b) pasa una curva integral y = rp (x) de la ecuacin (6),
y slo una.
EJEMPLO 5. Dcmostrnr que por dos cualesquiera puntos del plano
de a bscisas distintas pasa una extrema! ni ca de la funcional
J{y(x\j "'"" J c-
2
llt(y'2 - \)d,'(.
sOLUCION. La ecuacin de Eulcr para l a funcional considerada es
y# = 2y (1 + y'
2
)
de modo que se puede a plicar el teorema 3. En efecto, tenemos en
este caso
f (x, y, y') = 2y 11 + y'
2
) y fy = 2 (1 + y'
2
) > 2 = k.
Adems,
1 f (x, y, y') 1 = 1 2y (1 + ~ 1 ~ 2 1 y l y'
2
+ 2 1 y 1,
o sea, a = ~ = 2 1 y 1 -;;;, O.
EJEMPLO s. Demostrar que no hay extrema! de la fu ncional
J (y(x)J= J (yi+ Vi -l y'2) dx
que pase por dos cualesquiera puntos del plano de abscisas distintas.
sOLUCION. La ecuaci n de Euler tiene la forma
3
y" =2g{l + y'2)2 (7)
y el teore.ma 3 no se puede apli car ya que no se cumple l a segunda de
las condiciones (6) (debido a que f (x, y, y') crece, respecto a y' , ms
rpido que la segunda potencia de y'). Las condiciones del teorema 3
son de carcter suficiente. O sea, si estas condiciones no se cumplen, de
ello no se puede deducir que no hay extrema! que pase por dos puntos
cual esquiera de abscisas diferentes. Demostremos que por los puntos
A (O, O) y B { ; , 2) no pasa ninguna ex trema) de la funcional con
siderada-
Tomando en la ecuacin (7)
y' = P e
dp
y = P dy ,
4.
obtenemos
PROBLEMA ELEMENTAL

dp -
p - = 2y ( ( + p)Z 2 ,
dy
o sea, pdp
3

(1 +Pz)2
I ntegrando, encontramos
de modo q ue
1
----=y2-C, o
0+P2
(C- y
2
} Vi + y'2= 1
dy VI-(C-y2)2
-;;= C-y2
sea,
55
donde C es una constante real. Separando las variables en l a ltima
ecuacin e integrando del punto A al punto 8, obtenemos
2
1 r C-y
2
- = l dy
2 . Vl-(C-yZ)2 .
o
(8}
Cualquiera que sea el n mero real C, el denominador del integrando de
(8) ser complejo en cierto inter valo (a, e: (O, 2) de variacin de
l a variable y . Por consiguiente, la Igualdad (8) es imposible. Esto quiere
decir que no se puede trazar extrema! alguna por los puntos A (O, O)
2).
80. Demostrar que por dos cualesquiera puptos del plano
pasa una y slo una extrema! de la funcional
J [ !I (x)l = J JI 1 + y2 + y'
2
dx.
EJEMPLO 7. Demostrar que t oda ecuacin
y" (x} = f (x, y , y' ) (9)
es ecuaci n de Eul er para cierta funcional
r
J fy (x) } = J F (x, y , y' ) d."<.
1) Cmo se determina la funcin F (x, y, y ') a partir de la funcin
f (x, y, y' )?
2) Hallar tod<1s l as funcionales cuyas extremales son las rect as
y= C1x + C2.
SOLUCIN. Busq uemos la funcional cuy a ecuacin de Euler
Fy - Eyx-F u'uY' - FvvY" =0
56
CAP. II. EXTREMO DE FUNCIONALES
coinci da con la ecuacin (9). Es decir, debe cumplirse la identidad en
x, y e y'
F
11
-F
11
,x-F
11

11
y' -F
11
,
11
.f (x, y, y');;; O.
Derivando respecto a y', obtenemos
F 11 '11' x+ F111111Y
1
+ F111111f.+FY'11'f 11 , =0.
Tomando u = F
11
,
11
, , obtenemos para la funcin u
deriva das parciales:
una ecuacin en
au + , <Ju +f ou f o
rJx Y ay ay' + 11 u =
(IO)
Por la bsqueda de la funconal, o sea, de la funcin
F (x, y, y ), se reduce a la integracin de la ecuacin lineal en deriva-
das parc-iales (10) y a la cuadratura sucesiva.
Consideremos la segunda cuestin. En este caso la ecuacin de
Euler debe ser y" = O y para la funcin u se obtiene, de acuerdo con
(10), la ecuacin
Integremos esta ecuacin.
,2!!;__0
" u au - .
La ecuacin de las caracterlsticas tiene la forma
dx dg dy'
-1 =y;-=o-
Integrando este sistema, encontramos
y'= C1 e y= C1x + Ch
( 11)
de don de C
9
= y - xy'. Por eso, la solucin general de la ecuacin
(11) es
u (x, y. y') = <I> (y' , y - xy' ).
donde ID es una funcin ar bitraria di ferenciable de sus argumentos.
De aqul
:i
F(x, y, z) = a(x, y) t J (z-t) <l> (t, y-tx)dt,
o
donde a (x, y) y (x, y) son funciones arbitrarias de sus argumentos
que cumplen la relacin

O!J = ox .
Se puede ver de la solucin que existe una cantidad infinita de proble-
mas variacionales que tienen l a ecuaci n (9} como ecuacin de Euler.
PROBLEMA ELEMENTAL
CASOS ELEMENTAi.ES DE INTEGRACIN DE J.A ECUACIN DE EULER.
10. F no depende de y', o sea, F = F (x, y}.
En este caso la ecuacin de Euler tiene la forma
5
F
11
(x, y)= O. (12)
La solucin de esta ecuacin finita (no diferencial) no contiene ele
mentos arbitrarios y, por eso, no satisface, hablando en trminos gene
rales, las condiciones de frontera y (a) = A e y (b) = B.
Slo en casos excepcionales, cuando la curva (12) pase por los
puntos de frontera (a, A) y (b, 8), existir una curva en la que podr
alcanzarse el ex tremo.
EJEMPLO s. Hallar las extremales de la funcional
"
2
J (y (x)I = J y (2x -y) rlx; y (O) = O, y ( ) = .
o
SOLUCJON. La ecuacin de Euler tiene la forma 2x-2y = O, o sea,
y= x. Puesto que las condiciones de frontera se satisfacen, la integral
. :n
2
J y (2x - y) dx puede alcanzar su extremo en la recta y = x. Para
o
otras condiciones de frontera, por ejemplo, y (O) = O e y ( )= 1,
la extrema! y= x no pasar por los puntos frontera (0, O) y ( , 1)
de modo que el problema variacional con estas condiciones de frontera
no tendr solucin.
20. F depende de y' en forma lineal, o sea,
F (x, y, y')= M (x, y)+ N (x, y) y'.
En este caso la t>cuacin de Euler tiene la forma
aM aN
a - iJx = O.
Igual que en el caso 1, Ja ecuacin obtenida es finib:t y no diferencial.
En general, Ja curva determinada por Ja ecuacin ..,M - J aN = O
uy X
no satisface las condiciones de frontera y, por consiguiente, el
problema variacional no tiene, como regla, solucin en la clase de
funciones continuas. Por otra parte, si en un recinto D del
1 O t
. oM iJN o 1 ., F ( ' )
p ano x y se 1ene Ty - a; = , a expres1on :<, y, y =
""' M (x, y) dx + N (x, y) dy es una diferencial total exacta y la
58
CAP. 11 . EXTREMO DE FUNCIONALES
funcional
b (b, B)
J (y (x)J = J F (x, y, y') dx= J (M ttx+N dy)
a (a, A)
no depende dt'l camino seguido en la integracin: el valor de la fundo
na( J (y (x)l es el mi smo en todas las curvas admisibles. El problema
variaconal carece de sen ti do.
EJEMPLO 9. Analiiar el extremo de l a funcional
b
J(y(.x)J= J (y2+2xyy')dx; g{a) = A, y(b)=B.
a
SOLUCIN. Aqu F depende de y' en forma li neal. Tenemos
aA1 aN oM _ oN =
0
oy =
2
Y, ax =
211
y oy ax -
o sea, el integrando (y
2
+ 2xyy') dx es una diferencial exacta. Por
consiguiente. la integral no depende del camino seguido en la integra
cin:
b (ti, B J
J [y (.x)l = J (y2 dx + 2x dy) = J d (xy2) = xy2 1::: = bB2- aA2
a (11, A)
para cual quier curva de integracin y (x) que pase por los puntos (a, A)
y (b, B). El problema variacional carece de sentido.
3. F depende slo de y' . o sea, F = F (y').
La ecuacin de Euler ti ene la forma
Fy y!/ = O.
En este caso las extremales son t odas las rectas posibles
y= C
1
x + C3,
donde C
1
y C
2
son constantes arbitrarias.
EJEMPLO 1 o Hallar las ex tremales de la funci onal
b
Jf y(x)}= J Vl+y' 3dx; y (a) =A, y (b)=B.
a
Esta funcional determina la longitud de la curva que une los puntos
(a. A) y (b, 8 ). Desde el punto de vista geomtri co, el problema con
siste en hallar la curva de longitud mnima que une dos puntos fij os.
SOLUCJN. La ecuacin de Eul er t iene la formar/'= O. Su sol uci n
general es
4. PROBLEMA
59
La extrema\ que salis[ace las condiciones de y (a)= A e
y (b) = B es, ob\'iamente, la recta que p<i:>a por los puntos (a, A)
y (b. 8) :
y
B-11

4. F no dE'pende dt" y, o sea, F "= F <x, y' ) .
En esk caso la ecuacin de f. ul er t>s :X F!I' (x. y') : .... o. de donde
resulta
siendo C
1
una const ante arbi traria.
La ecuacin {13) es una ecuacin di ferr.ncial <le pr i mer orden.
lnh!grndola, enc:ont ra mos las extrcmales del prohlema.
EJEMPLO 11. E11trl las curvas que unt:n los puntos A (1, 3} y
8 (2, 5) hall ar la curv a en la que puede alcama r su c.:dr l' mo la luncional
2
J 111 ) y' (1 + x?.y') clx.
1
SOLUCi N' . Puesto 4ue F no dl"pcndl' de y, la <cuacin de Eulcr
tiene la forma : x F.y' (x, O, o sea,

(l \ 2x2y' ) =0 de donde
1 +2x2y' = C1.
E t
, C1 - t d q , e d d
n onces, y= Q.xZ de mo o que Y=-- -
1 2
, ou e
1-C
1
2
. Por consiguiente, las ex trema les represent an una
e* -
1 -
famtl ia
de hiprbolas. De terminemos la extrcmal que pas:i. por los puntos
fjos. Para hall ar los valores de l as constant es Cf y C
2
ormamos el
sistema
de donde res u Ita cr = - 4 y C z = 7. La ex trema 1 buscada es y =
=7-.!.
X
5". F no depende explcitamente de x, o sea, F = F (y, y') .
En este caso la ecuacin de. Eul er tiene la for ma
Fy - Fyy' Y' - F11' 11'!1h = 0.
60
CAP. 11. EXTREMO DE FUNCIONALES
Multipli cando ambos miembros de esta ecuacin por y', obtendremos
en el primer miembro la derivada exacta :x (F- y'Fv> o sea, la
ecuacin ser :i> (F- y' F
71
,) =O, de donde
F- y'F
11
, =C1 (14)
siendo C
1
una constan le arbitraria. Esta ecuacin puede ser integrada
resolvindola respecto a y' y separando las variables o introduciendo un
parmetro.
EJEMPLO 12 (cuerpo de resistencia mnima en un fluido). Deler
minar la for ma del cuerpo slido que, al moverse en un flui do de gas,
y
del fluido
/(
do gas
Fig. 4
encuentra resistencia mnima. Para simplificar consideraremos el
cuerpo de revolucin (fig. 4).
sOLUCIN Suponiendo que la densidad del gas es suficientemente
pequea y que las molculas, al chocar con la superficie del cuerpo, se
reflejan de forma especular, obtenemos para la componente normal de
Ja presin
p = 2pvll sen
2
e.
donde p es l a densidad del gas, v es la velocidad del gas respecto al
cuerpo y fl es el ngulo entre la velocidad y su componente tangencial.
La presin es perpendicular a la superficie de modo que la componente
segn el eje Ox de la fuerza que acta sobre un anillo de anchura
(1 + y') l/2 dx y de radio y (x) se puede representar en la forma
1
df =2pu2 sen2 0 [2ny (1 + y'2)
7
I sen O dx.
4 PROBLEMA ELEMENTAL
61
La fuerza resultante que acta en la direccin positiva del eje Ox
es igual a
l 1
F = ) 4npo2 sen3 6y ( 1+y'2)2 dx.
o
Supongamos, para simplificar el problema, que
Entonces, la fuerza de resistencia ser igual a
l
F= 4n:pu2 J y'3y dx.
o
(15)
El problema consiste en hallar la funcin y (x) en la que F alcanza
su valor menor posible siendo
y (O) = O e y (l) = R.. (16)
La ecuacin de Euler para la funcional (15) tene la forma
y'-3 ::e (yy'2.)=0. (17)
La solucin particular y= O de esta ecuacin debe ser rechazada en
virtud de las condiciones de frontera (16). La ecuacin (17) puede ser
representada as! :
y'
3
+ 3yy'y = o. (18)
Multiplicando p,or y' ambos miembros de (18), vemos que el primer
miembro es (y
3
y)'. Integrando, encontramos
y'Sy = q,
de donde resulta
, _ c1 e -f
y - V!i e y=( 1x+ C2} '.
Utilizando las condiciones de frontera (16), obtenemos
4
R.3
Ci = -
1
- y Ca = O
de modo que
62
CAP. 11. EXTREMO DE FUNCIONALES
o sea, el contorno con ex tremas fijos que corresponde a l a resistencia
mnima del cuerpo es una parbola de grado ! .
4
EJEMPLO 13. Hallar la extremal de la funcional
b,-
r V l +y'?.
J y (x)] =-- J !I dx
Cl
que por dos puntos fijos (a, A) y (b, B) pertenecientes al semipl ano
supenor.
SOLUCJON. Puesto que la funcin integrando no contiene explci-
tamente x, la ecuacin de Euler, segn (14). da
Vr+Y'2
y
Despus de simplificar, encontramos y lf 1 + y''j = e\, donde C1 =
1
= Ct. Integrando la ltima ecuacin, encontramos (x + CiJi +y'=
= Cf, o sea, una familia de circunferencias con centros en el eje Ox.
La extremal pedida ser la que pase por los puntos fijos. El problema
tiene solucin nica ya que por dos puntos cualesquiera del semi plano
superior pasa una y slo una semicircunferencia con centro en el eje Ox.
OBSERVACJN. Segn el principio de Fermat, el camino que recorre
un rayo de luz al propagarse con la velocidad u (x, y) en un medio bidi-
mensional no homogneo constituye una extrema! de la funcional
J
xi lf--::y;2
J [y (x)) = (J (x, y) dx.
xo
Como hemos visto en el ejemplo anterior, si la velocidad de la luz t1
es proporcional a y, los rayos de luz representan arcos de circunferencias
con centros en el e.je. Ox.
Sea q una curva. Denominaremos longitud ptica de la curoa q
el tiempo T (q) que se precisa para recorrerla al moverse segn esta
curva con la velocidad de la luz t1 (x, y}.
Supongamos que el semiplano superior y > O es un medio ptico
en el que. la velocidad de la lui en todo punto es igual a la ordenada del
mismo: v = y. Como hemos visto, los rayos de luz en este medio sern
semicircunferencias con centros en el eje Ox. Se puede demostrar que,
si uno de los extremos del arco AD de la semicircunferencia q se halla
'!n el eje Ox, su longitud ptica es infinita (ig. 5). Por eso, diremos
que los puntos del eje Ox estn en el infinito. Consideremos que 1 as
semicrcunfer<.>ncias con centros en el eje Ox son rectas, que las longilu
des pticas de los arcos de estas semicircunferencias son las longitudes
de dichas rectas y que los ngulos entre las tangentes a las semicir-
cunferencias en el punto de interseccin de las mismas son los
4. PROBLEMA ELEMENTAL 63
entre dichas rectas. Obtenemos una Geometra plana en l a que se
conservan muchas proposiciones de la Geometra habitual. Por ejem
plo, por dos puntos fij os se puede trazar una recta y slo una (y a que
o
X
Fig. S
por dos puntos del semiplano se puede trazar sl o una semicircunfe-
fenci a con cent ro en el eje Ox). Dos rectas se consi deran paralelas si
ti enen un punto i nfinito comn (o sea, si las dos semic ircunferencias
y
111
o X
Fig. 6
son tangentes en un punto B pertenec:ientt' al eje Ox). Entonces, por
todo punto A que no se hal!e en la recta q se pueden trazar dos rectas q,
y q
2
paral<'las a q. Liis rectas que pasan por el punto A v que se encuen-
tran en los ngulos vt>rtkales l y l ll, cortan la recta q mientras que las
rectas que se encuentran en los ngulos Il y IV no la cortan.
Hemos obtenido el modelo de Poincar de la Geomet ra plana de
Lobachevski (lig. 6).
64
CAP. Jl. l!XTREMO DE FUNCIONALES
Hallar las extremales de las funcionales:
b
81. J[y(x)]= S [2xy+ (x
2
+ e")y']dx; y(a) =A,
a
y(b)=B.
t
82. J(y(x)l= J (e
11
+ x/)dx; y(O)=O, y(l)=a.
o
ztJ /4 ( . 112
83. J [y (x)l = (y'2-y
2
) dx; y (O)= 1, y T ) = 2
o
1t
84. J [y(x))= } (y'
2
-y2)dx; y(O)= 1, y(n) = -1.
o
1
85. J lY (X)l = J (x+ !/
2
)dx; y(O) = 1, y( l ) = 2.
o
1
86. J[y(x)J = J (y
2
+ y'
2
)dx; y(O)=O, y(l)= l.
o
i
87. J[y(x)I= f (y'
2
+4y2)dx; y(O)=e
2
, y( l )=l.
o
1
88. J[y(x)]= J (2'-y
2
)dx; y(O)=l, y( l) =e.
o
b
89. J [y (x)] = J (xy' + y'2) dx.
(1
"
90. J (y(x)] = J (Y+ Y;) dx.
a
91. Demostrar que no tiene extremos la funcional lineal
b
J[y(x)) = J [p(x)y' +q(x}y+r(x)] dx,
Q
donde p (x) E c
1
la, bl, q (x) E e [a, bl y r (x) E C [a, bl ,
PROBLEMA ELEMENTAL
92. Consideremos la funcional
"
J[y(x)]= J F(x, y, y')dx
a
65
con las condiciones de frontera y (a) = A e y (b) = B. Demos
trar que la ecuacin de Euler subsiste al agregar al integrando
F (x, y, y') dx Ja diferencial tot al de cualquier funcin u =
=u (x, y).
b
93. J r y (x)l = J (y
2
+ y'
2
+ 2yex) dx.
a
94. J [y(x)J ='Y (!2-y'
2
-8ych x) dx;
1)
y(0)=2, y ( =2ch
95. Hallar las extremales de la funcional
b
J (y (x}I = J xny'
2
dx
a
y probar que para n 1 no existen extremales que pasen
por dos puntos situados a distintos lados del eje Oy.
PROBLEMAS VARIACIONALBS EN FORMA PARAMETRJCA. En muchos
problemas es cmodo, y a veces Imprescindible, emplear l a representa
cin paramtrica de las lineas
X= <p (/), }



Y=\ll (t),
donde <p (t) y ip (1) son funciones continuas con derivdas continuas
a trows siendo, adems, ( )
2
+ ( '!':' )
2
4= o.
Consideremos la funcional
t1
le= J F(f, x, y,;, y)df = J F(t, x, y)dt, (19)
e
dx dy
donde x =- e y= df

CAP. ii. EX'tR.eMO DE lftJNCIONALES
Para que los valores de la funcional (19) dependan slo de la
lnea, y no de su paramelrizacin que puede efectuarse de distintos
modos, es necesario y suficiente que la funcin integrando no contenga
explcitamente el parmetro t y sea positivamente homognea de grado
uno respecto a Jos argumentos x e y:
F (x, y, h, ky)=kF (x, 11, x, y), k> O.
Por ejemplo, en la funcional
Je = J x dy - y dx
e
la funcin Integrando es positivamente homognea de primer grado.
Efeclivamente, tenemos
F (x, y. ;, g) = y y ~
y es obvio que
F (x, y, k;, ttY) =kF (x, y, x, y).
Si Ja curva C
ofrece el extremo a la funcional Je en la clase de lineas C que unen los
puntos fijos (x
0
, y
0
) y (x
11
y
1
), las funciones q> (t) y 'i> (t) satisfacen
las ecuaciones de Euler
F, -: (F;)=O, }
F v- -;(F. ) =O.
11
Una de las ecuaciones (20) es consecuencia de Ja otra.
(20)
Las ecuaciones de Euler se pueden representar en la forma de
Weiersfrass
F.-F.
-=
r
%1/ 11%
3 '
(21)
F (;2+y2)2
donde res el radio de curvatura de la extrema! y F
1
es el valor comn
de las razones
P F .. F ..
%X 1/1/ %1f
F1=-- = --=--
yi ;z _;9
f 4. PROBLEMA LEMENtAL
EJEMPLO 14. Hallar las extremales de la funcional
(XI lft)
Je= J y2g' 2 dx.
(0, O)
SOLUCIN. Puesto que puede haber extrema les que se cortan con las
rectas paralelas al eje Oy en ms de un punto, consideraremos el pro-
blema en forma paramtrica.
o n i ~ n d o x = x (t) e y= y (t), encontramos que la funcin
y2 .
lrifegrando tiene la forma y' -;-- x, o sea, es posi t ivamente homognea
.x2
de primer grado respecto a ~ e g.
La primera de las ecuaciones (20) da
de donde
..!. ( g1. yi ) = o
dt '
xz
Y' ( 2 )
2
=Cf.
Integrando la ltima ecuacin, encont ramos
y
1
= 2C1x + C2.
Puesto que l a extrema! debe pasar por el origen de coordenadas,
1
tenemos C2 = O. La segunda condicin de rrontera da C
1
- !J;,,
o sea, en definitiva,
EJEMPl.O 1s. Hallar las extremales de la funcional
t1
Je= J lV x2+2+a2<x.V-yx)J dt.
to
SOLUCIN. Poniendo
F(x, y,;, u) = V;s+i +a(xy-y;),
vemos que l a funcin F es positivamente homognea de primer grado
respecto a ~ e y. Empleemos las ecuaciones de Euler en la forma de
s
68
CAP. 11. EXTREMO DE PUNCIONALES
Wcierstrass. T
F.=- -a2
l/X
y
F
=
F1 =---:-- = ----3-
g?.
Por eso, la ecuacin (2 1) tiene en nuestro caso la forma
l
-=2a2,
r
Es decir, la curvatura .!.. de la extrema! es constante. Por lo tanto, las
r
extremales son arcos de circunferencias; en particular. se tienen cir-
cunferencias completas si
x(lo) = x(t1), }
!I (lo)= Y (t l)
Hallar las extremales de las funcionales:
(x1, 1/f) ,
96. Je= } gz-yzxz di.
(0, 0) )C
(l, 2)
97. Je = J !/
2
-3el//X;c'I. di.
(0, O} X
(1, O)
98. le= J (K


( - 1, O)
donde K >O es una constante.
S Generalizaciones del problema elemental
del Clculo variacional
J. funcionales que dependen de derlvedes de rdenes superiores,
Supongamos que se tiene la runclonal
XI
J [.Y (x)J= S F {x, g (x), y' (x), ... , g<n> (x)J dx, (1)
:o
donde F es una funcin diferenciable n + 2 veces respecto a todos los
argumentos e y (x) E C
11
[x
0
, x
1
], y supongamos que las condiciones de
(rontera tienen la forma
!/:("o) = !IO Y (xo) = Y, , y< n-1> (xo) = ,;,,n- i), }
(2)
!/ (xl)=Yit !I' (x1)=111'. ,y<
1
H> (X1)=y\n-o.
GENERALIZ /\CIONES DEL PROBLEMA ELEMENTAi.
69
Las extremales de la funcional (1) con las condi ciones (2) son las
curvas integrales de la ecuacin de Euler - Posson
d d2 dn
Fv - Tx fv, + dx2 Fii#- . . . +(- t)n dxn f11<n> =0.
EJEMPLO 1. Hallar la extrema! de la funcional
1
J {y (x))- J (360x2y - ywZ) dx;
o
y (O) = O, y' (0) = 1, y (1) = o, y' (1) = 2,5.
SOLUCJN. La ecuacin de Euler - Poisson tiene la forma
tf'l
360x
2
+ dx
2
(-2y")=0, o sea, y
1
v = 180x2,
y su solucin general es
t
11 =2 x
6
-I C1x
3
+C2xZ+ C3x+ C
4

Empleando las condiciones de [rontera, encontramos
3
C1 =
2
. C
2
=-- -3, C;i=l y C
4
- 0.
La extrema! pedida es
1 3
u=c x6+ 2 x3-3x2 + x.
Consideremos el caso cuando en l a frontera no se dan todas las
condiciones (2) sino un nmero de las mismas de modo que, despus
de emplear l as condiciones de frontera, en la solucin general de la
ecuacin de Euler - Poisson contienen lodavia constantes arbitrarias_
Para resolver este problema es preciso hallar l a vari acin de la funcio-
nal (1), transformarla t omando en consideracin las condiciones de
frontera dadas y obtener condiciones complementarias l' n la frontera
igualando Ja variacin a cero.
EJEMPLO 2. Hallar la curva y = y (x) que ofrece valor ex trema! a
la funciona\
b
J (Y (x)J = T J y 'l dx
o
con las condiciones
y (a) = O e y (b) '= O.
SOLUCIN. La ecuacin de Euler - Poisson tiene la forma.
f/ IV = 0.
(3)
(4)
70
CAP. 11. EXTREMO DE PUNCIONALES
Su solucin general
y = C1 + C,.x + C
3
x2 + C, x3 (5)
contiene cuatro constan les arbitrarias Ct (i = J , 2, 3 y 4) y l as con-
diciones de frontera (4) no bastan para determinarlas. Por eso, como
hemos explicado, calculamos la vari acin de la funcional (3). Tenemos
b
6J = J y(jy dx.
CI
Integrando (G) por partes dos veces, obtenemos
b
BJ =y" (x) (jy' (x) 1: - J y"'fJy' d."'=
'1
(6)
b
=y" (x) 6y' (x) 1: - y"' (x} 6y (x) 1: + J yIYfJy dx. (7)
a
La expresin (7) debe anularse en la extrema} y (x) de la funcional
(3). Debido a la arbitrariedad de Ja funcin 6y, resulta que y
1
v = O;
sta es la ecuacin de -Euler - Poisson para la funcionar (3). Pero si
l a Integral del timo miembro de (7) se anula, la expresin. de frontera
fy" (x) fJy' (x) - y"' (x) 6y (x)J J:
tambin debe ser igual a cero idnticamente. Puesto que f>y (a) =
= y () = O (extremos fijos), resulta que debe ser
y" (b) 6y' (b) - y" (a) fJy' (a) = O.
En virtud de la arbitrariedad de las magnitudes 6y' () y fJy' (a},
obtenemos necesariamente
y" (a) = O e y" (b) = O. (8)
Las condiciones (8) conjuntamente con las condiciones (4) determinan
unvoca mente la extrema! en la familia (5): y = O.
2. Funciona les que dependen de m funciones. En el caso de una
funcional que depende de m funciones y
1
(x), Ya (x), ... , Ym (x)
x1
J (Yt. Y21 . , Yml = J F (x, Ytr J/2, , J/m. Y, y;, .. , y;,.) dx
y con las condiciones de frontera de t ipo
Yh (xo) = y ~ IJk (Xi) = Yl (k = 1, 2! ... , m)
p. GENERALIZACIONES DEL PROBLEMA ELEMENTAL
71
las extremales se determinan del siguiente sistema de ecuaciones dife-
renciales de segundo orden
d
F - -F , =0 (k= 1, 2, .... m) f.91
vk dx Vtt
que se denomina sistema de ecuaciones de Euler.
EJEMPLO 3. Hallar las extremales de la funcional
2
J [y (x), z (x))=) (g'2+ z2 +z'2) dx
t
con las condiciones de frontera
y (1) = 1, y (2) = 2, z ( 1) = o, z (2) = l.
SOLUCcN. En este caso el sistema de ecuaciones (9) tiene la forma
y =o, }
z-z" = O.
Resolviendo este sistema, encontramos
y = C1x + C2 Y z = C
8
ex + C ~ e : c .
En virtud de las condiciones de frontera, tenemos
1 e2
C
1
=1, C2=0, C3 = -
2
- - y C1t.= - --
e -1 e2-J
de modo que la extrema! pedida
Y=X, }
sh (X-1)
Z = shl
es una curva alabeada que constituye Ja interseccin de dos superfici es
ci 1 lndricas.
si
EJEMPLO " HaJlar las extremales de la funcional
"
J fy (x:}, z {x)J = J (2yz-2y2 + y'2-z'2) dx
o
y (O) = O, y (n) = I, z (0) = O y z (n) = -1.
sOLUClON. El sistema de ecuaciones (9) tiene Ja forma
y" +2y- Z= , }
Z'+y=O,
de donde, eli minando la funcin z, obtenemos
YIV + 2y + y= 0.
La solucin general de esta ecuacin tiene la forma
!J = C
1
cos ~ + C
1
sen x + x (Ca cos x + C4 sen x}.
72
CAP. 11 . EXTREMO OF. FUNCIONA LES
En virtud de las condiciones de frontera y (O) = O e y (n) = 1, tenemos
1
C
1
= O y C
3
= - - de modo que
1t
X
g=C2 sen x+C,xsen x- - cosx.
n
La funcin z se determina de la condicin z = g" + 2y. Tenemos
1
z= C
2
(2cos x+x sen x}+- (2 sen x-x cosx).
n
Las constantes C'J y C
4
se determinan de las condiciones de frontera
z (0) = O y z (n) = -1, de donde resulta que C4 = O y que C
2
es
arbitrarlo. Entonces.
1
z = C
2
sen x+- (2 sen x-x cos x).
n
La familia de extremales es
11=C2senx-_:_cosx,
n
z = C2 sen x+ f (2 sen cos x}, J
donde Cz es una constante arbitrarla.
3. Funcionales que dependen de funciones de varias variables
independientes. Consideremos la funcional
J [z (x, g)J = j F ( x, y , z, , } dx ay, (10)
D
donde F es una funcin diferenciable tres veces respecto a sus argumen-
tos, y supongamos que se pide hallar la funcin z = z (x, y) que sea
continua conjuntamente con sus derivadas hasta de segundo orden
inclusive en el recinto D, que lome valores fijos en la frontera f del
recinto D y que realice el extremo de la funcional ( 10) .
Si el extremo de la funcional (10) se alcanza en la superficie
z = z (x, y), la funcin z = z (x, y) satisface la ecuacin de Euler -
Ostrogradski
a
F:i.- ax {Fp}- ay {Fq}=O, (11)
o a
donde Tx {Fp} y
011
(Fq} son las derivadas parciales completas
respecto a x e y, respectivamente:
a az op oq
ax{Fp} = Fpx + Fp:r:ax+Fppa-+Fpq ox ,
a az op oq
ay { F q} = F qu + F qz ?Jg + F qp Ty + F qq ay
oz az
aqu se ha lomado, para abreviar, itl' = p y iJ JI = q,.
GENERALIZACIONES DEL PROBLEMA ELEMBNTAL
73
La ecuacin (11} representa la condicin necesaria de extremo de
la funcional (10). Es una ecuacin en derivadas parciales de segundo
orden; se busca su solucin z = z (x, y} que toma valores fijos en la
frontera r.
EJEMPLO 5. Escribi( la ecuacin de Euler - Ostrogradski para
la funcional
J !z(x, y)] = J J ( ( )2-( }2] dxdg.
D
SOLUCJN Tenemos F(x, y,z, p, q)-p2-qZ y, segn (11), encon-
a a
tramos -ax- (2p)-
011
(-2q) =O, o sea,
o2z 02z
Jx2 - iJy?. =O.
Pua la funcional
J[:z(xh xz, . .. , Xn)J = 5 J ... J F(xtt x2, ... , Xn, z, Pt
D
pz, .. ., Pn) dx1 dx2 .. dxn.,
oz
donde p
11
= -
0
(k= l, 2, ... , 11), la condicin necesaria de extremo
X11
viene dada por la siguiente ecuacin de Euler-Ostrogradski
n
Fz-

{Fpi}=O,
... 1
o, en forma desarrollada,
n n a
Fz- ( FXiP+FzPiPt + Fppj a:: ) = (12)
1-1 J-t
La funcin z = z (x
1
, x,, ... , xn.), solucin de esta ecuacin, debe
satisfacer en la frontera r del recinto n-dimensional D las condiciones
de frontera dadas.
EJEMPLO 6. Hallar las condiciones que debe cumplir la funcin
z (x
1
, x
2
, , xn) para que la integral de Drlchlet
n
... ,xn)l=)J ... J (::, )2dx1dx2 ... dxn
Q i-1
alcance en ella su mnimo si dicha fncin toma valores
en la r del reci nto Q.
74
CAP. 11. EXTREMO DE FUNCIONALES
n
SOLUCJON. En este caso F= (::t.)
2
, o sea, F no depende
{ ... j
expHcitamente de Xt. x
2
, , Xn, z. Por lo tanto,
Fi=Fip
F . = { 2 si l = j,
PtPJ 0 si i =/= j
y, apli cando la 6rmula (12), obtenemos
n
"\."' iJ?..z =0
4..1
; ... t ,
6
(ecuacin ndi mensional de Laplace).
OBSERVACION. Si bajo el signo de la integral figuran las derivadas
de Ja funcin z (x, y) hasta de orden 11, la ecuacin de Euler - Ostro
gradski tiene la forma
a a ifl az
Pi - ax {Fzx}- iJy {Fzu} + iJxz {Fzxx} + 2 iJxou {Frx11> +
az iJn
+ iJy2 {Fzw}- ... + (-J)n ayn {Fzv11 ... 11} = 0. (13)
--
n
EJEMPLO 1. Escribir la ecuacin de Euler - Ostrogradski para la
funcional
J (z (x, y)J = ) J [ ( ) 2 + ( r +
D
(
aiz )2 J
+ 2 iJx iJy - 2zf (x, y) dx dy.
SOLUCIN Tenemos
(
f)'l.z ) 2 { (}2z ) 2 ( iJ'l.z } a
F = iJxZ +
0112
+2 aia -2zf (x, y).
Aplicando la frmula (13). encontramos
(]2 ( i)'l.z ) iJZ ( iJZz )
-2t (x, .!/) + ox2 2 axz + iJy2 2 iJy2 +
()2 ( ()Zz )
+
2
axay
2
a.xay =
0

9 sea,
p. GENERALIZACIONES DEL PROBLEMA ELEMENTAL
75
La ltima ecuacin se representa brevemente asi:
M.'>Z = f (x, y).
Hallar las extremales de las funcionales s iguientes:
J
99. J [y (x}l = J (y
2
+ 2y.'2 + J/2) dx;
o
y (O)= O, y ( l) =O, y' (O} = 1
o
100. J [y (x)J = J (240y-y"'2) dx;
-1
y(-1)= l, y(O)=O,
y'(O)=O, y"(-1)=16,
b
101. J [y (x)] - ) (y+ y") dx;
a
e y' ( 1) = - sh 1 .
y' ( - 1) = - 4 ,5'
y" (O)= O.
y(a)-Yo. y(b) =YJt y' (a)= ~ y' (b) =Y;.
"
102. J [y (x)J = J (y'
2
+ yy") dx;
cJ
y(a) =A
1
, y' (a) =A
2
, y(b) =B., y' (b) = 8
2

1
f03. J {g (x)J = J (y'
2
+ f/
2
) dx;
o
y(O)=O, y(l)=shl, y'(O)=l, y'(l)=chl.
104. Hallar la extremal de la fundonal
1
J (y (x)l = -} } tf2 dx
o
con 1 as condiciones
y (O) = O, y' (O) = O, y' (1) = l.
11/l
105. J [y (x), z (x)] = J {2z-4!f + y'
2
-z'
2
) dx;
o
V (O)= O, y ( ~ ) = 1 z (O)= O, z ( ; ) = J.
76 CAP JI . EXTREMO DE FUNCIONALES
t
106. J[y(x), z(x)J= S (2xy-y'
2
+
3
dx;
-1
y(l)=O, y(-1}=2, z(l)=I, z(-1)=-l.
107. J[y(x), z(x)J= 1
2
(y'
2
+z'
2
-2yz) dx;
o
y (0) =O, y ( ~ ) = 1, z (O)= O, z ( ~ ) = l.
t
108. J[y(x), z(x)J = J (y'
2
+z'
2
+2y)dx
o
3
y(0)=1, y(l )=
2
, 2(0)=0, z(l)=l.
109. Probar que la ecuacin de Euler de Ja funcional
b
J[y(x), z(x)]= I F(x, y, z, y', z')dx
4
tiene. las s iguientes primeras integrales:
-JF
1) oy' = C si F no comprende y;
2) F
, oF J iJF e . F d
- y iJy' - z oz = s1 no compren e x.
Escribir la ecuacin de Euler-Ostrogradski para las
funcionales:
110. J (z(x, y)]= J [( :: )
4
+
D
+( :; )4+l2zf(x, y)jdxdy.
111. J [i(x, y)]= J J ( : ~ + : ~ )2dxdy.
D
t 12. J [z (xi. Xz, ... , Xn )l =
n
= ~ r J r a (Xi, Xz, ' Xn) ( ::J )2-
D j ... 1
.-c(x,, x
2
, , Xn)z
2
+2z/(xtt x
2
, ... , Xn)Jdx
1
dxa ... dxn
iNVARiANCI A oe LA ECUACIN DE EULE R
11
113. Deducir la ecuacin diferencial de las superficies
de rea mnima.
114. Hallar la extrema! de la funcional
1 1
J [z (x, y)]= J J ezv sen .z
11
dx dy
o o
con las condiciones z (x, O) = O y 2 (x, 1) = J.
6. lnvarianda de la ecuacin de Euler
Si l a funcional
I>
J [y (x)] = J F (x, y , y') dx
a
se transforma efect uando una sustitucin de Ja variable independiente
o una sustitucin simultnea de la funcin incgnita y de la variable
independiente, las extremales continan determinndose de la ecua-
cin de Euler que se obtiene a partir del Integrando transformado. En
esto consiste la invariancia de la ecuacin de Euler.
Sea x = x (u, v) e y = y (u, u) con la part icularidad de que
1
Xr, Xo 1 =f- Q.
Yu Yo
Entonces
J r
[
J
F (x, y, y')dx= F x(u, v), y(u, v), + , X
Xu Xvtl u
X (xu

du = J <Il (u, v, du
y las extremales de la funcional inicial se determinan de la ecuacin
de Euler para la funcional J <D (u, v, du:
d
<Dv-- <D ,=0.
du
0
EJEMPLO 1. Hallar l as extremales de la funcional
"'1
J [r (q>)J= J vri+r'2 dq.
"'
18
cM>. 11. DE PUNCONLES
SOLUCION. La ecuacin de Euler para esta funcional es
r d r'
V r2+r'2 yrz+r2
o.
La sustitucin de variables x=rcosq> e 11 = rsenlp da
y llt!Va a la funcional
b
J [JI (x)] = J '\l + y'2 dx
A
cuya ecuacin de Euler es 11" .... O de modo que
!1 = C1x +Cs.
Por consiguiente, las extremales de la funcional inicial vienen dadas
por la ecuacin
r sen qi = C
1
r cos c:p + C
1
,
donde C
1
y C
1
son constantes arbitrarlas.
EJEMPLO 2. Hallar las extremales de la funcional
lD 2
J (y (x)} = J (r"11'2- exyZ) dx.
o
sOLUCION. La ecuacin de Eulcrpara la funcional considerada tiene
la forma
y" - y' + et=y == o.
Realicemos la sustit ucin de variables
x= lnu, }
11=v.
La funcional inicial se transforma entonces en
2 2
t(v (u)] = J (e-In "u2v'2-elll vv2) = J (v'Z- v2) du
1 1
y su ecuacin de Euler v" + v = O se integra fcilmente:
V= C1 COS U+ C1 sen U,
Volviendo a las coordenadas iniciales x e y, obtenemos la ecuacin
de las extremales en la forma
g = C
1
cos eo: + Ca sen e".
s. NVtANCIA DE LA EUACIN DE eUtER
115. Ha llar las extrema les de la funcional
Ql(
J (r(<p)) = J rsen<p Vr
2
+r'
2
dcp.
!fo
116. Probar que las extremales de la funcional
'Pi
J [r (q>)] = i f (r sen q>) V r+ r'
2
d<p
To
se determinan por cuadraturas.
117. Hallar las extremales de Ja funcional
b
J (y(x)) = J Y x2+ y
2
lf l + y'
2
dx.
a
70
Igual que en el caso de una variable, la ecuacin de Euler -
Ostrogradskl es invariante respecto a las transformaciones de coorde
nadas.
EJEMPLO a. Escribir la ecuacin de Laplace
iJZz <)Zz .
axa-+ ay2 =O (t)
en coordenadas polares.
SOLUCION. Consideremos la funcional
D (z (x, y)]= 5 dx dy.
G
La ecuacin de Euler - Ostrogradski para esta funcional es precisa
mente la ecuacin (l). Pasemos en la funcional de las coordenadas
cartesianas (x, y) a las coordenadas polares (p, q>): .t = p cos cp, y =
= p sen q>. Tenemos
op op oq> sen q> aqi cos q>
ax = cos cp, iJy =sen qi, ax= --p- ' ay =-p-
y, por eso,
D (z (p, q>)J = J J [ ( Zp + Zqi )2 +
G
+(zp +zq> )
2
]pdpdq>= J J
G
Formando ta ecuacin de Euler - Ostrogradski para esta ltima
.Integral, obtendremos la ecuacin de Laplace en coordenadas polares;
1
p. Zcpq> + pzpp + Zp = 0.
cAP. i1. oe l'lvNctNAu1s
7. Campo de extremales
La familia de curvas y= y (x, e) forma un campo propio en el
recinto D del plano xOy si por cada punto (x, y) de este recinto pasa
una y slo una curva de la familia y = y (x, e).
El coeficiente angular p (x, y) de la tangente a Ja curva de la
fa milia y = y (x, e) que pasa por el punto (x, y) se denomina inclinacin
del campo en el punto (x, y).
La familia de curvas y= y (x, e) forma un campo central en el
recinto D del plano xOy si estas curvas cubren sin cruzarse todo el
y
X
Fig. 7J
reci nto D y arrancan de un mismo punto (x
0
, y
0
) que no pertenece al
recinto D. El punto (x
0
, y
0
) se llama centro del haz de curvas.
EJEMPLO 1. Dentro del circulo x
1
+ y
2
1 l a familia de curvas
y= Ce.,., donde Ces una constante arbitraria y, en particular, C = O,
forma un campo propio ya que estas curvas no se cortan en ningn
punto y por todo punto (x, y) del crculo pasa una y slo una curva
de esta familia (lig. 7). La lncllnacl6n del campo en un punto cual
qu iera (x, y) es igual a
p (x. y) = = y.
EJEMPLO 2. La familia de parbol as y = (x + C)'' no forma campo
propio del crculo x
2
+ l porque distintas curvas de la
familia se cortan dentro del circulo y no cubren todo el recinto (fig. 8),
EJEMPLO 3. La familia de curvas y = Cx forma un campo central
en el recinto x > O.
s 7. CAMPO DE EXTREMALES
81
y
X
Fig. 8
fforman campo (propio o central) en los recintos indi-
cados las siguientes familias de curvas?
tJ8. y=Ctgx;
11 9. y=Ccosx;
a) b)
Si

e)
x'l. y2
120. y=(x-C)3;

l.
121. y=C(x2-2x);
a) b)
122. y= e sen ( x - : ) ;
n n: n n
2
a)

b)

e)

n.
123. y = ex+c;
Si el campo (propio o central) est formado por una fa milia de
ex lremales de cierto problema variaciona l , se denomi na campo de
extrema les.
6- 01387
82
CAP. 11 . EXTREMO DE FUNCIONALES
EJEMPLO Consi deremos la funcional
i
J lY (x)] = J /2 dx.
o
Sus extremal es son l as rectas y = C
1
x + Ca. La familia de extremale
y = Ca forma un campo propio y la familia de extremales y = C
1
x
forma un campo central con centro en el origen de coordenadas.
124. Determinar para la funcional
o
J [y (x)] = J (y'
2
+y) dx
o
los campos de extremales propio y central.
125. Lo mismo para la funcional
n/4
J Jy(x)J = j (y'2-!f+x3+4) dx.
o
Supongamos que la curva y = y (x) es la extremal de l a funcional
XI
J fy (x)) = J F (x, y, y') ox
xo
que pasa por los puntos A (x
0
, y
0
) y 8 y
1
) .
Se dice que la extrema! y = y (x) est Incluida en un campo propio
si existe una familia de extremales y = y (x, C) que forma
un campo y que comprende la extrema! y = y (x) para cierto valor
C = C
0
y si, adems, esta ext rema! y= y (x) no pertenece a la frontera
del recinto D en el que la familia y = y (x, C) forma campo.
Si existe un haz de extremales, con centro en el punto (x
0
, y
0
) ,
que forma un campo en una vecindad de la extremal y = y (x) que
pasa por dicho punto, se dice que se ha encontrado un campo central
que incluye Ja extrema! considerada y= y (x). Como parmetro de la
familia y= 1J (x, C) se to ma el coefici ente angular de la tangente a
las curvas del haz en el punto (x
0
, Yo)
EJEMPLO 5. Consideremos el problema variacional elemental para
la funcional
2
J IY (x)] = J (y'3+ sen2 x) dx.
o
a) Sea y (0) ::..:: 1 e y (2) = l. La familia de extremales de nuestra
funcional viene dada por la ecuacin y = C
1
x + C
2
La extrema! que
satisface las condiciones de front era es y= l. Dicha ext rema! se
puede incluir el campo propio de extremaJP3 y = Cs. donde e,
es una constan te arbitraria.
i. tAMPO bE EXTREMALES
b) Sea y () = O e y (2) = 4. La extrema\ q i'sponde a estas
condiciones de frontera es la recta y = 2x que puede ser incluida en
el campo central de extremales u1 - C
1
x (C
1
es una constante arbitraria)
con centro en el punto O (O, O).
y
Fig. 9
1

)(
EJEMPLO 6. Consideremos el problema variaconal elemental
1
J ( (x)} = J y' ( 2x-+ y') dx;
-1
y ( - 1)=0,
La solucin de la ecuacin de Euler tit!ne la forma u=x2+c
1
x+
+c
2
La extrema! de este problema y=xi+ ~ - ! p u e e incluir
en el campo propio de extremalcs g=x
2
+: +c
2
(l ig. 9).
84
CAP. II. EXTREMO DE FUNCINALES
Probar que las de los siguientes problemas
variacionales elementales se pueden incluir en un campo de
extremales (propio o central).
1
126. J(y(x)] = J (y'
2
- 2xy)dx; y(O)=y(1)=0.
o
1
127. J[y(x)] = 1 (2exy+y'
2
)dx; y(O)= l , y(l)=e.
o
a
J 28. J [y (x)J = J (y2-y'
2
) dx (a :;6 kn);
o
y(O)=O, y (a)= O
2.
129. J f y(x)] = J (y'
2
+x
2
) dx; y (O)= 1, y(2)=3.
o
OEFI NI CION. Sea I> (x, y, C) =0 una familia de curvas planas. Se
llama C-dtscriminante de esta familia el lugar geomtrico de los puntos
determinado por el sistema de ecuaciones
<D (x, y, C) =0, }
oI> ex, u, C) _
0
ac -
( 1)
En el caso general, el C discrimlnante comprende la envolvent e de la
familia, el lugar geomt rico de los puntos rnl tiples y el lugar geom
trico de los puntos de retroceso.
la envolvente de la familia <I> (x, y, C) = O es la curva que en
cada uno de sus puntos es tangente a cierta curva de la famili a consi
derada y tal que cada una de sus partes es tangente a un conjunto nfi
nito de cur vas de l a familia.
Si se ti ene un haz de curvas con centro en el punto A (x
0
, y
0
), el
centro del haz pertenece al Cdiscrimnante.
EJEMPLO 1 Hallar el C-<l jscri minan t e de la familia de curvas
Y = (x - q2.
SOLUClN. Las ecuaciones (1) tienen en es te caso 1 a forma
y- (x-C)2=0, }
2 (x-C) =0,
de donde y = O Es fcil ver que l a y = O es la envol vente de
esta familia. Efectivamente, en cada uno de sus puntos x = x
0
la
lnea y = O tiene tangente comn con la curva correspondiente y =
p. CAMPO DE EXTREMALES
85
= (x - x
0
)
2
de la familia. Adems, si tomamos una parle de la lnea
y = O, por pequea que sea, habr un conjunto infnto de curvas de
la familia tangentes a esta parte. En el caso considerado el C-discrimi
nante consta de Ja envolvente nada ms.
En los problemas siguientes hallar los C-discriminantes
de las familias dadas.
130. y = Cx + O.
131. y (C - x) - C
2
= O.
132. (x - C)
2
+ yi = l.
Si el arco AB de la curva y= y (x) liene un punto comn A,
distinto del punto A, con el Cdiscriminante del haz y = y (x, C) que
tiene su centro en el punto A y que. comprende la curva considerada, se
dice que el punto A* es conjugado del punto A.
EJEMPLO s. Consideremos la familia monoparamtrica de curvas
y= C sen x. El Cdiscriminante de esta familia se determina por las
ecuaciones
y-Csenx= O, }
-!len x= O,
o sea, representa un conjunto discreto de puntos (ktt, O}, k = O, 1.
2, ... (que son los puntos de interseccin de la sinusoide y del eje
y
o
X.
r!g. 10
Ox). Tomando, ror ejemplo, e= 2, obtenemos la curva y= 2 sen X
que pertenece a haz de sinusoides con ceniro en el punto O (0, O}.
Si el otro ex tremo B (fig. l O) del arco de la curva y = 2 sen x tiene
Ja abscisa x E (n, 2n), el r ~ o OB tendr otro punto (a parle del punto
O (0, O)) perteneciente al C-discriminante, a saber el punto o (n, O),
que ser conjugado del punto O (O, O). Si es o <x < n, en el arco 08
no habr puntos conjugado: del punto O {O, O). 1
86
CAP. ll . EXTREMO DE FUNCIONALES
133. Se tiene la familia de curvas y = C (x - l) x.
Hallar el punto conjugado del punto O (O, O) .
134. Se tiene la familia de curvas y = C sh x. Hallar
el punto conjugado del punto O (O, O).
1 Condicin suficiente de Jacobl para poder Incluir la ext rema!
en un campo central de extrcmales. Condicin suficiente para que el arco
A B de una extrema/ pueda ser incluido en un camfo central de extrema/es
con centro en el punto A (x
0
, y
0
), es que el punto A conjugado del punto A
no perunezca al arco A B.
EJEMPLO 9. Consideremos la funcional
a
J[y (x)J = J (g'2- 9y2 + e"-"
1
- 1) dx;
o
y (O) = O, y (a) = O.
Analizar la posibilidad de incluir la extrema! y = O en un campo
central de extremales con centro en el punto O (O, O).
SOLUCIN. La ecuacin de Euler para la funcional considerada tiene
la forma !l + 9y =O y su solucin general esy = C
1
sen 3.x +Cacos 3x.
Si a =F k;, donde k es un nmero entero, la extrema! que satis-
face las condiciones de frontera es la recta !I = O. Consideremos la
familia monoparamtrica de extremales y = C
1
sen 3.r; es fcil ver que el
Cdlscrlminante de esta fa mil la consta de Jos puntos { k; . O), donde k
es un n(lmero entero; por eso, si a < , en la extrema! y = O no
habr punto conjugado del punto O (O, O) y entonces esta extrema! se
podr, obviamente, incluir en un campo central de extremales con
.n;
centro en el punto O (O, O). En cambio, sf a >
3
. en la extrema!
y = O habr como mlnimo un punto conjugado del punto O (O, O) y
no se cumplir la condicin suficiente de J acobi; en este caso las ex tri!*
males y = C
1
sen 3x no forman campo.
FORMA ANALfTI CA DE LA CONDICIN DB JACOBI. Consideremos et
problema varlaclonal elemental
:lCI
J(y(x)J= J F(x, y, y')d.x; y(.x1)==1/1
:i:e
Si la solucin u= u (.x) de la l!Cuacin de Jacobi
(F1111--1r- F1111) u- :x (F1111u')=O
(2)
que satisface la condicin u (xo) = O se anula tambin en algn otro
punto del Intervalo .r
0
< x < x
1
, el punto A conjugado del punto
A (x
0
, y
0
) pertenece al arco AB de la extrema! (el punto B tiene las
coordenadas (.rlr Y1
7. CAMPO DE EXTREMALBS
87
Si extste una solucwn u (x) de la ecuacin de J acob que satisface
la condicin u (x
0
) = O y que no se anula en ningn otro punto del seml-
interoalo x
0
< x ~ x
1
, en el arco AB no habr puntos con/ugados del
punto A. En este caso el arco AB de la extremal se puede incluir en un
campo central de extrema/es con centro en el punto A (x
0
, Yfl)
En la euacin (2) hay gue tomar en las funciones Fy
11
(x, y, y'),
F w' (x, y, y ) y F
71
,
11
, (x, y, y ) en lugar de y (x) el segundo miembro de
la ecuacin de la extrema\ IJ = y (x, C
0
).
EJEMPLO 10. Se cumple la condicin de Jacobi para la extrema!
de la funcional
a
J (y(x))= r (y'2+x2)dx
o
que pasa pot los puntos O (O, O) y B (a, 3}?
SOLUCl N. En este caso la ecuacin de Jacobi tiene la forma
u"= O. Su solucin general es u= C
1
x + e,. De la condicin u (0) =
= O encontramos e, = O de modo que u = C
1
x. Estas soluciones
u = C
1
x (C
1
::/=O) no se anulan para ningn valor de a > O. Por con
siguiente, en el arco OB de la extrema! no habr punto conjugado del
punto O (0, O). Es decir, este arco se puede incluir en un campo central
de extrema les con centro en el punto O (0, O). Es fcil ver que la extre-
ma! buscada es la recta y = ~ x que se puede incluir, obviamente, en
a
el campo central de extremales y = C
1
x.
EJEMPLO 11. Se cumple la condicin de Jacobl para la extremal
de la funcional
a
J[y(x)]= J (y'2-4y2+e-:x:i)dx (a-=F {n+}) n)
o
que pasa por los puntos A (0, O) y B (a, O}?
SOLUClN. La ecuacin de J acobi llene la forma u" + 4u = O y
su solucin general es u = C
1
sen 2x + C
1
cos 2x. De la condicin
u {O) = O encontramos Ca = O de modo que u= C
1
sen 2 ~ Si a < ~ ,
la funcin u no se anula para O< x..;;; a y la condicin de Jacobi se
cumple; en cambio, si a > ; , la solucin u= C
1
sen 2x de Ja ecua-
cin de J acobi se anula en el punto x = ; perteneciente al segmento
{O, al y en el arco de la extrema! y = O (O ~ x ~ a) hay un punto
conjugado del punto A (O, O). Por consiguiente, si a > ~ , no existe
campo central de extremales que comprenda la extrema) dada.
En los problemas siguientes analizar si se cumple o no la
condicin de J acobi.
88
CAP. 11 . EXTREMO DE FUNCIONALES
1
135. J [y (x)] = J (l 2xy + y'
2
+x2) dx;
-1
y(-1)= -2, y(l)=O.
a
136. J[y{x)I= J (y'
2
+9y
2
-3x)dx;
o
y(O)=O, y(a)= O.
l
137. J fy(x)] = J {l -f-y'
2
)dx; y(O)=Y(l)=O.
o
Q
138. J[y{x)]= J y'ev'dx; y(O) = 1, y(a) = b.
o
2n
139. J(y(x)] = ) (y'
2
-y'l.)dx; y() = O, y(2n)=l.
o
140. Demostrar que. si la funci n integrando de la fun-
cional
b
J fy(x}J = J F(x, y')dx
a
no contiene y explcitamente, cualquier extremal puede ser
siempre incluida en un campo de extremales.
La condicin de Jacobi es necesaria para que l a
runcional J ly (x)I alcance su valor extremo: o sea, cuando la extrema!
A 8 realiza e extremo, el punto conjugado de A no puede estar en el
intervalo xi)< x < x
1
Por ejemplo, la funcional
a
J[y(x)J = .\ (y'4 + 1)dx; y(O)=y(a)=O;
o
alcanza su valor mnimo en la exlremal y e O. En esta extrema! no
hay puntos conjugados del punto O (O, 0).
EJEMPLO 12. La funcional

;-n
J{y(x)}= J (V2-y'2)dx V(O)=O, u(: n) ==o
o
7. CAMPO DE E XTREMALES
89
no alcanza extremo en la e.x tremal y == O porque en el interval o
{O, ! Jt) est el punto o (n, O} conjugado del punto O (O, O) (ya que
Ja solucin de la ecuacin de Jacobi que se anul a en x = O es u=
=C
1
sen x y u se anula tambin en el punto x = n E (O, ! n)).
Efectivamente, tomemos como curva a y = O la curva
4
sen
5
nx
!In (x) = n<l ; para esta curva se cumplen C>LP1 iamt>nte las con-
diciones y (0) = y (
5
:) = O e (x} =

cos n x. Entonces
tenemos J [O) = O y
[
sen : 11xJ n
J n 2. = J
5
T "
o
dx-
- r (-
4
-)
2
cos2 t!X = Sn

< 0
J 5n 5 8n2 112 25
o
para todo entero n 2. Por consiguientc, la extremal y =O no realiza
el mnimo de la funcional considerada ya que existen curvas prxi mas
ay= O en las que son negativos los valores de la funci onal. Considere
mos ahora 1 a familia de cur vas !In (x) = +sen x cercanas a la curva
y s O en el sentdo de proximidad de orden cualquiera. Es facil ver que
5
T n Tn
r 5 r 16 2 4 9n
J IYn (x)j = J n2. dx - J 252ros 5 xd."t:=
40
n
2
>O.
o o
Por consiguiente, la extrema! 11 O tampoco realiza el mximo
de la funcional considerada.
141. Supongamos que en la funcional
b
J fy (x)] = J F (x, y, y'} dx
a
la funcin integrando F tiene derivadas parciales acotadas
de tercer orden a las variables !/ e y' en cualquier
90
CAP. 11. EXTREMO DE FUNCIONALES
recinto acotado de variacin de y e y'. Sean y = y (x) e y =
= y (x) + 11 (x) dos extremales cercanas. Demostrar que 1 a
funcin fJ (x) satisface la ecuacin de Jacobi
f 111/YI + F1111'lJ
1
- :X (F1111'fJ + F1111TJ
1
) = 0
salvo una infinitsima de orden superior respecto a la dis-
tancia de primer orden entre estas extremales.
2. Condiciones suficientes de legendre. Condicin suriciente para
que la extremal de la funcional
xi
J [y {x)l J F (x, y, y') dx;
!J (xo) =!lo.
xo
se pueda incluir en un campo de extremales es que se cumpla la condi
cin reforzada de Legendre. Esta consiste en que Ja desigualdad
F
11

11
, >O
se cumpla en todos los puntos de la extrema! considerada (o sea, para
todos los x E [xo, x
1
]).
EJEMPLO 13. Consideremos Ja funcional
2
J[y(x)]= J (y' '+y'")dx; y(O)=l, y(2)=5.
o
Sus extremales son las rectas y= C
1
x + C
2
La extremal buscada que
satisface las condiciones de frontera es la recta y = 2x + l.
En este caso F
11
,
11
, = 12y'
1
+ 2 y en todos Jos puntos de la extre-
ma! y= 2x + 1 tenemos F
11
,
11
, = 50 >O. Se cumple la condicin
reforzada de Legendre y, por consiguiente, la extremal y = 2x + 1
se puede inc.Julr en un campo de extremales.
Esto se ve tambin directamente. La extrema! y = 2x + 1 queda
comprendida en la familia monoparamtrica de extremales y ""'
;:::: 2x + a. (<X es el parmetro) que forma un campo propio.
EJEMPLO u. Consideremos Ja runcional
J (Y (x}) = ! (xlly'll + l 2yll ) dx;
-1
g(-1)=-1. y(l)=l.
La ecuacin de Euler para esta funcional tiene la forma
x.'ly'' + 2:ty' - 12y = o
y su solucin general es
!1 = C
1
x' + C
1
x
4

5 8. CONDICIONES SUFICIENTES DE EXTREMO
91
La extrema! que satisface las condiciones de rrontera consi deradas es
y= x8.
No puede ser incluida en un campo. La nica familia monoparamtrica
de extremales que la contiene es y = ax'. Pero esta familia no cubre
el recinto que contiene el punto de abscisa x = O (porque las extremales
de esta familia no pasan por los puntos del eje Oy con ordenadas distin-
tas de cero).
En este caso tenemos F'll''ll' = 2x
2
y Ja condicin de Legendre no
se cumple para x = O.
Analizar la posibilidad de incluir la extremal en un
campo para las funcionales siguientes
1
142. J (y(x)]= i (y'
2
-yy'
3
)dx; y(O)=O, y(l)=O.
o
a
143. J[y(x)J= J y'
3
dx; y(O)=O, y(a)=b>O.
o
:r1
144. J (y (x)I =) n (y) V 1 + y'
2
dx;
Y (xo) =Yo Y (x1) = Yt, n (y) > O.
a
145. J[y(x)J= (6y'
2
-y")dx;
o
y(O)=O, y(a) =b, b>O.
8. Condiciones suficientes de extremo de una funcional
Se considera el problema varlaclonal elemental, o sea, se considera
la funcional
:t:t
J [y (x)) = ) F (x, y, y') dx
(1)
:ro
con las condiciones de frontera
Y (xe1) = Yo 11 (x1) = Yi
{2)
to. Condiciones suficientes de Welerstrass. Se denomina funcin
dt Weiers/ra.ss E (x, y, p, y') la funcin defnlda mediante la igualdad
E (x, y, p, y')= F (x, y, y') - F (x, y, p) - (y' - p) Fp (x, y, p)

donde p = p (x, y) es la inclinacin. en el punto (x, y), del campo de
extremales del problema variaclonal (1) y (2).
92
CAP. 11. EXTREMO DE FUNCIONALES
CONDICIONES $UFICIENH.S DE EXTREMO DflBIL.
La curva C realiza el extremo dbil de la funcional ( l) si;
1) la curoa C es una extremal de la f u11clonal ( 1) que satisface las
condiciones de frontera (2), o sea, es la solucin de la ecuactt& de Euler
para la funcional ( l) que satisface las condiciones {2);
2) {a extrema/ C puede ser incluida en un campo de extrema/es
(esto tendr lugar. en particular, si se cumple la condicin de Jacobi);
3) la fur1cin de \reierstrass E (x, y, p, y') conserva su signo en
todos fos puntos {X, y) prximos a la extrema[ C y para valores de y'
prximos a p (x, y). La funcional J Jy (x)) tendr mximo en C si E O
y mnimo si E O.
CONDICIONES SUFICIENTES DE EXTREMO FUERTE.
La curva C realiza el extremo fuerte de la funcional (1) si:
1) la curva C es una extrema[ de la funcional (1) que satisface las
co11diciones de frontera (2);
2) la extrema[ C puede ser incluida en un campo de exfremales,
3) la funcin de Weierstrass E (x, y, p, y') conserva su sigtta en
todos los puntos (x, y) prximos a la extrema/ C y para valores cualesquiera
de y'. S E O, se tendr mximo y, si E O, se tendr mnimo.
OBSE'RVAClN. La condicin de Weierstrass es necesara para que
exista el extremo en el sentido siguente: si para ciertos valores de y'
la funcin E tiene signos opuestos en los puntos de la extremal, no se
alcanza el extremo fuerte si esto ocurre para cualesquiera valores de
y' por prximos que sean a p, tampoco se alcanza el extremo dbil.
EJEMPLO 1. Analizar el extremo de la funcional
1
Jy(x)J = J (y'+y')dx; y(0)=0, y(l)=2.
o
La ecuacin de Euler para la funcional considerada tiene
la forma y' y" = O de modo que las extremales son las rectas y =
= ;G.i_;f_ + C,. La extrema! que satisface las condiciones de frontera
es Ta recta y= 2x. La inclinacin del campo en los puntos de esta
es p = 2. Es evidente que la extrema\ y = 2x se puede
incluir en el campo central de. extremales y = Cx con centro en el
punto O (O, O). Es fcil probar as mismo que en este caso se cumple la
condi cin de J acobi. La ecuacin de J acobi es en este caso - :X (6y' u')=
= O, donde, debido a la ecuacin de la extrema!, hay que poner y' = 2.
Por consiguiente. la ecuacin de Jacobi toma la forma u"= O de modo
que u = C
1
x + C
2
De la condicin u (O} = O obtenemos C
2
= O.
Puesto que para C
1
=i!:: O esta solucin u = C
1
x no se anula en ningn
punto a excepcin del punto x = O, la condicin de J acobi se cumple.
Formamos la funcin de Weierstrass
(x, y. p, t) = y''+ y' - p
3
- p - (J/ - p) + 1) =
= (t - p} (V' + 2p).
CONbl lONl:.S SUFICIE NTES DE EXTREM 93
El primer factor es no nega tivo cualesquiera que sean los valores de
y' y el segundo es positivo para valores de y' prximos a 2. Por con-
siguiente, se cumpkn todas l as condiciones de mnimo ddl il. Adems,
es fcil ver que para y' < -4 la funci n E ser negativa y no se cum
plir l a condicin suficiente de e xtremo fuerte ya que para el extremo
fuerte se ex ige que la funcin de Weierstrass E conserve.> su signo para
valores cualesquiera de y' . Si se
tiene en cuenta la observacin de
la pg. 92, se puede llegar a la con v
clusin de que en este caso no hay
extremo fuerte.
EJEMPLO 2. Analizar el ext re-
mo de la funcional
l
J(g (x)]= J ( x+2y+ y'2) dx;
o
y(O)=O, y(l)=O.
SOLVClN. La ecuacin de
Euler paTa esta fu ncional tiene la
for ma y" = 2. Las ext remales son
las parbol as y = x
2
+ C
1
.x + C
2

La extrema\ que sa tisface l as
condici ones de frontera es y =
= x
2
- x. Formamos la ecuacin
de Jacobi -:x u' =- O, o sea,
u = O. Su sol uci n general es
u = C
1
x + C
2
La condicin
u (0) = O da C
2
= O y la
cin Jacobi se cumple ya que
u = C
1
x con C
1
-:fo O no se anula
en ningn punto del segmento (O, 1]
a excepcin del punto x = O; es de- Fig. 11
cr, la ex trema! y = x
2
- x se
puede incluir en un campo central
de extremales con centro en el punto O (0, O) , a sal>er, en el campo
y = x
2
+ Cx ( ig. 11 ). La funcin de We ierstrass tiene 1 a forma
E (x, y, p, y') = (y' - p)
1
. Se puede ver de aqu l que E=
= (y' - p)
2
O para cualesquiera valores de y' Por consiguiente,
la funcional considerada alcanza en la extre ma! y - x
2
- x mnimo
ruerte igual a J (x
11
- x] .
CAi>. 11. EXTREMO DE FUNCINLt:S
AnaJizar eJ extremo de las funcionales siguientes.
1
146. J[g(x)] = 5 (u2+;y'2)dx; y(O)=l, y(l)=e.
147.
148.
o
1
J (y (x)] = J eVy'
2
dx;
o
2
J [y\x)J = J ;,
3
i dx;
i
Cl
y(O)=O, y(l) = ln4.
y(l)= 1, y(2)=4.
149. J [y(x) )= J y(O)=O, y(a)=b, b>O.
o
1
150. J (y(x)]= J (l+x)y'
2
dx; y(O)=O, y(l)=l.
o
n/2
151. J[y(x)l= J (y2-!/
2
)dx; y(O)=l, y
o
2
152. J(y(x)J= J y'(l+x
2
g')dx; y( -1)== 1, y(2)=4.
-1
t
153. J [y (x)l = J (y'+ y') y(- 1) = -1, y(l) = 3.
-1
20. Condiciones sufici entes de Legendre. Supongamos que la
derivada parcial F 'J'IJ' (x, y, y') de la funcin F (x, y, y') es continua
y que la extrema! C est incluida en un campo de extremales.
SI en la extrema/ C se tiene F
11

11
, > O, en la curva C se alcanza
mlnimo dlbll; si en la extrema/ C se tiene F
11

11
<O, en ella se alcanza
el mximo de la funcional (!). Estas condiciones se llaman oondi
ciones reforzadas de Legendre.
En el caso en que F 'll' IJ' (x, y, y') ;;:;;:. O en todos los puntos (x, y)
prximos a la extrema/ C y para cualesquiera valores de y', se tiene mrzi
mo fuerte y en el caso en que F
11
.Y, (x, y, g' ) O para estos valores de
los argumentos, se fierie mximo uerte.
t 8. ONDJCIONES SUPICIENtM DE EXtREMO
EJEMPO 3: Analizar el extremo de la funcional
1
J(y (x}l= J (y'l-(J.y')dx; g(0)=0, y(l)=-2
o
9$
(a. es un nmero real cualquiera).
SOLUCiN. Puesto que la (uncin integrando depende slo de y',
las extremales son las rectas y = C
1
x + C
9
La extrema! que satis-
face las condiciones de frontera es la recta y= -2x que se puede incluir
en el campo central de extremales y= Cx. La inclinacin del campo
en esta extrema! es p = -2. A continuacin, calculamos F
11
,
11
, = 6y'.
En la extremal tenemos F'll' U' = -12 <O, o sea, la funcional alcanza
mximo dbil en la linea y= -2x. Para valores arbitrarios de y'
el signo de F
11

11
, no se conserva y, por consiguiente, no se cumplen las
condiciones suficientes de mximo fuerte.
En este caso la funcin de Welerstrass E (x, y, p, y') tiene Ja forma
E (x, y, p, y' } = (y' - p ~ (y' + 2p)
y para determinados valores de y' tiene signos opuestos. Teniendo en
cuenta Ja observacin de Ja pg. 92, deducimos que no hay mximo
fuerte.
EJEMPLO 4. Analizar el extremo de la funcional
2
J lu (x}] = ~ (e?'' +3) dx; y {O)= O, g (2) = l.
o
SOLUCJON. Las extremales son las rectas y=C
1
x+Ci. La extre-
ma! que satisface las condiciones de frontera es Ja recta y= ~ ; puede
ser incluida en el campo central de extremales y = Cx. Tenemos, ade
ms,. F
11
,
11
, (x, y, y') = e'I' > O para cualesquiera valores de y' . Por
consiguiente, Ja funcional tiene mnimo fuerte en la extrema! y = ~ .
EJEMPLO 5, Analizar el extremo de la funci onal
J(y (x)}= J vr dx; y(O)=O, y(a)=Yt
o !I
SOLUCIN. La funcin integrando no depende expllcitamente de
x y, por eso, tenernos F - y' Fu' = C
1
o, en nuestro caso,
y'2
CAP. 11. EXTREMO OB FUNCIONALES
de rlonde
1 -
.......,.=-...,...,,==- = C
1
y ( 1 r y'2> ,._. c.,
lfylfl+y' Z
(
1 )2 t I
donde Cs = ~ . Pongamos y' = ctg
2
. Tendremos y = Ciseni
2
=
= ~

(1-cost), Adems,
dx = ......!!J!.._= Cs sen t dt C1seni..!..dt
t t 2
ctg2 2ctg 2
e, integrando, encontramos
- C f ( 1- cost)dt C1 C
X - 1 j
2
=T(t + senl) +
2

Tenemos pues (e = ~ I )
x =e (f - sen t) + C2 }
y= e (1-cos t),
o sea, las ecuaciones paramtricas de una familia dt> cicloides. De la
2trR
J(
y
Fig. 12
condicin y (O) = O encontramos Ci = O. El haz de icloides
x= e (t-sen t), }
u= e < 1 - cos t)
forma un campo central con centro en el punto O (O, O) que comprende
la extrema!
x= R (t-sen t), }
y :=.R(l-cost),
donde R se determina de la condicin de que la cicloide pase por el
segundo punto frontera B (a, y
1
), s a < 2nR (fig. 12).
CONDICIONES SUFICIENTES DE EXTREMO
97
Empleamos la de Legendre. Tenemos
F11'11
1
= 3 >O
VY.o +u'
2
l
2
para cualesquiera valores de y'. Es decir, para a < 2nR la funcional
considerada tiene mnimo fuerte en la cicloide
x=R(t-sent), }
u=R (1-cos t).
Empleando la condicin de Legendre, analizar el extremo
de las funcionales siguientes:
1
154. J[y(x)]=S(y'
2
+x
2
)dx; y(0}=-1, y( l) =l.
o
3
155. J (y (x)J = J dx; y (2) = 4, y (3) = 9.
2 y
2
156. J{y(x))= 5 (xy'.,-2yy'
8
)dx; y( l) =O, y(2) = l.
1
a
157. Jly(x)) = J (J-e- lf')dx ; y(O) = O, y(a)=b.
o
1
158. J(y(x)]=) yy'
2
dx; y(O)=p>O, y(l)=q>O.
o
159. Analizar el extremo de la funcional
l
J [y(x)}=) (ey'
2
+y2+x
2
)dx; y(O)=O, y(l)= J;
o
para distintos valores del parmetro t: ..
EJEMPLO 6 (problema de .Euler). Una barra vertical de longitud l
se somete a una carga axial P. Para un valor determinado de P (fuerza
crtica de Euler) la barra se comba. Se pide determinar el valor mnimo
de la fuer za P que provoca la ftexl6n longitudinal.
SOLUCIN. Sea E el mdulo de elasticidad, sea I el momento de
inercia mnimo de las secciones transversales de la barra, se p el radi
de curvatura y sea cp .el ngulo entre la tangente y el eje.
1-01387
9R CAP. 11. EXTREMO DE FUNCIONALES
La ener:a potencial de la flexin se determina mediante la
f rmula
l
1 J dS
Ut=2 El -p2
o
Si el ex t remo de la barra desciende en
1
a= J (1-cos cp) dS,
o
la energa potencinl cte la barra disminuye en
l
llz= Pa = Pl-P) coscpdS.
o
Si antes de la delormaciu la energa potencial es igual a cero,
despuP.s de la delormacin estar dada por la frmula
l
U=U1-U,.= J
o
dS cp
9
Puesto que p = dq> y (para pequeos valores de q>) cos <> 1-T,
se tiene
1 l
U = t [El ( )
2
- Pipll] dS J [El ( )
2
- pq2] dx.
o o
En el caso de equilibrio, la energa. potencial toma su valor mini
mo. Por eso. el problema se reduce a la determinacin del mlnimo de
la integral
En este caso
F=EI (
2

y la ecuacin de Euler tiene la forma
q>" + aiq = O, donde el'= :r .
La !\Olucin general de esta ecuacin es
e> = C
1
sen cu + C
2
cos cu.
f&. CONDICTONES SUPICIE:NTES DE EX1REMO
99
Puesto que tg q> ~ q> para valores pequefios de q> y como, adems,
tg q> = y', se tiene
y' = C
1
sen ax+ C
1
cos ax,
de donde
11=
C, cosax+C2 senax C.
a a. +
Si el extremo inferior de la barra est en el origen de coordenadas,
ser y = o para X= o sea, C1 = e = O:
e
y= _z. sen<u.
cr.
Veamos si se cumplen las condiciones de Legendre y de J acobl.
Es obvio que l a condicin de Legendre se cumple:
(}Zf
oq>' = 2El >O.
La ecuacin de J acobl tiene la forma
E 1z + Pz = O z" + a'z = O
con la particularidad de que z (O) = O. Por eso, la solucin de la
ecuacin de J acobi ser
z = A sen ax.
La funcin z se anula para x,. =..!!!:_ (k = 1, 2, . . . ) de modo que la
a.
condicin de J acobi se cumplir si l ~ ..!!.. . De aqu
a
ni
P4:---El.
El valor mlnimo de la fuerui critica de Euler ser
ns
Pmtn=""j2EI
y l a ecuacin de la curva de ilexin ser
Cz. nx
y=-;:;:-sen -
1
-.
3. figuratrl2. Sea dada la funcional
b
J IY (x)J= 1 F (x, y, y') dx.
a
Tomando x e y como parmetros, consideremos la funcin y =
= F (x, y, y') en tanto que funcin del argumento y'. El ~ r f i o de
esta funcin en el pl ano de las vi1riables (y', Y) se denomina flguratriz.
7
100
CAP. 1 i. EXTREMO DE FUNClNA LES
Es fcil probar que l a funcin de Weierstrass E (x, y, p, y') representa
la dilerenca entre las ordenadas de la figuratriz y las ordenadas de la
tangente a la liguratriz trazada por el punto de abscisa y' = p. Si la
funcin de Weerstrass conserva su signo para ciertos valores de y',
ello significa que la figuratriz est por encima o por debajo de la
tangente para esos valores de y' . En este caso hay mfnimo dbil. Si
la figuratriz est a un lado de la tangente para todos los valores de
y' y para los valores de los parmetros x e y prximos a los puntos de
la extrema!. hay extremo fuerte.
La condicin suficiente de Legendre se expresa en estos trminos
as: si para todos los puntos (x, y) prximos a la extremal la ftguratriz
es cncava llacia las Y positivas o negativas, hay extremo fuerte.
EJEMPLO 1. Analizar el extremo de Ja funcional
(l
J(y(x))= y'"'dx; y(0)=0, y(a)=b, b>O.
o
SOLt:CiN- Las extremales son las rectas y= C
1
x + C
2
La extre-
ma! buscada viene dada por la ecuacin y = x. Puede ser i!'duida
a
y
Fig. 13
en un campo central de cxtremales. La figuratriz es la parbola
Y = y'm (f ll- 13). Es fcil ver que toda la figuratrz est por encima
s 8. CONDICIONES SUFICI ENTES DE E XTRE MO
101
de Ja tangente trazada a la misma en el punto p = .!!. cualesquiera
a
que sean a y b (a * O) . Por consiguiente, la unclonal considerada tiene
mnimo fuerte en la extremal y = x.
EJEMPLO s. Analizar el extremo de l a funcional
o
JJy (x)l= I y'Sdx; y (O)=O, y(a}=b, b>O.
o
h
SOLUCIN. La extrema! buscada es la recta y = - x que se puede
a
incluir en el campo central de extremales y = Cx con centro en el
y
Fig. 14
punto O (O, O). La fuguratriz. es la parbol a cbica Y = y'
3
(fig. 14).
Para valores de y' sufi cientemente prximos al valor p = .!. la igu-
a
ratriz est sobre la tangente a la misma en el punto de abscisa y' =
a
De la lig. 14 se puede ver que la figuratrlz corta Ja tangente en el punto
de abscisa y' = -
2
b y a la izquierda e este punto aparece por debajo
a
102
C:AP. JI . f.XTRf.MO J>E PUNCIONALL: s
de la tangente. Por" lo tanto. hay mnimo dbil en la t>xlremal
b
= -x.
a
Ntese que para p = O (esto corresponde al caso b "" O en el que
Ja extremal es un segmento del eje Ox) la tangente a la figuratriz es
el eje Oy' y el punto O (O, O) es un punto de inflexin de la figuratriz.
Teniendo en cuenta la observacin de la pg. 92, vemos que en cual-
quier vecindad del punto O (O, O), por pequea que sea, la figuratrz
tiene ordenadas tanto positivas como negativas. Por lo tanto, la
funcin de Weierstrass E tiene signos opuestos para valores de y'
tan prximos a p = O como se quiera y, por consiguiente, en esto caso
no se alcanza ni siquiera el extremo dbil.
EJEMPLO 9. Probar qui! la extrema! y = O del problema variaciona l
1
J\y(xiJ=) (y'i - yy'3)dx; y (O)=y ( l) = O;
o
realiza el mnimo dbil de la funcional.
SOLUCION. En este caso la condicin de Legcntlre da
F
11

11
, 1
11
-o=(2-6yy') 1
11
...
0
-=2>0.
o sea, se alcanza mnimo Mbil en la extrema! y = O. Demostremos
1.ue en esta extrema) no se alcanza el mnimo fuerte. Consideremos la
y
y'
Fig. 15
figuratriz Y = y'
2
- yy'' para los valores y > O (fig. 15). De la fig. 15
se ve que la tangente a la figuratri z en el punto de absci sa p = O corta
la figuratriz en el punto y' = ~ Es decir, para los puntos (x, y),
y
con y > O, prximos a los puntos de l a extrema! y= O, la funcin de
Weierstrass E es positiva para valores de y' menores que ~ y es negati-
!/
8. CONDICIONES !'UPICJENTES DE fXTIHoMO
!03
va para y' > 2.. Segn la observacin de la pg. 92, no hay mlnlmo
y
fuerte. Una situcin semejante se tiene tambin para y < O.
Lo que destaca este ejemplo es que en l la condicin F
11
,
11
, > O
se cumple en la extrema! para cualesquiera y' y, sin embargo, ello
no implica la existencia de extremo fuerte.
Empleando la figuratriz analizar el extremo de las funcio-
nales siguientes:
1
160. J[y(x )] = J (l+x)y'Zdx; y(O) = O, y(l)=-2.
o
2
161. Jly(x)] = J y'(l+x
2
y')dx; y(- l) = y(2)= 1.
_,
a
162. J[y(x))= J (l-e- 11'
4
)dx;
o
y(O)=O, y (a )=b, (b>O).
a
163. J (y (x)J = J (6y'
2
-Jl' +yf/)dx;
o
y(O)=O, y(a) = b (b>O).
ORSERVAClN. La no negatividad de la segunda variaci n es con
dicin necesaria, pero no sufici ente, para que la funcional J [y (x))
al cance mnimo en la \: urva.
t:JEMPLO 1 o. Consideremos la funcional
1
J[g (x)l = J yi (x-y) dx
o
en el espacio C [O, J} . La ecuacin de Euler tiene la forma F
11
= O
/1 = O. Para O ~ x ~ 1 la segunda variacin de l a funci onal en la
extrema\ y = o
l
{J2J (O, 6y1 = I x (y)s dx
o
es positiva para todo ~ I =6 O. Sin embargo, en cual quier vecindad del
cero la funcional J [y (x)l toma tambin valores negativos; basta fijar
l04
<.:AP. ll. EXTREMO DE FUNCIONALES
e > O y considerar la funcin
{
-x+e,
!Jg (x) = O,
e'
Entonces tendremos J f!le (x)] = -
6
< O para cualquier e> O.
DEFINICJN. Se dice que la funcional cuadrtica L
9
(h] definida en
un espacio normado es fuertemente posi/lva si existe una constante
k >O tal que
para todo h.
CONDICIN SUFICIENTE DE MtNIMO. Condicin suficiente para que
la funcional J ly (x)) definida en un espacio normado tenga mnimo
en el punto estacionario y = !lo es que su segunda variacin sea fuerte
~ n t positiva en y = y
0
, o sea, que se cumpla la condicin
fJ'J !yo. fJy) ~ k 11 fJy ue.
donde k = const, k > O.
4. Supongamos que se busca el extremo de la funcional
%i
J [!ltt IJz, .. , Yn]= J F (x, !11t !IZ .. ., Yn y;_, y;, ... , y;.) dx, (3)
%Q
que depende de n funciones y
1
(x), Yt (x), .. ., !In (x), con las condi
ciones de frontera
Yk (xo) = IJkO !11< (x1) = Yin (k = l , 2, ... , n).
La condicin refonada de Legendre consiste en que las desigualda-
des
F 1111
1 n
F1111
1 n >0
F11'11' F1111 F,,.,,.
n 1 n ll n 11
{4)
se cumplan en todos los puntos de la extrema! considerada de la fun
cional (3).
La condlc1on reforzada de Jacobi consiste en que el segmento
f.i
0
.i
1
J no contenga punto conjugado del punto x
0
.
f 8.
CONDICIONES SUFICIENTES DE EXTREMO
!05
La condicin reforzada de Legendre (4) conjuntamente con la
condicin reforzada de Jacobl garantizan por lo menos la existenci a
del mlnlmo dbil de la funcional (3).
EJEMPLO 11. Analizar el extremo de la funcional
1
J [.!/ (x), z (x)] = J (y'+2' 2) dx;
o
y (0) =O, z (0) = O, }
y(l)= l , z ( l )=2.
(5)
(6)
SOLUCION. Las ecuaciones de Euler para la funcional (5) son
de modo que
y* = O, z = O,
y=C1 +C2x. }
z=Cd C,x.
Empl eando 1 as condiciones {6}, obtenemos
C
1
=O, C
1
= I, C
3
= O y Ct. = 2.
La extrema) buscada
y=x, }
Z=2X
representa una rect a que pasa por el origen de coordenadas.
Tenemos
Fyy = 2, Fy'z' = O, Fz'y' = O y F Z'%' = 2.
La condicin reforzada de Legendre se cumple:
F11"11'=2>0. ' Fy'' F ~ = 2 1 =4 >0.
F:r.'y' F%'%' 1 O 2
(7)
(8)
Veamos si se cumple la condicin reforzada de Jacobi.
Una de las definiciones de punto conjugado es la siguiente (vase
[3)).
Supongamos que se tiene una familia de ex tremales de la funcio-
nal (3) que arrancan del punto inicial (x
0
, y
10
, .. , .!/no) en direc-
ciones prximas r ero linealmente independientes. Se dice que el
punto x E [x
0
, x
1
es conjugado del punto x
0
si existe una sucesin
de extremales, que arrancan todas del punto inicial y que son t an
prximas como se quiera a la ex tremal considerada, tal que cada una
Cle estas extremates corta la extrema! considerada con la particulari-
dad de que las abscisas de los puntos de Interseccin convergen haci a
el punto x.
En nuestro caso las extremales son las rectas (7). Todas las extre-
mat es que arrancan del punto (O, O, O) cortan la extrema! (8) en t>ste
106
C.'\I' , 11. EXTnEMO DE FUNCIO . ./.l\LES
punto Por lo tanto, el segmento (O 1) de varacin de x
no contiene punto conj ugado del punto x
0
= C. Es decir, se cumple
tanto la condicin reforzada de Legendre como la condicin reforzada
de Jacob de modo que la extremal (8} realiza el minimo dbil de la
funcional (5).
Analizar el extremo de las funcionah.!S siguientes:
t
164. J[y(x).z(x)I= j v1 +y
12
+z'
2
dx;
o
y(O) .,., o, y(l) = 2, z(O)=, z{1)=4.
1
165. J IY (x), z (x)l = ) (y'
2
+z'
2
+ 4z) dx;
o
y(O)=O, y(I)= l, z(O) = O, z(l) = O.
9. Extremo condicionado
l. Problema isoperimtrico. Sean F (x, y, y') y G (x, y, y') dos
funciones.
El problema isopermtrico consiste en Jo siguiente.: entre todas
las curvas y= y (x) E C
1
[x
0
, x
1
] a lo largo de las cuales la funcional
:lei
K[y(;c;)J= J G(.1;
1
y, y
1
)dx
llene un valor fij o ! hallar la curva en la que la funcional
:11:1
J f y (x)i = F (x, y, y') dx
xo
alcanza su valor extremo.
Suponemos que las funciones F y G tienen derivadas parciales
continuas dr primer y de segundo rdenes para x
0
x x
1
y para
valores cualesquiera de las variables y e y' .
TEOREMA DE EULER. Si la curva y = y (x) realiza el extremo de la
funcional
XI
J IY (x) I-= } F \X, y, y'> dx
Xo
p.
con tas condiciones
Xt
EXTREMO CONDICIONADO
K[y(x)l= J G(x, y, y')dx=l, y(xo)=Yo y(x1}=Yt
:ro
107
y si g = y (x) no es extremal de ta funcional K existe una constante f.
tal que la curva y = y (x) es e;ctremal de Ja funcional
XI
L (y (x))= J [F (x, y, y') +W (x, y, y')] dx.
:ro
EJEMPLO 1 (problema de Dido). Entre todas las curvas cerradas
de longitud 21 hallar la curva que comprende el rea mxima.
SOLUCIN. Observemo.s, ante todo, que dicha curva debe ser con
vexa. Efectivamente, de lo contrario encontraramos una recta l
A
e
Fig. 16
(fig. 16) tal que, al reflejar en ella la parte BCD de la rontera, obten-
dramos un re.cinto de rea mayor que el inicial siendo la longitud
de la rontera la misma que antes.
ObseJvemos tambin que toda recta que divide por la mitad la
cerrada que comprende el rea mxima ha de dividir por la mitad la
propia rea. Efectivamente, supongamos lo contrario y sea L
1
una recta
que no cumple esta propiedad. Reflejando en l
1
Ja parte de la figura
de rea mayor, obtendremos una curva de idntica longitud pero que
comprende un rea mayor.
Tomemos como eje Ox cualquiera de las rectas que dividen por
la mitad la curva; llegamos entonces al problema siguiente.
Hallar la lnea y = y (x), IJ (-a) = y (a) = O, de longitud fija
l > 2a que conjuntamente con el segmento a ~ x ~ a del eje O:c
encierre el rea mxima. Por lo tanto, el problema $e reduce a hallar
el extremo de la funcional
(1
J[y(x))= J ydx; y(-a)=y(a) = O;
-a
108 CAP. 11 . EXTREMO DE Pl:INCIONALES
con la condici n complementarla de que
a
K (y (x)) = J Vl+y''dx=l (l>2a).
-a
Formemos la funcin auxiliar
H=F+W=y(x}+'.).. lfl +y' 2(x}
y consi deremos l a [uncional auxiliar
a
l IY (x)) = J H (x, y, y') dx.
-a
La ecuacin de Euler para la funci onal (2) tiene la for ma
- - 1
d ( ')..y' )
dx V1 +u' - ,
de donde
'J...y'
Vi + u'
Resolviendo la lt i ma ecuaci n respecto a y', encontramos
dy x+C1
di = V C
1
)i .
Integrando la ecuacin (3) , obtenemos
(x+C
1
)i+ (y+c
2
)2=J..2,
(1)
(2)
(3)
o sea, la circunl e.rencia de con centro en el punto (-C
1
, -C
1
) .
Las constantes C
1
y as como el parmetro A. se determinan de las
condiciones de frontera y (-a) = y (a) = O y de la condicin isoperi
mtrica (1). Tenemos
de donde
de modo que

}
C! =V-(C1+a)2,
e
X
y' = - --====-
V !.2-xZ
CNOICIONADO
109
La condicin (1) da entonces
o
1
').. dx X x-a a
l= _ =Aarcsen-;- = 2J..arcsen-:-
v )..2. - xz ,.. x--ci ,..
-o
o sea,
a l
-,;=sen
2
1..
Resolviendo respecto a A. esta ecuacin trascendente, encontramos un
valor determinado A.= A.
0
y despus encontramos el valor de C
2
=
= y).g - '
E.s fcil persuadirse de que Ja ecuacin = sen ;').. tiene siem-
pre solucin. Efectivamente, tomando = t, esta ecuacin quedar
reducida a sen t = t, donde =<I por hiptesis del problema.
La inclinacin de la tangente a la funcin y = sen ten el punto t = O
es Z mientras que la inclinacin de la funcin y =a.I es menor. Por
consiguiente, los grficos de estas funciones tienen un punto de inter.
seccin como mlnimo, a parte del punto O (O, O).
PRlNCJPlO DE RECIPROCIDAD EN EL PROBLEMA
Las extrema les de la funcional
"'
J IY (x)I = J F (x, y, y') dx
con la condicin complenuntarla
:Xt
K l.Y (x)]= ) a (x, y, y') dx= consl

colnclden con las extremales de la funcional K. [y (x)I con la condicin
J (y (.x)] = const.
Basndonos en el principio de reciprocidad, deducimos del proble
ma de Dido el resultado siguiente: entre todas las curoas cerradas que
comprenden un drea fiia, la circunferencia es la curva de longitud
mlnima.
Es cil obtener este resultado directamente si se recurre a, la
forma paramtric.a de\ problema variacional.
Sean
x=x{t), x(to)=x(t1).}

t
1
,
!I =Y (t), Y (to) =y {t),
110
CAP. 11. EXTREMO oe FUNCIONALES
las ecuaciones de una curva cerrada. El problema consiste en hallar
el extremo de la funcional
1
f dt
con la condicin
J dx = C.
Introduciendo la funcin
1
>..
encont ramos (vase Ja pg. 68} que la curvatura de Ja curva que
r
realiza el extremo es constante:
_!.=l..
r
Por consiguiente, la extrema) buscada es una circunferencia.
Utilizando el principio de reciprocidad, se pueden resol ver, sin
realizar clculos, problemas cvarlacionales de la Geometria
elemental.
EJEMPLO 2. Demostrar que: 1) entre todos los tringulos de base
y permetro fijos, el de rea mxima es el tringulo issceles; 2) siendo
fijas el rea y la base, el tringulo issceles es el 'tringulo de per-
metro mlnimo.
SOLUCIN. 1) Tomemos una elipse cuyos focos son los extremos
de la base de los t ringulos considerados (fig. 17). De la propiedad
y
c
0
(o,b)
Fig. 17
de la elipse deducimos que lodos los tringulos ACB tienen el mismo
permetro. Es evidente que el rea mxima corresponder al tringulo
EXTREMO CONDICION1\DO
111
de altura max1ma lo que significa que el ver tke del t ringulo debe
coincidi r con el vrt ice C
0
de la elipse. El tringul o AC
0
B es Issce-
les.
2) Segn el princpio de reciprocidad, siendo Jijas el rea y la
base, el per met ro mnimo cor responde al tringulo issceles.
EJEMPLO J. Hallar el mni mo de la integral
"
J [y(x)) = J y'2dx
o
lt
con las condic iones i y2dx= 1, y (O)=y ln).=0.
SOL.UClN. For memos la funcional auxili ar
"'
l [y (x)J = J (y'2+ J..y2) ru:
o
f consideremos su ecuacin de Euler
ZA.y - :x (2y') =0, o sea. (4)
Su ecuacin ca racterst ica es r
2
- A. = O, de donde r
1
,
2
= lfl.
Est claro que J. debe ser menor que cero: s i 11ct.>ptamos que J. > O,
l a solucin general de l a ecuacin (4) tendr la for ma y = C
1
e +
+ c,e- V;:x, las condiciones de frontera y (O) = y (n) = o se cumpli-
rn slo para C
1
= O, C
2
= O, o sea, resultar y = O y no se cumpli r
11
en este caso la condicin J y
2
dx = 1; de la misma for ma, s i J..= O,
o
la soluci n de la ecuacin de Euler (4) que sa t isface las condiciones
de frontera t ambin ser la funci n y = O. Por eso, consideramos que
A. <o de modo que ' 1.a = i v =x1 y l a solucin general de la
ecuacin (4) es y = C
1
sen V - 1.. x + C
2
cos 1f 'A.x. La condicin
y (0) = O da C
1
= O y la condicin y (n) = O da - /.. = k
1
(k = \ , 2, . .. ). Es decir, y = C
1
sen kx, donde C
1
no se ha deler mi-
n
nado a n. Utilizando la condi cin J y
2
dx = I, obtenemos
o
1'I
J Cf scn2kx dx= 1,
o
112
CAP. 11 . EXTREMO DE flUNClONALES
de donde C
1
= V! . O sea, 11== V! senkx. Todas las extre-
males 11= V! senkx pasan por los puntos (O, O) y (Jt , O), pero
slo para dos, a saber, y= V! sen x, se cumple la condicin
de Jecobl. En estas dos extremales se tiene
r 1- 2
J ly (x)I = J y'Zdx= J "'i"cos2xdx=I.
o o
EJEMPLO 4 (problema de Kelvin . Supongamos que en el plano
xOy est distr ibui da una masa de densidad continua {x, y) y supon
gamos que se tiene en el plano una curva C suave a trozos y dos puntos
P
1
y P, sobre la misma. Entre todas 1 as curvas de longitud ja l
que unen Jos puntos P
1
y Pz hallar la curva que conjuntamente con
el arco P
1
P
2
de la curva C forme un recinto D de ma!>a mxima. Los
puntos P
1
y P.1 pueden coi ncidir.
SOLUCl N. Ja funcin
V (x, y)= J {x, y) dx.
Segn la frmula de Green, tenernos
J J tx. y)dxdy= J J dxd!J """' f Vdy,
D D r
donde el contorno f i.e compone de la curva L y de la parte P
1
P
2
de la curva C. A lo largo de esta ltimu parle la integral toma un
valor determinado que designaremos por K. Aceptando que la curva
L est dada en forma paramtrica
tendremos entonces
x=x (t), }
lo< t
Y= y(t),
t
J J (x, y)dxdy= J V(x, y)ydi+K.
D
Por .;onsiguiente, el problema ha quedado reduci do a la determi
nac in del mximo de la funcion al
tt
Je,= ) V (x, y) y dt
ti
s 9. EXTREMO CONDI CIONADO
con la condicin de que
tt
Yxz+Ji2dt=1.
,
Consideremos la funcin auxil iar
F=Vy+X
113
y empleemos la forma de Weierstrass de la ecuacin de Euler. Tenemos
av
F = iJx ' F = O,
XII yx
F.. .
F, =---==-- =---
3
y2 2
(x2 + y2)
de modo que la ecuacin de Euler en l a forma de Weierstrass <:i.
1 1 av
7=-;a;-
0, rerordando la expresin de la funcin V (x, gi.
1 (x, y)
7= A.
donde r es el radi o de curvatura de la curva pedida.
En el caso en que 1-l (x, y) = const resulta que l a curvat ura de
la curva pedida es constante y, por consiguiente, las extremales son
circunferencias. Queda claro que realizan el mximo de la funcional
JL.
Tambin se denominan problemas isoferimtricos los problemas
variacionales en los que se pide hall ar e extremo de la funcional
Xt
I (Y11 !/21 . , YnJ= J F (x, Yt. yz, ., Yn y\, Y . . . , Y) dx (5)
:ico
con las asl llamadas condiciones isoperimtr icas
Xl
J G (x, !11 !Jz, . , Yn Yt .. ., t6)
xo
(i= 1, 2, . .. , m),
donde 1
1
son unas constant es.
Para obtener la condicin necesaria fundamental en el problema
soperimlrico sobre la determinacin del extremo de la funcional (5}
8- 011187
114
CAP 11 . EXTREMO DE FUNCIONALES
COr1 las Condiciones (6) hay que formar Ja UOcionaJ auxiliar
Xt m
cI> IYt !h. - .. , Ynl = 1 ( F + ,G) dx,
xo i>=t
donde At son unas constantes y escribir sus ecuaciones de Euler. Las
constantes arbitrarlas C
1
C
2
, . , Cin de la solucin general del
sistema dt' ecuaciones de Euler as como las constantes A.
1
, A.
2
,
. . . , Am se determi nan de las condiciones de frontera
!J1t, (xp) = Y1t,u . !lk (x, ) = Ykt (k = 1. 2, ... , n)
y de las condiciones isoperi mtricas (6)
XI
(i=l,2, ... ,m).
xo
EJEMPLO s. Hallar la extrema! en el problema isoperimlrico
sobre el extremo de la runcional
1
J IY (x) , z ( x) ) = I (y'2 + zi _ 4xz' - 4z) cfx;
ll
y(0) =0, z(O)=O, y(l )= l, z(J) = I;
con la condicin
1
J (y'2- x/-z'2) dx = 2.
u
SOLUCIN. Formame>s la funciona 1 auxiliar
1
<D fy (x), z (x)j = J [y'2 + z 2-4xz'-4z + J.. (y'2-xy' -z'2)) dx
(1
y escribimos para ell11 el sistema de ecuaciones de Eu ler
- (2y' + 2Ay' -A.x) = O, }
_ 4 _ !!:_ (2z' - 4x-2A.z') = O
dx '
r esolviendolo, encontrnrnos
s 9. EXTREMO CONDICIONADO
115
Las condiciones de frontera dan
Ct -
_311.-j-4.
2 C2=0,
Ca=2 (1 -A.} y
de modo que
.A.x2 + l3"'+4)x }
y= 4 (1 +"-) ,
Z=X.
Para determinar l. recurrimos a la condicin isoperimt ri ca (7). Puesto
, 2"-x+3A.+4 ,
que y =
4
(l +"-) y z = l, obtenemos
1
r [(2A.x+-3). + 4)2 (21..x-f-3A. -\- 4) x 1] d:c= 2,
J 16(l + A. )2 4 (1+1..)
i
de donde. despus de unos clculos sencillos pero voluminosos, obte
nemos para A. la ecuacin siguiente:
~ (23A.2-H6A.+24)=48(A.2+21.+ 1).
De aqu resulta 1..
1
= - : ~ y A.2 = - ~ ~ . Introduci endo en (7), vemos
que A.
1
= - : ~ sati sface y 1
2
= - ~ no satisface la condici n iso-
perimlrica.
La extrema! buscada se determina por las ecuaciont-s
y= 7x-;_5x2 , }
Z=x.
Hall ar las extremales en l ~ siguientes problemas isope-
rimtricos.
166. Entre todas las curvas planas <le longitud l con
extremos en los puntos fijos M
0
(x
0
, Yo) y M
1
(xJt y
1
) hallar
la curva con ordenada mnima del centro de gravedad (pro
blema sobre la forma de equilibrio que toma un cable pesado
homogneo por accin de la gravedad).
'
167. J{y(x)) = .l y'
2
dx; y(O) = 1, y(l) = 6; con la
o
1
condicin J' y dx = 3.
o
116
CAP. ll. EXTREMO DE FUNCIONALES
l
l68. J [y(x)J= J (x
2
+ y'
2
)dx; y(O) = O, y(l)=O; con
o
1
Ja condicin i !f dx = 2.
o
1
169. J[y(x)) =) y'
2
dx; y(O)=O, y (l) = ! ; con la
o
t
condicin ~ (y- y'
2
) dx =
1
1
2

o
20. Tambin es un problema variacional de extremo condicionado
el problema de Lagrange en el que se pide ha llar el extremo de la
funcional J [y
1
, y
2
, ., Yn1 con la part icularidad de que se imponen
ciert as condcones de enl ace a las funciones de l as cuales depende
la fu ncional J.
El problema se plantea as. Hall ar el extremo de la uncional
Xt
Jfy. YZ Ynl= J F(x, Y1 Y2 . ., Yn Yt y2, .. , y ~ d x ; (8)
:xo
YJ(Xo) =Y;o. Yj (Xt ) =y (j=I. 2, ... , rt);
con las condici ones
CJl1 (x, Y1. Y1. .. Yn) = O
(i = 1, 2, .. ., m; m < n)
(9)
que se consideran independientes.
TEOREMA. Las furiciones y
1
, y
2
, .... Yn que realizan el extremo
de la funcional (8) con las condiciones (9) satis/ acen, siempre que los
factores '-t (x) (i = 1, 2, . . . , m) se escojan debidamente, las ecuaciones
de E uler de la funcional
Xi Pl
J = J [ f + l.cp ] dx.
xo ..,. 1
m
Para abreviar pondremos F + 2: A.tq =et> (x, Ytt Y2 . Yn Y
i=1
Y .. . , y ~ . Entonces las. funciones A. i (x) e y (x) se determinan de
s 9.
EXTREMO CONDICIONADO
ecuaciones de Euler
<D' (J = l, 2, .. ., n)
llJ dx Vj
y de las ecuaciones
q> (x, y
1
, Yi .. ., y,.)= O (i = I, 2 . . . , m).
117
Las ecuaciones q>
1
= O tambin se pueden considerar como ecuac iones
de Euler para l a funcional J"' si se acepta que los argumentos de esta
funcional son tanto las funciones y
1
, !h. . ., !lri como las funciones
A (x), /..1 (x), .. . '-m (x).
EJl?MPLO 6. Hall ar la distancia mnima entre los puntos
A (1, -1, O) y B (2, 1, - 1) que pertenecen a la superfici e 15x -
- 1y + z - 22 = o.
SOLUCIN. C-Omosesaben l a distancia entre dos puntos A (x
0
, y
0
; z
0
)
y 8 (x
1
, Ytt z
1
) en la superficie q (x, y, z) = O se determina por la
frmula
Xi
l = J y1 +u2+z'2dx,
"
donde y = y (x), z = z (x).
Se pi de, pues, hall ar el minimo de l con la condicin q> (x, y, z) =
= O. En nuestro caso,
.to= 1, XJ=2, <p (x,y,z)= J5x-7y+z- 22.
Formamos la funcional auxiliar
2
J = J lYI +1/2+z'2+)..(x) (\5x- 7y+z-22)l dx
1
y escribimos pa.ra sta las ecuaciones de Euler
). (x)( - 7)- dd (-. r y' )= j
X V 1 +u'Z+z'2
d { z' )
). (x}-l --d-x V l + y' 2+z'2 =O.
(10)
( ll)
Resol vamos el sistema de ecuaciones (10) y (11) empleando la condi
cin de enlace
l 5x - 7y + z - 22 = O. (12)
Las funciones incgnitas y= y (x) y z = z (x) satisfacen las sguientes
condiciones de fronter a:
!I ( 1) = - l, !I (2) = 1, t ( 1) = z (2) = -1. ( 13)
Multiplicando por 7 la ecuacin (11) y agregndola a (1 O), obtenemos
( y' -1- 7z' ) _
0
dx V 1 + u'i + ri -
118
CAP. 11 . EXTREMO DE FUNCIONALES
de donde
yt+y'' +z2
11'+7z'
(14)
De (12) tenemos
z' = 7y' - 15. (1 5)
In trodu ciendo este valor de 2' en (14) y resolviendo la ecuacin dife
rencial obtenida, encontramos y = C
1
x + Ci Las condiciones de
fr ontera (13) dan c
1
= 2 y = -3 de modo que
y= 2x -3. (1 6)
Teniendo en cuenta (16) , de (15) resulta
Z = l - X (1 7)
(es obvio que la funcin (l 7) s.at isface las condiciones de frontera).
De (10) o de (11 ) obtenemos /,, = O. La distancia buscada es
2
l= J lfl + y'' +z'2dx=\f6 .
1
Este resultado se puede obt ener inmediatamente de considera-
ci ones geomtricas evidentes.
30, Lneas Sea
r = r (u, v) (18)
la ecuacin vectorial de una superficie.
Se llama lnea geodsica la l inea de menor longitud que per tenece
a la superficie considerada y que une dos puntos ijos de la misma.
Las ecuacione!; de las lneas geodsicas se pueden obtener como
l as ecuaciones de Euler correspondientes al problema variacional
sobre la distancia minima en la superficie entre dos puntos fijos ..
Toda lnea perteneciente a la superfi cie r = r (u, v) se puede
representar por las ecuaciones paramtrcas
U = U (/), V _., V (t).
La longitud de su parle comprendida entre los pu ntos corre5pondientes
a los valores 1
0
y t
1
del parmetro t es igual a
ti
J[u(l) , u(t )]= J VEu' 2+2Fu'u
1
-j- Gv'2 dt, (1 9)
41
donde E, F y O son Jos coeficientes de la
de la superfi cie (18), o sea,
E ( ar or ) F ( ar or )
= auTu. = au
1
Tv,
primera forma cuadrtica
=(.E!...
au , av
Aqu (a, b) es el producto escalar de los vectores a y b.
5 9. f. XTREMO CONDICI ONADO
ll9
Para la funci onal (19) el sistema de ecuaciones de Euler tiene
la forma
Euu' 3+ 2Fuu'v' + Guv'
3
d 2 (Eu' + Fv' ) = O
V Eu": + 2Fu'u' -', Gv'2 - dt v -- 2Fu'u' +Gv
1
2 '
E
11
u'
2
-! 2Fvu'r.l+Gvtl
2
2(Fu'l Gu') = O.
V Eu':I+ 2Fu'u'-+- Gv'2 dt V Eu' <i + 2Fu'v' + Gu'2
EJEMTLO 1. Entre todas l as cur vas que estn sobre una superficie
esfrka de rado R y que unen dos puntos f ijos de la misma. halla r la
curva de longitud mnima (la curva
SOLUCJN . Sean qi y O las coordenadas del punto en l a esfer a
y sea cp <p (fl) la ecuacin de la curva pedida. Tenemos entonces
r = r (<p. 0) = x (<p. 0) l + g (<p. 6) j + z (q;. El} k ,
donde
x R cos q sen 0, y ..,,. R sen q; sen e.
z = R cos 0.
Por t>so,
E = (r ip, r qi) = R.<J. sen
9
0; G = (re. r e) = R.
2
, F = (re. rqi) = O.
De aqu. segn la frmula (19). tenemos
61 61
J[q(0)1 = R J Vd6z+sen
2
Udq': = R J

El integrando no contiene la func in incgnita qi (O) y, por eso, la
ecuacin de Euler ser
de modo que
de donde
sent O<i' (A)
donde f IP' = -:-;====::;::===-
V 1 +se ns 0<f"2 (0) '
sen2 O<p' ( 0)
v 1+ seas0q>'2 <>
e,
<>' (!:l) = C1
sen aysenza-q

a ti q sen
2
1---
st>n20
C
1
Ctd 0)
sen
2
0 V(t-CU-Cf ctg
2
6 -vc1-q)-q ctg
1
o
Integrando, obtenemos
C1ctg6
q> (6) = arccos , r + C2
V 1-C:
120
CAP. 11 EXTREMO DE FUNCIONALES
6
<p (6) = arccos (C ctg 6} + C
2
, donde
De aqu
C ctg 0= cos (<> (0) - C:?J
6
ctg 0 =A cos <p (O) + B sen cp (O), (20)
donde
A= y
8
=


Multipl icando por R sen 0 ambos miembros de (20). obtenemm.
R cos 0 = AR cos cp sen 0 + B R sen <p sen O
o. pasando a las coordenadas cartesianas,
z =Ax+ By.
Esta es la ecuacin de un plano que pasa por el centro de l a esfera
y que corta su superficie segn un cfrculo mximo. Por consiguiente,
la lnea ms corta (linea geodsica) es el arco del circulo mximo.
EJEMPLO 8. Demostrar que para una superficie de revolucin
es constante en cada uno de los puntos de las geodsicas el producto
del radio del paralelo por e.\ seno del ngulo entre la geodsica y el
meridiano (teorema de Clairaut).
SOLUCIN. En coordenadas cilfndricas la ecuacin de una super-
ficie de revolucion tiene la forma
x = p cos cp, y = p sen cp, z = f (p).
Determinemos los coeficientes E, F y O:
E = 1 + f2, F = O, G = p'.
Por eso, la dHerencial ds de la longitud de arco en la superficie de
revolucin tiene la forma
ds == V p
1
+ (l + f,t)p'
2
dq>.
En la superficie de revolucin las lneas geodsicas sern extremales
de Ja funcional
q>
) V P'+(l
qlo
Como Ja funcin integrando no contiene explcitamente cp, obtenemos
Inmediatamente
pi
i.r - const,
... p2+(l+fi>)p'
1 o. PROBLEMAS CON FRONTERAS MVI U:'.5
121
o sea, p =Const. Observando que p :: =senw (fig. 18), obte-
nemos psenca>=const que es lo que se quera demostrar.
170. Hallar la distancia ms corta entre los puntos
A (l . O, -1) y B (O, -1, 1) en la superficie x +y+ z = O.
171. Hallar las lineas geo-
dsicas del cilindro circular
r =R.
JO. Problemas variacionales
con fronteras mviles
10. Problema elemental con fron-
teras mviles. Sea F = F (x, y, y' ) una
funcin diferenciable tres veces respec-
t o a sus argumentos y sean
y = q> (x) e y = "1 (x) (1)
donde cp (x) E C
1
fa, b] y "1 (x) E
E C
1
la, b], dos curvas en el plano xOy.
Consideremos la funcional
J [y (x)J = J P (x, y, y') dx (2)
'V
Fig. 18
definida para las curvas suaves y = y (x) cuyos extremos A (x
0
, y
0
)
y B (x
1
y
1
) se encuentran en las curvas (1) de modo que !lo= <p (Xo)
e y
1
= '1)1 (.x
1
). Se pide hallar el extremo de la funckinal (2).
TEOREMA. Supongamos que en la curva Yo : y = y (x) se alcanza
el extremo de la funcional
J(y(x)J= J F( x, y, y')dx
y
entre todas la curvas de la clase C
1
que unen dos puntos arbitrarios de
dos curvas fijas y= q> (x) e y = ip (;e). Entonces ta curva 'Yo es una
extremal y en los extremos A (x
0
, y
0
) y B (x
1
, y
1
) de la curva 'Yo se cumplen
las condiciones de tran.sversalidad
f:c-xo= O, } (3)
fF+(ip'-y')F
11
.J


Es decir, para resolver el problema elemental con (ronteras
mviles es preciso:
1) Escribir y resolver la ecuacin de
Como resultado, se obtiene una famllla de extrernales /J = f (x, C
1
, Cz),
que depende de dos parmetros C
1
y Cz.
122
CAP. 11. EXTREMO DE FUNCIONALES
2) Determinar las constantes C
1
, C
2
x
0
y x
1
de las condiciones
de transversalidad (3) y de las ecuaciones
f(x
0
, Ct. C2) = q>(xo), } (
4
)
f (xi, C , Ci) = \I> (xi)
3) Calcular el ext remo de Ja funcional (2).
EJEMPLO 1. Hallar la condicin de transversalidad para la fun-
cional
XI
J (y (x)J = J f (x. y) earctg !I' V 1 dx, f (x, Y) * o.
XI)
SOLUCIN. Supongamos que el extremo de la izquierda de la ex tre-
ma! se ha fijado en el punto A (x
0
, y
0
) mientras que el extremo de Ja
derecha B (X, y, ) puede desplazarse por una curva y = 11> (x) . Tendre-
mos entonces
En nuestro caso es
F = f (x, y) r/ V 1 + yi y
l +y'
F ti'= f (x, y) earc.tg y---''-=--
V 1
La condicin de transversalidad se representa as
f t <x, u> ercti: 11' Vi + !1'2+
+ C'1>'-y')/(x,y)e11rctg11' l + u'. J 1 = 0.
V1 x=x1
De aqu obtenemos, debido a la condicin f (x. y) :::/= O,
'1>'-y'
1 + ip'y'
-1. (5)
Desde el punto de vista geomtrico, la condicin (5) gnifi ca que las
extremal es y= y (x) deben cortar la curva y = q, (x) por la cual
se desplaza el punto extremo B (xl> y
1
) de modo que el ngulo entre
n
es tas curvas sea de T
Efecti vamente, la r elacin (5) se puede tran!'.formar del modo
s iguient e: s upongamos que Ja tangente a la extrema! en el punto
B (x
1
, y
1
), que pert enece a la curva y = '11' (x), for ma ngulo a. con
el Ox y que la tangen le a 1 a curva ija y = <p (x) forma ngul o
(fig. 19); entonces, se tiene tg a.= y', lg = 'lj>' y el primer mi embro
de la frmula (5) da tg - a); pero como -1 = tg ( - : ), resulta
- a = - , de donde a = + que es lo oue se quer a demos
trar.
f 10, PROBLEMAS CON FRONTERAS MVILES
123
EJEMPLO 2. Hallar la distancia de la parbola g = X'- a la recta
x-y = 5.
SOLUCION. El problema consiste en hallar el valor extremo de la
integra l
!lt l
J [y (x)J = J V 1 + 11'
2
dx
%1)
con la condcin de que el extremo de la izquierda de la extrema! se
puede desplazar por la curva g = x mientras que el extremo de l a
Fig. 19
derecha, por la reda y = x - 5. Por consiguiente, tenemos en este
caso <p (x) = x
2
y \Ji (x) = x - 5. La solucin general de la ecuacin
de Eul er ser y = C
1
x + C
2
, donde C
1
y C, son constantes arbitrarias
que deben ser determinadas.
Las condiciones de transversalidad (3) tienen la forma
donde y' = C
1
Las ecuaci ones (4) t ienen en nuestro caso la forma
}
C1X1 +C2=X1-5.
124
C.AP. ll. fXTREMO DE FUNCIONALES
Tenemos, pues, un sistema de cuatro ecuaciones con cuatro incgni-
tas C
1
, C2. Xo y x1:
/
-- C1
l 1+q+c2xo-C1) -. r = 0,
V 1+q
-V1 +Cf+O-C1) V e, =O,
1+q

C1X1 +C2=X1-5;
resol vindolo, obtenemos
3 23
C1=- I. C2=4, xo=2 y Xt=g
Es decir, la ecuacin de la extrema! es y = -x + ! y la distancia
de la parbola a la recta es igual a
Z3
8 23
l = J lf1+(- 1)
1
dx=-V2x l: =
19
f
2
.
1
2
172. Hallar la distancia ms corta del punto A (1, 0)
a la elipse 4x
2
+ 9y-z = 36.
173. Hallar la distancia ms corta del punto A (-l. 5)
a la parbola y
2
= x.
174. Hallar la distancia ms corta de la circunferencia
x
2
+ y
2
= 1 a la recta x +y= 4.
175. Hallar la dist ancia ms corta del punto A (-1, 3)
a Ja recta y = 1 - 3x.
176. Demostrar que en el caso de la funcional
1
J[y(x)] = ) h(x, y)V1+y'
2
dx,
donde h (x, y) ::F O en los puntos frontera, las condiciones
de transversal idad tienen la forma
y' (x) = - qi'
1
(x) e y' (x) = - il>' 'ex) ,
o sea, las condiciones de transversalidad se reducen a las
condiciones de ortogonalidad.
S JO. PROBLEMAS CON FRONTERAS MVILES
125
2. Problemas con fronteras mviles para funcionales de la forma
XI
J [!/ (x), z (x)J = I F (x, y, z, !I', z') dx.
:ro
(6)
Al analizar el extremo de la f uncional (6) aceptamos que por lo
menos uno de los puntos frontera A (x
0
, y
0
, z
0
) o B {x
1
, y
1
, :z
1
) se
desplaza por una curva fija.
El extremo de J l!I (x), z (x)} se puede alcanzar slo en l as curvas
integrales del sist ema de ecuaciones de Eulcr
d }
F
11
- - F , = 0,
dx 11
F,-..!!_F. =0.
d,t
Supongamos que el punto A (x
0
y
0
, z
0
) est lijo mientras que el
otro punto frontera B (x
1
, y
1
, z
1
) se. desplaza por una curva definida
median te las ecuaciones
!I = <p (x), }
z =, {x).
En este caso, la condicin de transuersolidad tiene la forma
[F+(cp'-y') F
11
,+ (1jl'-z') Fzl lx-:ti =0.
Anlogamente se escribe la condicin de transversalidad para el
extremo de la izquierda (si .ste tambin se desplaza por una
curva g= (x) }) :
Y=1'> (x)
[F + fq' -y') lx-ro=O.
EJEMPLO 3. Hall ar l a distancia ms corta del punto M (x
0
, y
0
, z
0
)
a la re.eta
g=mx+p,}
Z=nx+ q.
SOLUCtON. El problema se reduce a la determinacin del extremo
(mlnimo) de la integral
"1
J [g (x), z {x)J= J V 1 + y'+z'dx
(7)
::ro
126
CAP. 11. EXTaEMO DE FUNCIONALES
con la condicin de que el extremo de la derecha de Ja extr ema! puede
desplazarse por 1 a recta
y=mx+p,}
(8)
Z=nx+q.
o sea, en nuestro caso las funciones rp y tienen, respectivamente,
la forma
q> (x) = mx + p y '!> (x) = nx + q.
La solucin general del correspondiente sistema de ecuaciones de
Euler ser
y=C
1
x+c
2
, }
z =C
3
x+C4,
(9)
donde C (i = 1. 2. 3 y 4) deben ser determinadas.
La condicin de lransversal idad (en el ext remo de la derecha)
tiene la forma
[
VI +Y''+z'
2
+(m-y') y' +
Vi+u't+z'i
+Cn-z') =0, z' J 1
Vt +Y'
2
+i'
2
x-x1
de donde, puesto que y' = C
1
y z' = C
3
, obtenemos
l -1- mC
1
+ nC
3
= O.
(10)
La relacin (10) expresa la condicin de perpendicularidad entre la
recta buscada (9) y la r ecta dada (8).
Empleemos el hecho de que la rect a buscada (9) pasa por el punto
M (xo. !lo zo}:
(11)
y tambin el hecho de que el ext remo de l a derecha se desplaza por
la recta (8):
Clx1+Cz = mx1+P }
C3x1-J-C4=nx
1
-!-q.
(12}
De las cinco ecuaciones ( 10, {l I) y (12) debemos determinar CJo c.,
e,, e, y X1 (xo. Yo Zo. m. n, p y q son nmeros dados). Para calcul ar
la integral (7) basta conocer Xi. C
1
y e,. Tenemos
xo+ m (110 - P}+n (io-9)
X
- i+nz+mz '
C _ mxo+mn (zo-9) -(1 +nZ) (Yo- P}
t - m(110- P)+n(zo- q)-(m
2
+ n2)xo'
C _ rtxo+ mn(yo-p)-(l+mZ)(zo- q)
3
- m(y
0
-p)+n(z
0
-q)-(mz+ nz}Xo
1 o. PROBLEMAS CON FRONTERAS MVILES
Introduciendo estos valores en (7), obtenemos
h = mn .1 (y (x), z (x)I =
_ , / x2 + (y -p)Z+ (z _ q)
2
- (xo+m (go-P)+n (z0-q)J2
- y o o o 1 + n2 + m2
127
Si el punto front era A (xo. Yo Zo) est fijo y el otro punt o rrontera
B (x
1
, y
1
, z
1
) puede desplazarse por una supt'rl icie z <p (x, y) , las
condiciones de l ransversalidad sern


Fi, ll x ... x
1
=_ }
f F
11
, + F
2
; <p
11
J lx-:ci - O.
( 13)
Las condiciones (1 3) conjunta mente con la ecuacin z = <p (x, y)
permiten determinar, hablando en trminos generales, dos constantes
arbitrarias de la solucin general del s ist ema de ecuaciones de Euler
(l as otras dos constantes se determinan de la condi cin de que la
extrema! ha de pasar por el punto fi jo A (x
0
, y
0
, z
0
)).
Si el punto mvil es el punto frontera A (x
0
, y
0
, z
0
) . obtenemos
para x =-= x
0
unas condiciones anlogas completamente a las condi-
ciones (13).
EJEMPLO 4. Hallar la distancia ms corta del punto A ( 1, 1, 1)
a la superficie esfrica
;x2 + yZ + zZ = J. (14)
SOLUCtON. El problema consiste en analizar el extremo de la
runciooal
j
J[y(.l:). z(x}J = I v1+y'2+z'2dx. (15)
Xt
donde el punto B (x
11
y
1
, z
1
) debe estar en la superficie es[rica (14) .
Las ex lremales de Ja funcional (1 5) son las rectas
y = Csx+Cz, }
( 16)
z = C
3
x+C4.
De la condicin de que Ja extremal (1 6) pase por d punto A (1,. 1, 1) ,
obtenemos
C1+C2= 1, }
l.
tienen la orma
(1 7)
128
CAP. (l. DE
t enlendo en cuenta (16), despus de unos clculos sencillos encontra-
mos de aqu
Z1-C3X1=0, }
C
1
z1-C31J1 =0,
(18)
donde x,. y
1
y z
1
son las coordenadas del punto buscado B.
De la condicin de que la extrema! (16) pasa por el punto
B {x
1
, y
1
, z
1
) tenemos
Y1=C1x1+Cz, }
z
1
=C
3
x
1
+C
4

De (17), (18) y (19) encontramos
C
1
= I, O, C3= y o
de modo que la ecuacin de la extrema] es
!J =X, }
Z=X
(19)
(20)
Puesto que el punto 8 (x
1
, y
1
, z
1
) debe estar en la superf icie esfrica
(14). obtenemos, tomando t:>n consideracin (20), que + + xf =
1
= 1, o sea. que x, = va .
Por consiguiente, obtenemos dos puntos
B (-1 _J _1 )
' -V3' V3' Y
(
1 1 1 )
B - y 3 --v3 --va
Es fcil ver, por razones geomtricas, que en la ex trema! (20) que
une los puntos A y 8
1
la funcional (15) alcanza su mnimo igual a
1
Jmtn = J V1+1 +1dx=V3-1
t
va
mientras que en la extrema! (20) que une los puntos A y esta fun
cional alcanza su mximo
1
Jmx= \ V3dx= VJ+ l.
1
- -V3
OBSERVACI N 1. Al deducir las condciones de lransversalidad (18)
hemos considerado que <> (x, g) = yt - xi - y
2
Es fci l ver que
las condiciones (18) subsisten si q> (x, y} == - V i - x'l - y
9

OBSERVACIN 2 . Queda claro, por razones geomtricas, que la
extrema) (20) es ortogonal a la s uperficie esfrica x2 + y'J + = l.
10. PROBLEMAS CON FRONTERAS MVJ LES
129
EJEMPLO 5. Consideremos el mismo problema sobre el extremo
de la funcional (15) pero tomando como A el centro de la esfera
O (O, O, O) .
SOLUCIN Las exlremalcs de la funcional son las rectas (16) y la
condicin de que la extrema! pase por el punto O (O, O, O) da inme-
diatamente C
9
= e, = O.
Las condiciones de transversalidad sern las mismas
Z1-C3X1=0, }
C1z
1
-Ca!J
1
=0,
y las condiciones en el extremo mvil sern
!/1 =C1x1. }
Zs =C3X1,
Por ltimo,
(21)
(22)
(23)
Para determinar las cinco magnitudes C, e,. Xi. Y1 y Z1 tenemos
einco relaciones (21), (22) y (23) de las cuales solo tres son indepen
qientes:
(24)
Empleando las relaciones (24), encontramos
1 l
Xt=l(l-1-C:+q' Y1=v1+q+q'
1
z l = ":'":'-;:::::::::;;:;;:=::;::;-
V 1-1-q+q'
donde C
1
y C
3
son unas constantes arbitrarias.
Consideraciones geomtricas aclaran esta arbitrariedad: la distan
cia del punto O (O, o. O) a la superficie esfrica (14) es la misma en
cualquier direccin, o sea, para cualesquiera valores de C
1
y C
3

El valor de la funcional J [y (x), z (x)] en las extremales
es igual a
9-01387
y=C1x. }
z=Csx
Vi+chci
Jy(x),z(x)J= v1+q+qdx=I.
o
130
CAP. 11. EXTRf!MO DB f'UNCIONALe.S
EJEMPLO 6. Hallar ta condicin de transversalidad para Ja fun
e ion al
%j
J[u(x), z(x)J = J f(x, !f, z)"J( I +u'z+z'2dx, (25)
si el punto A (x
0
, g
0
, z
0
) est fijo y el punto B (x
1
, y
1
, 2
1
) se encuentra
en 1 a superficie z = q> (x, y).
SOLUCJON. En este caso las condiciones de transversalidad sern
(l lx-xt =0, }
(y' + <i>vZ') l;l;lal%J = 0,
o sea,
' I 1 l
q>; x .... :ic = cpv X'o:o.ltl =-=-r - :ic1.
ta condicin de paralellsmo del vector -r {I, y', z'}
tangente en et runto B (x., fl' Zt) a la extrema! buscada y del vector
n {cp;, - 1 de la norma a la superficie z = q> (x, y) en este mismo
punto. P-or consiguiente, para las funclonales de la forma (25) las
con,diciones de transversal ida d se reducen a las condiciones de ortogo-
halidad.
177. Demostrar que, si la condicin de transversalidad
y la condicin de ortogonalidad coinciden para todos loo
datos iniciales, la funcin integrando F tiene la estructura
siguiente:.
P = f (x. y, z) VI+ y'
2
+z'
2
,
donde f (x, y, z) es una funcin diferenciable cualquiera de
X, !J y Z.
178. Ha!Jar la distancia ms corta del punto M (O, O, 3)
a la superficie z = x
2
+ yi.
t 79. Hallar la distancia ms corta del punto M (2, O, 5)
a la superficie z = + y".
180. Hallar la distancia ms corta entre las superficies
+ + = 1 y x2+!f+z2=4.
181. Analizar el extremo de la funcional

/ [y(x)
1
z (x)I= J (y'
2
+z'+2yz)dx
o
s 10. PROBLEMAS CON PRONtERAS MOVI LES
131
si y {O) = z (O) = O y el punto B (x
1
, y
1
, z
1
) se desplaza por
el plano x = x
1

3. Distancia geodsica, El valor de la integral
B
J [y (x)) = J F (x, y, y') dx,

(26)
ca1culada segn una linea T desde el punto A hasta el punto B, se
denomina J-longitud de la nea y. Si y es una extrema! se dice que
J [y (x)] es la distancia geodlsica entre Los puntos A y B, o simplemente
Jdistancla, y la propia extremsl se demomina J-recta.
EJEMPLO 7. Hallar la distancia geodsica del pun.to A (O, O) al
punto B (1, 1) si esta distancia se define mediante la funcional
B
J [y (x)) = J !Pr/2 dx.
A
SOLUCION. La distancia geodsica del punto A al punto B es Igual
al valor de esta funcional en la extremar que une dichos puntos. La
ecuacin de Euler es
2yy'Z - :X (2g2y') = 0 !JY'+y'2= 0.
Es Hici 1 ver que
d
llJI" + g'2 = dX (gy'),
de modo que 2yy' = C
1
e y
1
= C
1
x + C
9
Empleando las condiciones
de frontera !J l:r=o = O e y 1 x=i = 1, obtenemos C
1
= 1 y Ca = O.
Por consiguiente, la extrema! que une los puntos A y 8 es la parbola
yl =X.
Tenemos ahora 2yy' = 1, yy' = !11. por consiguiente, (yy') = ! .
Por defi nicin, la distancia geodsica entre los puntos A y B es igual a
f.
l (A, B)= J ! dx= ! .
o
Supongamos que se tiene una linea Z: q> (x, y) = O.
la distancia geodsica entre un punio B que no pertenece a :C
y esta linea se defi ne como la distancia geodsica del punto B a un
punto A E :C que se obtiene calculando la funcional (26) segn Ja
extremal 'i que une los puntos B y A con la particularidad de que y
corta transversalmente Ja lfnea :C en el punto A.
CAP. 11 . EXTREMO OE FUN lONALS
Se denomina !circunferencia (o circunferencia qeodtsica) la lnea
formada por los puntos que esUn a una misma distancia geodsica
de un punto ijo. Anlogamente se definen los conceptos de / elipse
y de J hiphbola.
EJEMPLO s. Hallar la / circunferencia de radio R y con centro
en el punto O (O, O) si l a distancia geodsica se define mediante la
funcional
B
J (y (x)J = J y'2/ 2 dx.
A
SOLUCION. Las exlremales de la funcional corlan transversal-
mente la circunferenciageodsica. Para las extremales tenemos (vase
el ejemplo anterior)
g = C
1
x, 2yy' = C
1
y, por consiguiente,
,,1 _ y
" - 2x
De la condicin de transversalidad
y'y' (2q>' - y') = o
encontramos que el coeficiente angular de la tangente a la Jcircun
ferencia es q>' = y; y, por eso, la ecuacin diferencial de la J-circun
ferencia es y'= fx , de donde resulta la ecuacin de la / -circunfe-
rencia: y' = Cx. Para determinar el valor de C observemos que el
punto (to, C) est en la circunferencia geodsica y que la ecuacin del
radio geodsico (o sea, de la extrema\) que pasa por dicho punto es
!f = ~ . De aqui tenemos yy' = ~ y, por lo tanto,
es es
R= J (yy')2dx= J ~
2
dx= ~
o o
Es decir, C = 4R y la ecuacin de la circunferencia geodsica de ra
dio R. y con centro en el origen de coordenadas es !!' = 4Rx.
EJEMPLO 9. Hallar la / circunferencia de radio R. y con centro en
el runto O (0, O} si la distancia geodsica se define mediante la funcio-
na
B
Jy(x)J= J lft+y'2dx.
A
10. PROBLEMAS CON FRONTERAS MVILES
133
SOLUCIN. Las exlremales de la funcional son las rectas y=
= C1X + e,. De la condicin de que la extrema! deba pasar por el
punto O (O, O) encontramos C
9
= O de modo que y= C
1
x y, por consi-
guiente, y' = JL
X
La condici n de transversalldad coincide en este caso con la
condicin de ortogonalidad y, por eso, el coeficiente angular de la
tangente a !circunferencia es -q>' = - :, . Por consiguiente, la
ecuacin diferencial de la /-circunfenca es y'= - ~ De aqur resulta
y
la ecuacin de la !circunferencia: x
2
+y" = C'. El punto (C, O)
est en dicha circunferencia. La ecuacin del radio geodsco que pasa
por este punto es y = O de modo que y' = O y
e
R= ) dx -c.
o
Es decir, C = R y la ecuacin de la circunferencia geodsica buscada
de radio R es la ecuacin de la circunferencia corrien te x.
2
+ y
2
= R
2

OBSERVACION. Los conceptos Introducidos permiten hablar de la
Geometrla no euclidea con la diferencial de arco
ds = F (x, y, y'} dx.
Si F = Yl + y'2, las /-rectas se convierten, como hemos visto, en
l as rectas corrientes y nuestra Geometra se convierte en l a euclldea
corriente.
Si F es una funcin arbitraria, que slo satislace las condiciones
habitual es de ser continua y derivabl e respecto a los tres argumentos,
1 a Geomelria construida muy poco recuerda 1 a corriente: no siempre
se puede trazar una J -recta por dos puntos y puede suceder que por
dos puntos pasen varias J rectas y, por consiguiente, que la J distancia
entre dos puntos no sea una funcin unvoca de las coordenadas.
182. Hallar la distancia geodsica del punto A (O, O)
al punto B (1, 2) si esta distancia se define- mediante la
funcional
J(y(x)J=) (!f+y'
2
)dx.
183. Hallar la distancia geodsica del punto A (O, l) al
punto B (1, l) si esta distancia se define mediante la funcional
J [y (x)J = ) (12xy+ !/
2
) dx.
184. Hallar la J-circ:unferencia de radio R = 8 y con
centro en el punto O (O, O) si Ja distanci a geodsica se define
134
CAP. 11. EXTREMO DE. FUNCIONALES
mediante Ja funcional
J [y (x)J = J y'
3
dx.
t t. Problemas discontinuos. Variaciones unilaterales
10. Problemas discontinuos. La extrema! y=y (x) de la funcional
::<
J (y (x)] = J F (x, y, y') dx (1)
xo
es una funcin que tiene dos derivadas continuas siempre que la deri
vada F V'll' (x, y (x). y' {x)) sea diferente de cero. Sin embargo, existen
problemas variacionales en los cuales el extremo se alcanza en una
curva suave a trozos solamente.
o) PROBLEMAS DISCONTINUOS DE PRIMERA ESPECIE. Consideremos
el problema sobre la determinacin del extremo de la funcional (l)
aceptando que las curvas admisibles satisfacen las condiciones de
frontera
(2)
y pueden tener un punto angular en el punto de abscisa e (x
0
<e < x
1
).
Este punto angular puede darse slo all donde F
11

11
= O (vase el
teorema 2 de !a pg. 53). En el punto angular Ja extremal debe satisfa-
cer las con.diciones de Weierstrass - Erdmann




}
(F-y' F
11
,) ... c-o-(F-y' F
11
,) b:=e+o=O.
(3}
Conjuntamente con las condiciones de continuidad de la extremal
buscada, estas condiciones permiten determinar las coordenadas del
punto angular.
En cada uno de los segmentos [x
0
, e) y [e, xi] la extrema! debe
satisfacer la ec.uacin de Euler, o sea, una ecuacin diferencial de
segundo orden. Al resolver estas dos ecuaciones se obtienen cuatro
constantes arbtrarias que, hablando en trminos generales, se deter
minan de las condiciones de frontera (2) y de las condiciones (3) en el
punto angular.
EJEMPLO 1. Hallar las extremales quebradas (si es que
de la funcional
a
JI!! {x)) = J (y'2- y2) dx,
Q
11. DISCONTINUOS
1. 35
SOLUClN. Escribimos la primera de las condiciones (3) que deben
cumplirse en el punto angular:
F ll' b:-c-o = F v' (O <e < a).
En nuestro caso tiene la forma
y' (e - O) = y' (e + O)
y significa que l a derivada y' (x) es continua en x = c. Por conslgulen-
!e, no hay puntos angulares. Esto se puede ver ta mbin de que en
nuestro caso F
11
,
11
, = 2 >O en todo punto. Por lo ta nto, en el proble-
ma considerado puede alcanzarse slo en curvas suaves.
EJEMPLO 2. ttallar las exl remal es quebradas de la funcional
2
J(g(x}J= S (g'4-6g'2)dx; y(0)=0, y(2)=0;
o
aceptando que y' puede ser discontinua en el punto correspondiente
a la abscisa x = c.
SOLUCION. En este caso F
11
,
11
, = 12y'2- I2 se puede anular, y, por
eso, puede ocurrir que la extrema! t enga puntos angulares. Puesto
que la funcin integrando depende slo de y', las extremales son l as
rectas
Pongamos
y_ = mx + /1 (O < x < e) e Y+ = px + q (e < x 2).
De las condiciones de frontera encontramos n = O y q = - 2p de
modo que
Y- = mx e IJ+ = p (x - 2).
La condicin de continuidad de l a extremal da
me= p (e - 2). (4)
Escribamos las condiciones de Weierstrass - Erdmann. Tenemos
F
11
,=4g'S-t2y' }
F-y'Fv, = - 3r'' + 6y'2.
Puesto que y:_= m e u: = p, obtenemos
o sea..
4m3-12m=4p3-12p, }
-3m' +6m2= -3P'+6p2.,
(m- p) (m2+mp+p2- 3)=0, }
(m'l. - p?.)
(5)
136
CAP. 11. EXTREMO DE FUNCIONALES
La segunda de las ecuaciones (5) da inmediatamente m = p, m = -p o
m' + p
1
- 2 = O.
La solucin m = p debe ser excluida: en este caso la extrema\ tiene
derivada continua y de Ja condicin (4) obtenemos m = O, o sea, la
extrema! es un segmento del eje Ox.
Por consiguiente, para resolver el sistema (5) hay que resolver
dos sistemas de ecuaciones:
y
m= -p, } 6)
(
m2+p2=2, }
m2+mp+P2=3.
(7)
La solucin del sistema (6) es: m=Y3, p=-113 y m = - y3,
p = y3. La sol ucin del sistema (7) es m = p y debe se.r excluida.
Es decir, m ::: -p y la condicin de continuidad (4) da e '= 1.
Por consiguiente, las extremal es buscadas son:
Y=
{
"V3x,
-V3 (x- 2),
e
u=
{
--V3x,
Y3(x-2), 1
185. Hallar las extremales con punto angular para la
funcional
2
J(y(x)] = J y'
2
(y' - l)
2
dx; y(O)=O, y(2) = 1.
o
186. Hall ar la solucin c.on un punto angular en el pro-
bl ema sobre el mnimo de la funcional
4
J(y(x)] = J (y' - 1)
1
(y'+1)
2
dx; y (O)= O, y (4) = 2.
o
187. Existen soluciones con puntos angulares en el
problema sobre el extremo de la funcional
x1
J (y (x) 1 = J (y'
2
+ 2xy-!/') dx; Y (Xo) = Yo1 !J {x1) = Yt?
.lCg
11. PROBLEMAS DISCONTINUOS
137
188. Hall ar la solucin con punto angular en el problema
sobre el extremo de la funcional
1
J {y(x) J= J y2(1-y'
2
)dx; y( - 1) =. 0, y(l )=l.
- 1
189. Hallar la solucin con punto angular en el problema
sobre el mnimo de la funcional
:l<1
J (y'' - 2y'
2
) dx.
%1
190. En el problema sobre el extremo de la funcional
(:l<'r 111)
J sen y' dx
(0, 0)
hallar ta solucin continua y la solucin con punto angular.
OBSERVACIN. Las condiciones (3) de Weierslrass-Erdmann
admiten la siguiente interpretacin geomtrica.
Consideremos la fguratriz o sea, la curva Y = F (x, y, y') cm
tanto que funcin de y'.
Las condiciones (3) signHican entonces que para los valores de los
parmetros x = e y y = c
1
, que corresponden al punto angular, la
figuratriz debe tener una misma tangente en Jos puntos de abscisas
y:._ = y' (e - O) e yi, = g' (e + O).
Al mismo tiempo se obtiene una Interpretacin cl ara de la condl
cin F
11

11
=PO que excluye la posibilidad de puntos angulares en las
extremales. Efectivamente, si, por ejemplo, es F
11
,
11
> O, la fgurat riz
es cncava hada las Y positivas y no podrn coincidir sus tangentes
trazadas en dos puntos distintos. Es dec ir, en este caso la extrema!
no podr tener punto angular.
Consideremos de nuevo el problema sobre la determinacin de las
ex trema les quebradas de la funcional del ejemplo 2 de este pargrafo.
Tenemos
2
Jly(x)]= J (y' ' -6y'2)dx; y(O) = O, y{2) = 0.
o
Las extremales son rectas. En este caso la figuratrlz Y = y'
4
-
- Gv'
2
no depende del punto (x, y) . Tiene una tangente comn eJl
138
CAP. JI . EXTREMO DB PUNCIONALES
Jos puntos de abscisas y'= 1(3 (fig. 20). Por eso, las condiciones
de Weierstrass-Erdmann quedarn cumplidas si como extremales
y
Fig. 20
quebradas tomamos las quebradas cuyos lados formen ngulos de
con el eje Ox.
En la quebrada y
1
con un punto angular (fig. 21) l a funcional
toma el valor J fyd = - 18. Este mismo valor tendr J (y (x)] en la
y
X
Fig. 21
quebrada y
11
con dos puntos angulares, (ig. 22), en la quebrada y
1
con t res puntos (fig. 23), etc.
b) PROBLEMAS DISCONTINUOS DE SEGUNDA ESPECle. Se denominan
problemas dist;onlinuos de segunda especie los sobre el extre-
s 11.
PROBLEMAS DtSCONTI NUOS
139
mo de la funcional
XI
J[(x)J= J F(x, y, g')dx; g(x1)=Y y(x2)=v2; (8)
;X:f
en la que la funcin Integrando es di sconti nua.
Supongamos, por ejemplo. que F (x. y, y') es discontinua a lo
largo de la curva y = <!> (x) y sea F (x, y, y') igual a F
1
(x, y, y')
a un lado de la lnea y = di (x) e Igual a F
2
(x, y, y') al otro lado.
y
2 X
Fig. 22
SI existe la extrema\ quebrada, deb.er componerse de los trozos
de las extremales y = y
1
(x) e y = Ys (x) que tienen un punto comn
(e, <!> (e)), e E (x
1
, xa), en la linea de discontinuidad. Para hallar l a
y
X
Flg. 23
extremal quebrada obtenemos dos ecuaciones diferenciales de Euler
cuyas soluciones generales contienen cuatro constantes arbitrarlas C
1
,
C
9
, C
1
y e,. Para determinar estas constantes as! como la abscisa e
del punto en el que la extrema! encuentra la curva g = <!> (x) tenemos:
1) dos condiciones de fronteras (8), 2) dos condiciones segn las cuales
las ordenadas de los extremos de las extremales en el punto de juncin
han de ser iguales a la ordenada de la curva .Y = <l> (x) y, por ltimo,
3) la condici6n de la jun.cin
F1 +(q>' -() f w lii:-c-o """ .F2+ (<I>' -y') !'211
140
CAP. 11. EXTREMO DE FUNCIONALES
Hablando en trminos generales, estas condiciones alcanzan para
determinar la exfremal quebrada.
EJEMPLO a (problema de la refraccin de un rayo de luz). La luz
se propaga con una velocidad constante u
1
en el medio I y con una
ve.Jocidad cqnstante v
2
en el medio l l. La curva y= <I> (x) separa los
medios I y I l.
Deducir la ley_ de refraccin del rayo de luz que va desde el punto
A del medio l hasta el punto B del medio 11 si se sabe que es minimo
el tiempo durante el cual el rayo de luz recorre este camino.
SOLUCl N. El .problema consiste en hallar el mlnlmo de la inetgral
J [Y (x)J= r Vf+Y'2 dx+ Jb lt'T+vz dx,
J Ot V2
(10)
o t:
ya que la primer.a y la segunda integrales de (10) representan, respecti
vamente, el tiempo que necesita el rayo de lu.t para llegar del punto A
a la lfnea de separacin y de la linea de separacin al punto B.
Tenemos un prot>lema discontinuo de segunda especie siendo
Para determinar los trozos de las extremales debemos hallar l as extre-
males de la funcional
J V l+y'Zdx
que, como se sabe, son rectas. Por consiguiente
!lt =mx+n e y
2
=px+ q.
Escribamos la condicin (9). Tenemos
F _ aF t _ l f Yi'
1
!li - t11 uqf 1 + Y' Ut V 1 + Y'
, i1F2 1
Fz-y,-, =
au, 2 V'+ y;
Introduciendo estas expresiones en (9), encontramos
1 + <ll 'Y
v1-V1 +u
1
1 +a>'y
oi-V1+y;2
(11)
Sea y el ngulo que forma con el eje Ox la tangente a la lfnea de
racin en el punto de abscisa e, sea a el ngulo que forma con el eje
Ox el rayo de la izquierda y sea f} el ngulo que forma con el eje Ox
el rayo de la derecha. EntQnces, se tiene - tg y, Yi. == tg a, yp =
s t l. OISONTINUOS
= tg y la condicin (11) toma la forma
o
1 + tg a tg i' l + tg tg -y
v
1
V1 + tgz a V l +
cos (y-ex)
Ut

Vz
141
donde y - e.t y i' - son los ngulos entre los rayos y la tangente
a l a llnea de separacin. Tomando en lugar de los ngulos cp y 0
entre la normal a l a lnea de separacin y los rayos, incident e y re-
fractado, obtenemos
sen q> v1
--=-=consl,
sen a Clz
o sea, l a ley de refraccin del rayo de luz.
20. Vari aciones unila t erales. Se pide hallar el ext remo de la
funcional

J [y (x)) = J F (x, y, y') dx;
a;
con la condicin
y - q> (x) O ( y - cp (x) O) (12)
(l as condiciones de limitacin pueden ser de forma ms compleja).
En este caso Ja extrema! busca da puede estar formada por trozos
de extremales que pertenecen al recinto (12) y por trozos de la fronte
ra y = q> (x) de este recinto. En Jos puntos de j uncin de estos trozos
la ex1remal buscada puede ser suave y tambin puede tener puntos
angulares.
La condicin en el punto de juncin tiene la forma
[F(x, y, y')-F(x, y, ip')-(q' - g')F
11
.(x, y, y' )J I - = O.
;ll:mo2:
_Si_F..,,
11
, ::FO, la extrema! es t angente en el punto de juncln
M (x, y) a la frontera y = q> (x) del recinto.
EJEMPLO 4. Hallar en el recinto y x
2
el camino ms corto del
punto A (-2, 3) al punto B (2, 3).
SOLUCION. El problema consiste en hallar el ext remo de l a funcio
nal
con las condiciones
2
1 IY <x) 1 = j V 1 +u i d.x
-2
!J x' , y (-2) = 3, y (2) = 3.
(13)
i42
CAP. 11. extREMO DE PUNC!ONALS
Las extremales de la funcional (13) son las rectas
y= C
1
+ C
1
x.
En nuestro caso
Fvv = :1+0
(l+y'2)2
y la extrema! buscada se compone de los trozos AM y NB de las rectas
tangentes a la parbola y= xi y del trozo MON de esta parbola
(fig. 24). Representemos las abscisas de los puntos de tangencia por x
y -i (utilizamos la simetra del problema). En el punto de tangencia
y
A(-2,J)
2 )(
Fig. 24
coinciden las ordenadas y los coeficientes angulares de la recta y de la
tangente a la parbola de modo se tiene
C1 C a i = ~ : } (l)
Cz=2x.
Por otra parte, la tangente debe pasar por el punto B (2, 3) y, por
consiguiente,
C
1
+ 2C, = 3.
(15)
Eliminando C
1
y C, de (14) y (15). encontramos x' - 4.i + 3 = O,
de donde i
1
= 1 y i, == 3. El segundo valor de x no sirve. Es decir,
i = 1 y C1 = -1 y C, = 2. La extrema! buscada (nica) es
{
-2x- l si
11 =- x?. si
2x-l si
- 2<;:x<-l.
-1 <x< l,
1 < x<;:2.
Queda claro que la funcional (13) alcanza en ella su minimo.
TE.ORfA DE HAMIL tON .. JACBl
143
191. Hallar las curvas en las cuales puede alcanzarse el
extremo de la funcional
10
J[y(x)]= J y'
3
dx; y(O) = , y(lO) = O;
o
si las curvas admisibles no pueden pasar por el interior del
crculo que limita la circunferencia (x - 5)
2
+ yz = 9.
192. Entre las curvas que unen los puntos A (a, y
0
)
y B (b, y
1
) hallar la curva que ofrece el valor extremo a la
funcional
l>
1 y <xH == j y V 1 - b'2u'
2
dx
a
con las condiciones y;;::r:o, 1- y2y'
2
;;;:r:o.
12. Teora de Hamiltoo-Jacobi. Principios variaclo
nales de la Mecnica
I. Forma cannica (hamlltoniana) de las 1:.cu1tiones de Euler.
Las ecuaciones de Euler para la funcional
J [Y1t Y2 ., Ynl=
= r F (x, Yt. !h., ., Yn Y1 y;, ... , ~ dx
Xt
tienen la forma
d
F
11
--d F ,-o {k= 1, 2, .. ., n).
ll X lllt.
En e.I caso en el que el determinante
F 11'11' F 11' 11' '' ' F 11' v'
11 1a in
F11' 11'F 11'11' F 11'u '
a1 t2 zn *
pondremos
F , = Pk (k = l , 2, .. , n).
111
De las ecuaciones (4) se puede expresar yi en trmi nos de x, JJi, y,,
Yn Pi. Pi. ., Pn=
g = CJ>1t. {x, Y1, Y ., Yn P11 Ps. Pn)
( 1)
(2)
(3)
(4)
i 44
CAi'>. JI. EXTREMO OE FUNCIONALES
La runcin H de las variables x, y
1
, y, , . . ., y
71
, p
1
, p
2
, , , Pn
definida media nte Ja igualdad
ff = [-F(x, !lt Y'2. ... , Yn Y y;, ... , y,)+
n
+ u., F , (x, Y1. u2 ..... u, .. u;.. .. . , 1 , ,

se denomina hamtltoniano de la funcional ( 1 ).
El hamiltoniano satisface las relaciones s iguientes
iJH dg11, iJH dp1t
{Jph = ""iiX, iJy1 = - dx (k = l, 2, . .. , n). (5)
Se dice que las ecuaciones (5) son el sistema cannico o hamilfoniano
de las ecuaciones de Euler (2); las variables y
1
, !lt . . , Yn Pt> Pi .. .
. . . , Pn ll evan el nombre de oariables cannicas.
OBSER.VACION 1. La condicin (3) en el caso de la funcional
oca
J (y (x)l = J F (x, y, y') ) dx da F ll'v' =F O en [x i . x2).
:i;
OBSER.VACJON 2. Hablando en trminos generales, las ecuaciones
(4) no se pueden resolver unfvocamente respecto a yk en todo el seg-
mento lxir x
1
) . Si se cumplen las condiciones del teorema de existencia
de l a funcin implicita, las ecuaciones (4) admiten solucin unvoca
localmente.
EJEMPLO 1. Formar el sistema cannico de las ecuaciones de Euler
para 1 a funcional
;(
J [Y1 !12)= J


o
SOLUCIN. En nuestro caso
F = F (x, Y1. Yi. Y. Y2) = 2YiYs - + Y
2
- Y*
Ponemos
Entonces
F = P1
lit
Y F , = Ps
112
P1 = 2y.i Y Ps = -2y;.
Aqui el determinante
F1111 Fll'll ' 12 O 1
l 1 l s = o - 2 = -4-:;. o.
Fll"ll' Fll'V'
2 1 8 2
Resolv!end.o respecto a Y e y2 las relaciones obteni das, encont ramos
Y
,_ Pt e y' P2
1 -2 ,=-2
12. TEORfA DE HAMILTON JACOBI
Formamos el hamlltoniano de la funcional considerada
H = (-F +11i.F
11
+ yF u)
1 I Pt
11
1- 2
11'=-P
2
p: PI
, 111 =2g,-2111112+4-- 4
11
1 ... T

:a 2
145
Empleando las relaciones (5), obtenemos el sistema cannico de las
ecuaciones de Euler
dyt = .!!l..
dx 2 '
dp, .
dx = -4Yt +2112;
}
dx 2 '
dp2 _ 211
-;- t .
Aqu y
1
= Y1 (x), Ya = /l:i (x), P1 =Pi (x) Y Ps = Pa (x) son funciones
Incgnitas de x.
EJEMPLO 2. Formar el sistema cannico de las ecuaciones de Euler
para la funcional
1b'11 Yz)= J yfyl(x
2
+ Yi+ll)dx.
SOLUCIN. Aqui
F = 11f11I (x
11
+ Y + u).
Determinamos las derivadas parciales
F
11
= Y?YI. y F
11
= Y?YI
1
Ponemos
Pt Y
Estas relaciones no comprenden las derivadas vi e Yi de las funciones
Incgnitas y
1
e y
2
; por eso, no se puede expresar y' e Y en trminos
de p
1
y p
1
Por consiguiente, no se puede formar e\ hamiltoniano de
esta funcional. En este ejemplo no se cumple la condicin (3):
1
F llll F t1lf \-1 O O 1-O
F
11
,
11
, F
11

11
, - O O = .
1 1 s
EJEMPLO s. Formar el sistema cannico de las ecuaciones de
Euler para la funcion al
J [y (x) 1 = J xgy'
3
dx.
10- 01581
146 CAP. 11 . EXTREMO De FUNCIONALES
SOLUCIN. Tenemos
F = xyy'
3
y F
11
, = 3xyy'!.
Pongamos p = 3:r11y'
2
, de donde
.e
La funcional considerada tene dos hamlltonianos
H1=(-F+y'F11.1 vp-=
11--
=2xyy'3 I .. / P
3
11
, __ , rv sva V xg
V 3xV
1
2 ;;a
fl1 = (-F+y'F11' "'-l P =3V3 V :cy,
" ll:tl/
En concordancia con esto obtenemos dos sistemas cannicos de las
ecuaciones de Euler:
dy , /p l
dx = r 3xy.
dp l .. /7 J
"dX=s V ax
11
a
Formar los sistema cannicos de las ecuaci'ones de Euler
para las funcionales siguientes:
193. J [y (x)} = J X!J V y' dx.
194. J [y (x)J = J xyy'
2
dx.
195. J [y(x)J = J J/ x
2
+ y
2
J(l +x':>. dx.
t 96. J[y1. Y2} = J (y;
2
+ y: +

dx.
s 12. 'l'EOIHA t>E HAMILTON-J ACOBI
197. JLyt. y
2
) = J


198. J l!ltt Y21 = J ( 2.xgt - y .. y;) dx.
2. Ecuacin de Hamllton-Jacobl. Teorema de Jacobl. El
sistema cannico (5) de las ecuaciones de Euler es el sistema de ecua-
ciones de Euler para la funcional
J fYt U2, ., Ynl =
= r [ -H (x, Yh flz, .. , !In Pt /Ja, . , Pn)] dx
ot( l-t
si Yi !Jt .. , !Jru P1t p
1
, , Pn se consideran en tanto que fun
clones incgnitas de x.
Esta funcional J es la solucin de la ecuacin en derivadas par
dales de primer orden
aw ( aw iJW ow )
-iJ + H x, Yt. yz, ., Yn -
0
, ., -iJ = O,
x !lt '-'112 ':In
que se denomina ecuacin t Hamilton-Jacobi.
Tl?OR.EMA DE JACOBJ . Supongamos qu.e W es la integral completa
de la ecuacin de Hamllton- Jacobi y satisface la condicin
iJ2W 02W
J2W
Yt 8Ct
iJy, 8C2
.. .
lit iJCn
iJ2W
01-W
a2w
iJy20Ct iJy2iJC2
. iJ112iJCn
+o.
iJ2W o:>.W 02W
iJyn iJC t iJyn lC2 ... aun iJC,.
Entonces las igualdades
aw
iJC11. =B1t.,
aw
iJy" =P1i (k-1, 2, . .. ,n),
don.de C11. y B1c son unas constantes arbitrarias, determinan u.na solucin
MI sistema cannico (5) dependiente M 2n constan/es arbitrarias.
EJEMPLO 4. Hallar las extremales de la funcional
J IY (x)] =11:J Y x
2
+ !12 V l + y'
2
dx
:ti
utilizando la solucin de la ecuacin de Hamilton-J acobi.
10
148 CAP. 11. EXTREMO DE FUNCIONALES
sotuc10N. Para obtener la ecuacin de Hamilton- J acobi forma
mos el hamiltoni ano de la funcional considerada. Tenemos
H= - lf x2+y2- p2.
La ecuacin de Hami lton-Jacobi tiene la forma
aw /, (w )2
- - l .- xz+ yz- - =O
iJx r iJy

(
oW)2 (W)2
ax + Tu = xz+yz.
(6)
la ecuacin (6) en la forma
( r -xZ + ( )2 - yz =o
y apliquemos el mtodo de separacin de variables. Queda claro que
la ecuacin (6) se verHca si se exige que
(
aw ) 2 _ xz = _e Y ( aw ) 2 _ yz =e
ax ay
1
donde C es una constante arbitraria. De aqu encontramos
aw =Vxz- c aw =1f y2+c.
ax y ov
La integral completa de la ecuacin (6) ser
W == J lfx2-C dx+ ) V !!2+Cdy=
=..!_x,l'xz- c-.E..1n) x+ V xz- cl+..!..y Y y2+c+
2 V 2 2
+ ln (y+ VY
2
+c l+Co,
donde C y C
0
son constantes arbitrarias.
De la relacin ={, donde A es una constan le arbitrara,
determinamos l a solucin general de la ecuacin de Euler. Tenemos
X 11 'l /-, C l
---=-.........,- --- x+ v x2-C +- -+
4 V x2 e 2 4 (x+ VxZ-C) 1f x2-C
++
11
;+c+-} ln ly+VY
2
+cl+
e t A'
+--(y+ -V y2+c) V yz+c T
12.
TEORA DE HAMlLTONJACOBI
Despus de unas simplificaciones sencillas, ohtenemos
In u+ VY2+"C l=A, o sea,
x+Vx2-C
(A= eA.),
de donde resulta definitivamente
es decir, una familia de hiprbolas.
u+Vyq:G =A
x+lfx2-C
Hallar las extremales de las funcionales siguientes:
:x:2
199. J [y(x)] = j xyV y' dx.
;q
e
200. J{y(x)J=jxyy''<-dx; y(l)=O, y(e)=l.
1
201. J [y(x)J= r G(y)Vl +y'
2
dx.
202. Hallar el mnimo de la funcional
1
J {y(x)] = J (f y'
2
+ yy' +y'+ y) dx
o
149
si se desconocen los valores en los extremos del segmento.
203. Hallar la funcin del campo p (x, y) y el propio
campo de extremales que pasan por el origen de coordenadas
para la funcional
c:x:.11>v--
Jfy(x)J=)
1
:Y'
2
dx (y>O).
(0 , 0)
204. Entre las lneas que unen el punto x = O con el
punto M1 (xlt y
1
), donde X1 >O e Y1 > O, hallar la lnea
en la que alcanza su mnimo la funcional
150
CAP. H . EXT'REMO DE FUNCIONALES
XI
J [y (x)] = ) "Vr+?2 dx (y> O).
o !I
Supongamos que se tiene Ja funcional
%
J [Y (x)[ = J F (x, y, y') dx
Xf
y se conoce su campo de extremales y = cp (x, C). Entonces en todo
punto del campo se conoce l a direccin de la transversal del campo que
pasa por este punto. Todas las transversales del e.ampo se obtienen
como las soluciones de la ecuacin diferencial de primer orden
F
11
, [x, qi (x, C), (x, C)] = H {x, q> (x, C), <Px (x, C)),
donde en lugar del parmetro C, que determina las exlremales del
campo, hay que introducir su expresin en trminos de las coordenadas
de los puntos del campo. Aqul H (x, y, p) es el hamiltoniano.
EJEMPLO 6. Hallar las transversales para el campo de extremales
y = Cx de la funcional
J [y (x}] = r y'2 dx.
:Et
SOLUCIN. Formamos el hamiltoniano de la funcional considerada.
Tenemos
F = y'2 y F '//' = 2y' {F
11
,
11
,2 =I= O).
Poniendo P=F
11
,, encontramos y
H= C-fl'2+2y'g' ) 1 = .
,,,_
Las transversales se obtienen resol viendo la ecuacin diferencial
F
11
' l11-C% = H IJ>-2'//' - zc t
donde en lugar de e hay que tomar su expresin en trminos de las
coordenadas de los puntos del campo: C = ..f!_. Tenemos
)(,
2y' lv-cx = /p-2c 2C = C2.
Puesto que C *O, se tiene 2 =C, o sea, 2 = . De aquf
cue ta de transversales son las parbolas 112===
12. TEOR!A DE HAM!L TON- JACOBI
151
205. Hallar las transversales del campo de extremales
y = Cx de la funcional

J [y (x)J = J F (y') dx.
206. Hallar las transversales del campo de extremales
y = X+ C de la funciona!
:1:2
J (y (x)) = J (xy'' -2yy'
3
) dx.
:q
207. Hallar las transversales del campo de extremales
x2
y= x-c de fa funcional
:r2
J [y(x)]= J Vy(l-y'
2
)dx (C>O, x>O,
Conociendo la ecuacin de Hamilton-J acobi
aw { aw)
ax +H x, u. au =O
de la funcional
:x:a
J [!I (x)J = J F (x, y, ;y') dx,
=1
se puede reconstruir la funcin integrando F (x, y. y'). Esta es solu-
cin de la ecuacin diferencial de primer orden
F - zF; = -H (,t', y, (7)
donde H (x, IJ, p) es el hamiltoniano de la funcional considerada
y F (x, y, z) es 1 a funcin incgnita (se considera que x e y son par
metros). Despus de determinar F (x, y, z) hay que tomar en ella la
derivada y' en lugar de z.
OBSERVAC.I(>N. La ecuacin (7) es la ecuacin de Clairaut. Como
regla, la solucin general de la ecuacin de Clairau t se omite pues
en este caso l a funcin integrando F (x, y, y') es lineal en y' y el
problema variacional no siempre tiene solucin (vase el 4). Por
eso, se toma slo la solucin ingular de la ecuacin de Clairaut que
ser precisamente la funcin buscada F (x, y, z).
EJEMPLO 6. La ecuacin de Hamilton-J acobi en el problemji
.:X-1
sobre el extremo de la l [y (x)l = } F (x, y, $1') 4x Hem1
:114
152
CAP. 11 , E X TREMO DI! FUNCIONALES
la forma
(
oW)2 (W)2
OX + 7i = xz+ y2.
Hallar la Funcin F (x, y, y').
SOLtJCION. la ecuacin dada respecto a la derivada
oW
ax , tenemos
aw . / ( aw )2
-=V x2 + 1J2- ay .
o sea,
Por consiguiente, el ha miltoniano es
H = -yx2+y2-p2.
La ecuaciou (7) para la determinacin de la unci6n F tiene la
forma
dF V ( dF )2
F - z-= x2+ y2- _
dz di
(8)
Deri vando respedo a z ambos miembros de la ecuacin (8), resulta
dZF
Dejando a un !arlo el caso dzZ =O (que da la solucin general, tene-
mos
dF
dz
z = -,---:: /:=====d==F::=2;::-
V :cz+uz-( t1Z)
Resolviendo esta relacin respecto a la derivada ,
dF
-z - v1+t
encontramos
(9)
12 . TEORA DE HMAILTQN .. J ACOBI
153
Introduciendo (9) en (8), obtenemos
F=z z Vii'+Y2 + .. / x+ 2_ z
2

v x+ yi y1 +zz.
"JI+zz Y Y l+zi
Por consiguiente, la funcin Integrando buscado tiene la forma
F="Jx2+!12 Vl + y'z.
En los problemas que siguen hallar las funcionales a parti r
de sus ecuaciones de Hamiltoo-J acobi:
209.
210.
211.
4
ow ow _ ..2.,,2
iJx og - .-!:f
aw ( aw )2
4xy ax+ <f =0.
(
aw )2 ( aw )2 2
X;- + y iJy =X +!f.
so. Principios variaclonales de la Mecnica.
a) PRINCI PIO oe HAMIL TON-OSTROORADSI<J. Supongamos que se
ti ene un sistema den puntos material es M11. (x
1
P Yk :z11.} {k = 1, 2, ... , n)
con masas respectivas mk (k = 1, 2, . . . , n). Supongamos que el
movimiento del sistema est sometido a enl aces
'PJ (x, y, z, t) = O {/ = 1, 2, . .. , m; m n) (10)
y se realza bajo la accin de.las fuerzas P-,., (X1., Yk, Z
11
) (k = 1, 2, ... , n)
que tienen el potencial (funcin de fuerza} U = U (xA, Y1u z11. . t) :
au au au
Xh=;;-, Y11. =:;--, ..
vX v1jR, V'k
La energa cintica de este ser igual a
"
1 ""' a a a
T=2 L.J m11 (x1t+Y.11+.t1t).
Jt ... J
Supongamos que este sistema pasa de cierto estado A correspondiente
al momento del tiempo t = t
0
a otro estado B correspondiente al
momento de tiempo t = t
1
Entre todos los desplazamientos posibles
del sistema de A a B se escoje la clase de movimientos admsibles que
concuerdan con los enlaces dados y que hacen pasar el siste.ma del
estado A al estado B en el intervalo de tiempo dado ( 1
0
, 1
1
).
El principio de Hamilton-Ostrogradski consiste en lo siguiente:
entre todos los moulmiento.s admislbles que hacen JJasar el sistema
154
CAP- JI. E XTREMO DE FUNCIONALES
del estado A al estado B, el movimienio real se caracteriza por el
cumplimiento de la Cflfldicidrt necesaria (JJ =0 de ex/remo de la
funcional
t1
J = J (T-i-U)dt .
to
(11)
A cada moYimiento admisi ble del sistema le corresponden 3n
funciones X1t (1), 1111: (t), :z
11
(I} (k = l , 2, . . , n) que estn definidas
en el intervalo [1
0
, /
1
], que sat isacen las ecuaciones (10) y que toman
determinados val ores en los extremos del Intervalo [t
0
, t
1
J. Por consi-
guiente, tenemos un problema Yariacional con los enlaces (1 0} y con
fronteras fij as.
Para resolver este problema formamos la funcin auxiliar de
Lagrange
m
F= T + U + 2} '-1 (t) 'PJ
i=-1
y escribimos para cita el sistema de ecuaciones de Euler- Ostrogradski:
m Ucpj
m1iY11. - Y1t- X1(t) 0!11i = 0,
i - 1
m 'PJ
m11,z11,-Z11.- ozh = 0.
j=1
(12)
El ss tema ( 12) conklde con las ecuaciones diferenciales del movlmlen
to real del sistema.
b) PRINCIPIO DE 1.A ACCIN BN 1.A FORMA DI! LAGRANOB.
Supongamos que los enlaces q> y el potencial U no dependen del
tiempo t. En este caso tiene' lugar la integral de energ a T - U =
= h = const. La integral
11
J =} Tdt
'
se !C<;(<'!p. De la integral (11) se deduce
IJ h 11
J Tdt- J hdt.
t9 IQ
l :Z . TEOR fA DE HAMI L TONJ ACOBl
15.5
El principio de la accin minima en la forma de Lagrange consiste
en lo siguiente: para el movimiento real, la integral de la accin debe
tomar su valor min.imo, o sea,
lt
I= J Tdt=min.
to
El principio de la accin mlnima puede ser representado en la
forma de Jacobl
J V2(U+li) ds = mn
y
(ds es la diferencial del arco y) en la que no interviene el tiempo.
OBSERVACIN t . Aqui se consideran admisibles los movimientos
que satisfacen las ecuaciones de enlace IJ>J (x, y, z) = O (J = 1, 2, , m}
y la ecuacin T - U= h con el mismo valor de h que para el movi-
miento real y que tienen los estados inicial y final ij os, siendo tam-
bin fijo el momento Inicial /
0
del tiempo. El momento final del tiempo
no se fija para estos movimientos.
OBSERVACIN 2 . La energa potencial figura no en Ja Integral sino
en la condicin complementaria T - U= h. Formamos la funcin
auxiliar de Lagrange
m
l l "1
F = 2 T+2 (U+h) + Ll ),,JIJ>J
;-1
Despus escribimos las ecuaciones de Euler-Ostrogradski para nuestro
problema
m
<JU oq>
m11,X11, 'A.J
VX]l. LJ v:CJt
;- 1
m
au iJ<p1
mhYk=-::i - +2 "1 A.1 ;;-- ,
vflft Ll v/lf1.
;-1
" iJU m ocp
= OZ1t +2 A.1 ozk
;-1
que representan las ecuaciones del movimiento real.
EJEMPLO 1 . Basndose en el principio de la accin mlnima, hallar
la trayectoria del punto material (de masa unitaria) que se mueve
por accin de la gravedad.
sOLUClN. Tomando eje Oy haci a arri ba, el potencial de la fuer
ia de gravedad es
(l3)
156 CAP, 11 . EXTREMO DE FUNCIONAt.l!S
Segn el principio de la accin mnima, para la trayectoria bus
cada 'Yo la integral
J .... s (14)
y
debe alcanzar su valor mnimo. Por consiguiente, la trayectoria ser
una extrema! de la funcional (14). Introduciendo (13) en (14), obtene
mos
i.:1
J= J V2<h-eY>Vt+y'2 dx. (15)
La ecuacin de Hamilton-Jacobi tiene la forma
ow ,, - ( ow )2
ax - V 2h-2gy - au =
o sea,
( 2+ ( t =2 (h-gy).
Su integra 1 completa es
3
r 1 -
W =Ax+ J V2h-2gy-A2 dy=Ax- ag { 2h - 2gy-A2)
2
+ B,
donde A y B son constantes arbitrarias.
Determinamos las exlremales de la funcional (15) :
t
A 2
x + -g(2h - 2gy- A2) =C,
o sea,
!1= - :; -


(x-C)2; A y C constantes.
En particular, las extremales que pasan por el oriien de coorde
nadas se determinan de la condicin y {O) =0. Obtenemos una fami
lis monoparamlrica de parbolas
g 2
Y= - 2A2 X + A x.
212. Hallar en el plano ta trayectoria de un punto que
se mueve por efecto de una fuerza repulsiva que acta desde
el eje Ox en direccin del eje Oy y que es proporcional a Ja
distancia del punto al eje Ox aceptando que la integral de
la fuerza viva tiene la forma - = O y basndose en
12. TEORtA DE HAMI l. TON JACOBI
i57
la integral de la accin
X2
J [y (x)] = J y Vl + y'
2
dx (y> O).
213. Un punto material describe la circunferencia p =
= 2R cos q> (P. cp son las coordnadas polares) de radio R
bajo la accin de una fuerza central :
6
inversamente propor-
cional a la quinta potencia de la distancia al centro que se
encuentra en el origen de coordenadas. Demostrar que la
integral de la accin alcanza minimo fuerte en cualquier
arco de esta circunferencia ( - < <p
1
tp (f>\l < ) .
214. Analizar el movimiento de un punto material por
efecto de una fuerza central de atraccin proporcional a la
dis tancia al centro O basndose en el principio de la accn
mnima y aplicando el mtodo de 1-familton-J acobi.
Captulo i 11
MTODOS DIRECTOS EN EL CLCULO V ARIACIONAL
13. Mtodo de diferencias finitas de Euler
Consideremos el problema variacional elemental: hallar el ext re-
mo de la funcional
b
J[y(x)] = 1F(x,g,g')dx; y(a) = A, y(b) = B. (1)
a
Segn el mtodo de Euler, los valores de la funcional (l) se toman
no e.n las curvas arbitrarias que admite este problema varacional,
sino en las quebradas compuestas por un nmero dado n de segmentos
rectllneos cuyos vrt ices tienen abscisas fijas
a+x, a+2x, .. . , a+(n- l )x,
b-a
donde Ax=--.
n
En estas quebradas la funcional J [y (x)} se convierte en una funcin
<D (Yi. !12 . ., Yo-
1
) de las ordenadas YI> y,, . .. , Yn-1 de los vrtices
de la quebrada. Las ordenadas y
1
, y, . .. , Yn- i se escogen de modo
que la fu ncin I> (!/i. y
2
, . , Yn-i) tenga extremo, o sea, se determi-
nan del sistema de ecuaciones
oI>
iJy2 =0, ... ,
~ o
Yn-t - -
La quebrada as obtenida es la solucin aproximada del problema
variacional (!).
EJEMPLO. Hallar l a solucin aproximada del problema sobre el
mlnimo de la funcional
1
J(y(x)l= J (y
1
2+2y)dx; y(O)=y(l)=O.
o
1-0
SOLUCIN. Toml'mos Ax= -
5
- = 0,2 y pongamos
!lo = y {O) = O,
Ya = Y (0,6),
Y1 = !1 (0,2),
!14 = y (0,8),
y ~ = y (0, 4),
!15 = g ( 1) = o.
13. 01$ DIFERENCIAS FINITAS
159
Sustituimos los valores de la dervada segn l a frmula aproximada
!J
, _y' (x ) ,..., Y1t+t - Yk
lt -- 11 ,.., l\x
Entonces
y' (O)- Yt-0 g' (O 2)- !h.-Yl
-----ay-, ' - 0,2
y' (O oi)- Y
4
- !1
3
y' (O 8) - O-Y+.
' . - 0,2 ' . ' - 0,2 .
Sustituimos la integral por una suma empleando la [rmula de los
rectngul os:

J f (x) dx [f (a)+ f (x1) + f (x2} + ... + f (Xn.-tH l\x.
(l
Tendremos
<D (Ytd'2, !13.!/i.) = [ ( y+ ( Yl )2 + 2Yt +
+ ( !!
3
;;!1
2
)
2
+2u2 + (
11
;;!1
3
)
2
+2us+ ( - )
2
+2y,J 0,2.
Formamos el sistema de ecuacones para determinar las ordenndas
y,. y,. Ya e y, de los "'rtices de Ja quebrada buscada:
o sea,
0:2 = - - +
2
=
0

_l_ o<D = !12-Yt y3 -!J2 +
2
=
0
,
0,2 oy
2
0,02 0,02
_ 1_ o<D = Ya-Y2 +
2
=
0
,
0,2 i'Jy3 0,02 0,02
1 i'JCD !J4-Ys !14
0,2 iJy4. = 0,02 + 0,02 +
2
=
2!11 -!/2 =-0,04,}
-1J1+2!12-!!s= -0,04,
-u2+2Y3 -!J4= -0,04,
-ua+2y, = -0,04.
La soluci n de este sistema es y
1
= -0,08, y
2
= -0, 12, y
3
= -0,12
e y
4
= -0,08. Estos valores de la solucin aproximada coinciden
con los valores que t iene en los puntos respectivos la solucin exacta
x2-x
y= -2-
160
GAP. 111. MTODOS OIRl!CTOS
Hall ar las soluciones aproximadas de los problemas sobre
el mnimo de las funcionales:
1
21 5. J{y (x)}=) (y'
2
+y
2
+2xy)dx; y ()= y( l )=.
o
SUGERENCIA. Tomar X=,2.
1
216 .1 [y (x)] = J (y'
2
+ l) dx;
o
a) y (O) = O,
b) y (O) = O,
y (1) =O;
y(l)= 1.
14. Mtodo de ftz. Mtodo de Kantorvich
10. Mtodo de Rlb. La idea del mtodo consiste en que al hallar
el extremo de la funcional J [y (x)) se consideran, en lugar del espacio
de l as funciones admisibles, slo fas funciones que se pueden represen-
tar como combi naciones lnealcs de l as funciones admisibles:
n
Yn (x)= 2j a11P1 (x), ( 1)
}col
donde IX son unas constantes y el sistema {q> (x) }. llamado sistema
de funciones coordenadas, est formado por funciones cp
1
(x) que son
linealmente independientes y que constituyen un sis tema completo
de funciones en el espacio considerado.
Hablando en trminos generales, cuando pedi mos qut:' las (un
ciones Yn (x) sean admisibles, Imponemos a las funciones coordenadas
cp
1
(x) ciertas condiciones complementarias como, por ejemplo, limita
ciones en cuanto a la derivabilidad o en cuanto a la verificacin de
las condiciones de frontera.
En estas combinaciones li neales la funcional J fy (x)) se convierte
en una funcin de los argumentos a
1
, as, ... , an:
l IYn (x)I = <D (et., ct2, ., ctn)
Determinamos los valores a
1
, a
2
, , an que ofrecen extremo a la
funcin <D (1X
1
, ct
2
, . , 1Xn); para ello resolvemos el sistema de ecua-
ciones
()<D
-a =0 (i= 1, 2, ... , n),
a
no lineales, como regl a, respecto a IX
1
, a
2
, , ct
1
u e introducimos
en (1) Jos valores encontrados para IX. La suct>sin {Yn (x)} que asl
resulta P.S una sucesin mi nimlzante, o sea, la sucesin de los valores
de la luncional {J (y
11
{x)I } obtenida a parti r de ella converge hacia
J 14 . MaTODO DE RITZ. M!TODO DE KANTOROVICl t
tfH
el mnimo o hacia la cota inferior de la funcional J fy (x)}. Sin embargo,
de
lim J IYn (X}]= min J 111 (x)J
n ... oo
no se deduce an que lim Yn (x) = y (x). La sucesin rninimizante
n-+oo
puede no converger hacia la funcin que realiza el extremo en la
clase de las funciones admisibles.
Se pueden indicar las condiciones que garanticen que el mnimo
absoluto de Ja funcional exista y se alcance en las funciones {l/n (x) }.
En el caso en el que se trata del extremo de la funcional
:i:s
J (Y (x)! = J F (x, g, y') dx;
Zt
fl (x1)-= Ytt 11 (x2) = U2:
estas condiciones son:
1) la funcin F (x, y, .t) es continua respecto al conjunto de sus
argumentos para cualquier t y para (x, y) E D. donde D es un recinto
cerrado del plano xOy al que pertenecen las lineas Jln (x);
2) existen unas constantes <X > 0, p > 1 y P tales que
F (x, y, z) ;;;;;a 1 z IP + P
cualquiera que sea z y para cualquier punto (x, y) E D;
. 3) la funcin F (x, y, %) tiene la derivada parcial conlinua
Fz (x, y, z) y esta derivada es una funcin no decreciente de
z (- oo < z < +oo) cualquiera que sea el punto (x, y) E D.
En particular, las condiciones enunciadas se cumplen para las
funcionales
J (y (x)J= r (p (x) y'z+q (x} y2-t-2r (x) yl dx;

if.(x
1
) =a, y (xa) = b;
donde p (x), q (x) y r (x) son funciones dadas, continuas en lxt, x
1
),
con la particularidad de que existe la derivada continua p' x) de
p (x) y de que p (x) >O y q (.x) O.
Si por este mtodo se determina el extremo absoluto de la fundo
nal, el valor aproximado de su mfnlmo se obtiene por exceso y el
valor aproximado de su mximo, por defecto. Al aplicar este mtodo,
el xHo depende en gran medida de la eleccin adecuada del sistema
{cp
1
(x)} de funciones coordenadas.
En muchos casos basta tomar l a combinacin lineal de dos o tres
funciones q>
1
(x) para obtener una aproximacin bastante satisfactoria
de la solucln exacta.
Si hay que determinar el extremo aproximado de la funciona)
J (z (xi. x,, ... , xn)) que dependen de las funciones de varias varia
H-01381
t62
CAP. 111. MeTODOS DIRECTOS
bles independientes, se escoge un sistema de funciones coordenadas
IJ>1 (xi. X2, , Xn),
y la solucin aproximada del problema variacional se busca en la
forma
m
Zm (X1' X2
1
, Xn) = akQ>lt. (X1t Xz, .. , Xn).
k ... 1
donde los coeficientes ak son unos nmeros constantes. Para determi-
narlos se forma, por analoga con lo que hemos explicado, el sistema
de ecuaciones !> = O (k = 1, 2, .. ., n), donde Cll (ai. et
11
, , a n)
vcik
es el resultado de introducir Zm en la funcional J (z (x
1
, x
11
, ,
EJEMPLO t. Hallar la solucin aproximada del problema sobre
el mfnmo de la funci onal
t
J [y (x}} = J (y'2 - y1. + 2xy) dx; (2)
o
y (O) = y (l) = O;
y compararla con la solucin exacta.
SOLUCIN. Como sistema de funciones coordenadas q>k (x) tomamos
cpk (x) = (1 - x) xll. (k = l, 2, ... ).
Es. evidente que las funciones 'Pk (x) satisfacen las condiciones de
frontera 'Pk (O) = q>,1 (1) = O, son linealmente independientes y for
man un sistema completo en el espacio C
1
IO, 1).
Para k = l tenemos !h (x) = et (x - x'). Introduciendo esta
expresin de y
1
(x) en la funcional (2), obtenemos
i
J IVt (x)J = J (af {1-2x)2+af (x-x2)2+2a
1
x (x-x2)J dx=
o
1
= J


o
3 1
=10 a.f + 6 ci1 = <D (at).
E1 coeHciente at se determina de Ja ecuacon
Mi 3 l
t =s
1
+6=
0

14. .METODO DE RI TZ. MBTODO oe KANTOROVICH
163
5
de donde result" a
1
= -
18
. Por consiguiente,
5 5
y, (x) = --x+-xz
18 18 .
SOLUCIN EXACTA. La ecuacin de Euler de la funcional considl'
rada es
y-+ y= x.
Resolviendo esta ecuacin lineal no homognea, encontramos
y = C
1
cos x + Ca sen x + x.
Empleando las condiciones de frontera y (O) = y (1) = O, obtenemos
definitivamente
sen x
y=x-seT.
Comparemos las soluciones exacta y aproximada:
IC Solucl6n exada Solucin aproxln1ada
0,25
0,50
0,75
1,00
o
- 0,044
-0,070
-0,060
o
o
-0.052
-0,069
-0,052
o
EJEMPLO 2. Hallar la solucin aproximada de la ecuacin no
lineal
3
y=-z y2
que satisfaga 1 as condiciones y (0) = 4, y (1) = l.
SOLVC.10N. A este problema de contorno le corresponde el proble
ma variacional
1
J lu(x)J= J (y'2+y3)dx; g(0)=4, y(l)= I.
o
Buscaremos la solucin en ta forma
Y1 (x) = 4 - 3x + a
1
(x - x
2
);
es evidente que y
1
(x) satisface las condiciones de frontera dadas cual
quiera que sea el valor de
Tenernos
'
J [yt(x))= J (l at(I -2x)-3}2+ [4-3x + a
1
(x-xZ)]} dx,
{)
164
CAP. Ill . Merooos DIRECTOS
de donde
1
oJ(y(x)} = ~ {(l-2x)2[a.l(l--2x)-3J+
.
o
+3 (x-x2) (4- 3.x+a.
1
(x-x2)l2) dx.
La condicin i)J [Yi (x)) =0 toma la forma
iJa1
9 e t ~ + 490a.1 + 1407 = o
y para a..
1
= -3,0413 obtenc.mos la solucin del problema
Yi (x) = 3,0413x' - 6,0413x + 4
posi tiva en lodos los puntos.
Hallar las soluciones aproximadas y compararlas con las
exactas en Jos problemas que siguen sobre el mnimo de las
funci onales:
1
217. J[y(x)l = J (y'
2
+ 2y)dx; y(O)=y(l)=O.
o
2
218. J[y(x)] = J (2xy+y
2
+ y'
2
) dx; y(O)=y(2)=0.
o
219. Hallar la solucin aproximada del problema sobre
el mnimo de la funcional
1
J (y(x)}= J (y'
2
- k
2
if)dx; y(-l)=y(l)=O;
-1
1
con Ja condicin complementaria J if' dx = 1.
-t
EJEMPLO 3. Hallar la solucin aproxi mada del probll'ma sobre
el extremo de la !uncional
r r - ( az ) 2 az ) 2 J
J [z (x, g)J= J J l ax + ( " - 2z dx dy.
D
donde D es el cuadrado - a x ~ a, -a y ~ a. siendo z = O
en la frontera del cuadrado.
SOLUCIN. Buscamos la solucin aproximada en la for ma
z
0
(x, y) = ct
0
(xi - ex) (!/' - a).
' l 4.
METOOO DE RITZ. DE KANTOROVICH
165
Es evidente que est a funcin z
0
(x, y) satisface las condiciones de
frontera planteadas. Introduciendo 2
0
(x, y), Zx (x, y) Y Zy (x, y) en
Ja funcional e integrando. obtenemos
256 32
J (zo (x, y)) = 45 aoa
8
= (J) (ao).
Tenemos despus
o<D s12 32

- 45 9 - '
5 5
de donde cto= l6aZ de modo que

y) =
1642
r_x2 - a2) (y2-a2).
220. Hallar la solucin aproximada del problema sobre
el extremo <te la funcional
J (z (x, y)J = j J [ ( -y)
2
+ ( + x)
2
j dx dy,
D
donde D es el recinto limitado por la elipse -+ t: = l.
221. Hallar la solucin aproximada z
3
(x, y) del proble-
ma sobre el mnimo de la funcional
J [z (x, y) J = J J [ ( :: )
2
+ (
2
J dx dy,
D
donde D es el recinto: x > O, !J > O y x + y < 1, si la fun-
cin z (x, y) satisface en la frontera f: x = O, y = O y x +
+y = 1 la condicin z Ir= x
2
+y?..
20. Mtodo de Kantorovlch. Este mtodo ocupa una posicin
intermedia entre la resolucin exacta y el mtodo de R ilz y se aplica
para analizar el extremo de las funcionales
J (z (x
1
, x
1
, ., Xn)] (3)
que dependen de funciones de varias variables independientes (11 2) .
Igual que en el mtodo de R itz, escogemos un sistema {<pk (x
1
, x
2
, .. , Xn)
de (unciones coordenadas y buscamos la solucin aproximada en la
forma
(4)
pero considrrando los coeficientes (xJ) como funciones incgnitas
de una de las vari ables independientes.
166
CAP. 111. M2TOOOS DIRECTOS
En las funciones (4) Ja funcional (3) se convierte en una funcional
J[a.i (x
1
), a
11
(x1), .. , ~ (x
1
)] que depende de m funciones
a:
1
(x), a
1
(x
1
), ... , cim (x
1
). Estas funciones se escogen de modo
que la funcional l alcance el extremo y se determinan de las condicio
nes necesarias de extremo para la funcional 7.
Empleando el mtodo de Kantorovich, se obtiene una solucin
aproximada, como regla, ms exacta que la solucin que da el mtodo
de ~ itz con las mismas funciones coordenadas 'Pk (x
1
, x
1
, , xn)
y con eJ mismo nmero m de trminos en la aproximacin.
EJEMPLO 4. Hallar la solucin aproximada de la ecuacin de
Poisson
A t 1 t. 1 D { -a< x <a,
.,.z = - en e rec angu o : -b <;y<; b
si z = O en la frontera.
SOL.UCiN. La ecuacin Az = -1 es l a ecuacin de Euler-Oslro
gradski para la funcional
J[z(x, y}J = J J [( : }
2
+ ( :; )
2
-2z] dxdy. (5)
D
Buscamos la solucin en la forma
z
1
(x, y) = (b' - y
1
) a (x);
es evidente que la funcin z
1
(..t, y) satisface las condiciones de frontera
z = O en las rectas y = +b.
lntroducierido esta expresin de z
1
en la funcional (5), encontramos
a
J [zt(x y)]= J ~ b5cz.'2+! b3a,2 .! ba.) dx.
' 15 3 3
(6)
- o
La ecuacin de Eu ler para la r uncional (6) es
" 5 5
- 2b2. = - 4b2
(7)
La ecuacin (7) es una ecuacin lineal no homognea con coeficientes
constantes y su solucin general es
Las constantes C
1
y Ci se determinan de las condiciones de frontera
a (-a)-a (a) =0
s 16. LOS VALORES Y LAS FUNCIONES PROPIOS
lo queda C2 =0yC1= - V de modo que
5 a
2ch --
2 b
l
, /S X l
1 ch V 2"b
a(x)- - 1- .
2 , /5 aj
chy '21'
Es decir, obtenemos
1
,/sx l
b2-yz ch V 2b"
t 1 (x y) = 2 1- V 5 a
ch --
2 b
167
Para obtener una aproximacin ms exacta se puede buscar la solucin
del problema en la forma
i
2
(x, y) = (b' - y) a
1
(x) + (b' - y') a, (.r).
222. Hallar en el recinto D la solucin aproximada de
la ecuacin de Poisson 6z = -1 que se anule en su frontera
si D es el tringulo equiltero formado por las rectas y =
= ~ a X y X= b.
223. Hall ar en el recinto D la solucin aproximada de
la ecuacin ~ z = - 1 que se anule en su frontera si D es el
trapecio issceles formado por las rectas y = ~

x, x =
y X= 3.
15. Mtodo,, variacionales para la determ;nacin
de los valores y de las f un clones propios
La ecuacin de Sturm-Liouvill e
d
- dx (p (x) y' J +q (x) g = Ay, l l l
donde p (x) > O ti ene derivada continua y q (x) es continua, con las
condiciones
y (a) = O e y (b) = O (2)
tiene la solucin nula (trivial) !I s O cual qulera que sea el valor rea
o compl ejo de A.
168
CAP. ll l. MeTODOS DI E T O S
El conjunto de la ecuacin (1) y de las condiciones de frontera (2)
se denomina problema d coniorno /U Sturm- Liouoil/e (1)-(2).
Los va lores de>.. para los cuales el problema de contorno (1)-(2)
tiene soluciones no triviales y == O se denominan valores propios
y las soluciones mismas llevan el nombre de /unciones propia; del
probletn11 de contorno.
La ecuacin (1) es l a ecuacin de Euler para el slfulente problema
sobre extremo condicionado: hal lar el mlnlmo de a funcional
I>
J lu (x)J = J (py'2+ qg2.) dx
a
con las condiciones (2) y la condicin
b
J y2 dx= l.
()
(3)
(4)
Si y = y (x) es una solucin de este problema variacional , tambin
ser una solucin del problema (1 )-(2) distinta del cero Idntico en
vi rtud de la condicin (4). Por eso, los valores propios y las funciones
propias del problema de contorno de Slurm-Liouville se denominan
tambin valores propios y runciones propias de la funcional (3) con 1 as
condiciones (2) y (4).
La funcin propia y = !I (x) se denomina normada si
EJEMPLO 1. Hallar los valores propios y las funciones propiae
de la funcional
3
J [y (x)J = J ((2x + 3)2 y'2- y2 dx
o
con las condlcones
y (0) = O, y (3) = O,
:
J g2dx= l.
o
SOLUCION. La ecuacin de Sturm-Liouville tiene la forma
-y- :x ((2x+3)2 y'] =>..y,
o sea,
(5)
(2x + 3)
1
y' + 4 (2x + 3) y' + (A+ 1) N = O. (6)
su. LOS VALORES Y LAS PUNCIONES PROPIOS 169
Mediante la sustitucin 2x + 3 = ti la ecuacin (6) se reduce a la
ecuacin lineal con coeficientes constantes
d2y dy
_. dt'- +4dT+<:i..+nu=O.
(7)
Su ecuacin caracterlst ica
tiene Jas ralees
4k' + 4k + ),, + 1 = o
t t ,r-
kt,2=-22v -A..
Consideremos tres casos.
l} A. < O. Entonces la solucin general de la ecuacin (7) es
y = C1e
11
'
1
+ C
1
e
112
t,
donde ki y k
1
son nmeros reales; en consecuencia, la solucin general
de la ecuacin (6) es
!I == C1 (2x + 3)
11
' + Cs (2x + 3)
111

las condiciones de frontera (5) dan
Ct3"'+C23
111
= 0,}


de donde C
1
= O, C
2
- O e y s O.
2) A. = O. Entonces
y, por consiguiente,
De las condicione de frontera obtenemos
C1+C21n3=0,
C1 +C
2
ln 9 =0,
de donde C
1
=0 y C2 = 0, o sea, ysO.
1 vx
3) A.> O. Entonces k
1
,
2
= -
2
i -
2
- y la solucin general
de la ecuaci.n (7) es
y=e-f ( c,cos t +C2sen t)
170
C.l\P. 111 . M8TODOS DI RECTOS
Pasando a la variable x, obtenemos
ln(2x+3)J +C
2
sen[
Y v 2x+a
Las condiciones de frontera t5> dan
c,cos ln 3)+c2sC?n (

tn3) =0, )
v VX } <
9
>
C
1
cos (-
2
- ln 9) +C
2
sen (-
2
- In 9} =0. J
El sistema (9) tendr soluciones no tri viales s i su determinante es
igual a cero
v- v-
eos (-f In 3) sen ( T In 3)
=0;
-v-
eos (-i- Jn9} sen(-+ ln9)
por lo tanto, sen In 3- In 3 ) =O, es decir, S<.'n X
V -VI
X (-
2
- In 3) =- 0, de donde -
2
- In 3= ruc, Los valores propios sern
4n2n2
An '"" 'i''23 (n= 1, 2, .. ).
Tomando cualquier ecuacin del sistema (9), por ejemplo, la pri mera,
e introduciendo en ella An en lugar de A., obtenemos
C
1
cos nn + C
1
sen nn = O,
o sea, C
1
( - t}n = O. de donde C
1
= O. Tomando en (8) C
1
= O y
41i2n2
"-
11
= JnZ
3
, obtenemos las funciones propias del problema conside-
rado
[
AA In (2x+3) J
sen . In 3
(n=I , 2, ... ).
Los coeicientes Cn se determinan de la condi cin de normacin
3
J

(x) dX= 1
o
5 15. LOS VALORES Y LAS PUNCIONES PROPIOS
lo que da
y, por consiguiente,
2
Cn=,r-
v ln3
[
rm In (2.x +a) J
2 &en ln3
Vn(x)=ylii3 Y
2
x+s (n=l,2, ... ).
171
Hallar los valores propios y las funciones propias en los
problemas que siguen:
1
224. J(y(x)l= J (y
2
+y'
2
)dx; y(O)=y(l)=O;
o
2
225. J[y(x)J= J x
2
y'dx; y(l) = y(2)=0;
t
e
226. J {y (x)I = J (6y
2
+ x
2
y'
2
} dx;
1
t
J !f dx= J.
o
2
J !/'dx= 1.
i

y(l) = y(e)=O; j ydx= l.
1
2n
227. J (y (x)J = J (y
2
-y') dx; y (n) =y (2n} = O;
" 2:n
J tfdx= J.
n
i
228. J (y (x)} = J (3y2-(x+ 1)
2
y'
3
) dx;
o
1
y (O) =y ( 1) =O; J y
2
dx = J.
o
172
CAP. 11. MtTODOS DIRECTOS
Los valores prof los y las funciones propias del problema varia
cional (3), (2) y (4 tienen varias propiedades importantes.
1) Si A,,. y A.n son dos valores propios diferentes de la funcional (3)
con las corioiciones (2) y (4) y si llm (x) y !Jn (x) son las funciones pro
plas que les corresponden, estas !unciones !lm (x) e Yn (x) son ortogo
nales, o sea,
b
J !lm {x) !In (x) dx =O (m + n).
Q
2) Todos los valores propios A.n de la funcional (3) son reales.
3) Si An es un valor propio de la funcional (3) e lln (x) es la fun
cl6n propia normada que le cotresponde, se tiene
J l!ln (x)] = "-n
4) El menor de los valores propios coincide con el mlnimo de la
funcional (3) con las condiciones (2) y (4) .
EJEMPLO 2. Demostrar la desigualdad
n
J y'Z (X)
o
n
> J yz (x) dx, y (0) = y (n) = o.
o
n
:>OLUCION Determinemos el mfn J g't. (x) dx con las condiciones
o
lt
J y2 (x) dx= \, y (0) = Y (n) =O.
()
La ~ y ~ 1 o n de Euler para la funcional
tiene la forma
n
J (U (x}J = J (g'2 - f..g2) dx
o
y + 1.y = O; y (O) = O, y (n) = O.
Las funciones propias de este lt imo problema son !In (x) = sen nx
y los valores propios son A.n = n
1

El valor propio mlnlmo es A.i = 1. Por eso, en virtud de la propie
dad 4),
n
mn J y'2 (x)dx= l.
o
!5. LOS VALORES Y LAS F UNCI ONES PROPIOS
En consecuencia, para cualquier funcin y (x) tal que r yi (x) dx = l
o
lenemos
n lt
I y' 2 (x) dx :?- J yZ (x) dx.
o o
sen x
Esta desigua !dad no se puC'de precisar ya que para Y1 (x) = lf
se tiene
J Yi
9
(x) dx = J yf (x) dx= l.
o o
1(
OBSERVACIN. Si J y 2(x)dx=k2 + 1, el problema se reduce al
o
anterior introduciendo la funcin z (x) = y ~
Empleando l a definicin extremal de los valores propios, seale-
mos cmo pueden ser calculados aproximadamente a partir del mtodo
de Ritz. Debe tenerse en cuenta que el mtodo de Rit:i da una aproxi
macn por exceso del valor propio.
EJEMPLO 3. Hallar aproximadamente el primer valor propi o del
problema
y" + 'J..'y =
!J (-1) = y (l) = o.
SOLUCIN. El problema sobre el mlnimo de la funcional
t
con las condiciones
r y'2 dx
-t
1
g(--l)=y(l)=O y J y2dx =I
-1
es un probl ema isoperimtrico y se reduce al problema sobre el mini
mo de la funcional
1
J {y (x)] = J (y'Z - 'J..ZyZ) dx
-1
174
CAP. 111. DIRECTOS
cuya ecuacin de Euler coincide con la ecuacin dHerencial considera
da y"+ "J..y = O, y (-1) = y (1) = O.
La solucin general de la ecuacin es y = C
1
cos 'Ax + Ca sen 'Ax.
De las condci ont>s de frontera encontramos
C1COSA-Czsenf.. = O, }
C 1 cos A.+ C2 sen A. = O,
(10)
de modo que Ja condicin de existencia de una solucin no nula del
sistema (10) es la condicin de que sen2A. =0, o sea, l. =
Por consiquiente, para el primer valor propio tenemos =
= ( )2 y la pr imera armnica de la cuerda viene dada por la solu
.. t nx . "'1 d
c1on exac a g =cos
2
, ' =T; a segun a armomca es y=sennx,

1
t . . 3nx , 3 t
11. =.n; a ercera armonica es y=cos-
2
-, 11.=2n, e c.
A ttulo de comparacin, busquemos las soluciones pares (armni-
cas pares de la cuerda} aproximadas en forma de un pol inomio segn
las potencias de x. Tomando las funciones coordenadas en la forma
q:
11
(x) =x2112-x211 (k= 1, 2, .. . } minimicemos la funcional J en las
m
funciones llm (x) = c.1<p1t (x). Limit ndonos al trmino .11
1
(x) =
11-t
=c
1
rp
1
(x), tendremos J (y
1
( :- ,,2) y para determinar
c
1
obtendremos
iJJ (.!lt (X)] 2 (! _ _!! 2) =0
OC1 -- Ci 3 15 "' .
Puesto que debe ser c
1
=rf:: O, resulta X' = 2,5. Tomando para y
Y2 (x) = c
1
cp
1
(x) + Ci'Pll (x),
encontramos
J [Y2(x)]=cl ( - t.
2
) +2c1c2 {
1
8
5
-


X
2
)+
+cf (
1
:-

1.2),
y para determinar Ct y c
2
obtenemos el sistema
oJ [Y2(x)) _ ,,2) + ,,_2 ) =O '\
Oct -Ci 3 15 Ca 15 105 ' 1
oJ l.!12 {X)) 32 ,,2)-j ( 176 _2!_ A,2) =O J
OC2 Ct 15 105 c
2
I05 ,315 ,
S IS. LOS VALORES Y LAS FUNCIONES PROPIOS
175
La condicin de existencia de soluciones no nulas r
1
y e, de este ltimo
sistema es que su determinante sea igual a cero; esto da ).
4
- 28>..
2
+
+ 63 = O. de donde hJ = 2,46744 y hl = 25,53256. Comparemos los
valores aproximados obtenidos para hf y A.f con sus valores exactos.
El valor exacto de >..? es (
2
2,46740 y el valor exacto de es
22,20661 de modo que el va lor aproximado obtenido para
A.f es de gran exactitud mientras que para el segundo valor propio se
obtiene una aproximacin tosca.
EJEMPLO Hallar el primer valor propio del problema
yw + A. (1 + x') y = O, y (-1) = y (l) = O.
SOLUCtN. Tomemos como funciones coordenadas las funciones
q>Ji (x) = 1 - x
2
1t (k = 1, 2, ... ) que satisfacen, obvamente, las
condiciones de frontera. Poniendo
Y2 (x) = c
1
(1 - x
2
} + e, (1 - x'),
planteemos el probl ema sobre la minimiz.acin de la funcional
1
J (!I (x)) = J [y'2-I. ( \ + xZ) yZ) dx
- 1
que tiene la ecuacin dada como ecuacin de Euler. Tendremos
J [!1'2(x)) =el ( - +2c1c2 (
1
:- 7
2 ( 32
+c
2
T - 3465 "- J
Para determinar c
1
y c'l obtenemos el slstema
iJJ[!12(x)]_
2
(!- 128 '- )..L
2
'\
OC - Ct 3 104
1
c
2
5 45 - , 1
aJ [y
2
(x)J
2
+
2
( 32 _ 5888 ') - o J
OC2 C 5 45 />. C
2
7 3465 "' -
La condicin de existencia de solucin no nula de este ltimo sistema
da
- 1068). + 2079 ,.,,
de donde. lomando la ralz menor. encontramos 1.
1
= 2, l 775.
PRINCIPIO oe RAJLEIGH. Supongamos que se tiene el problema de
valores prop os
L(y) s -:x [p(x) (ll"I
a,y(a) (a)=O, }
a2Y (b) + (b) =O, 0, ( l 2)
donde p (x), p' (x), q (x) y r (x) son continuas en (a, b}; p (x) > O
en [a, bl.
176
CAP 111 . Me TODOS 01 'RECTOS
Diremos que la funcin y (x) es admisible (y E D) si tiene dos
derivadas continuas y satisface las condiciones de frontera (12).
Supongamos que para toda funcin admisible y (x) se cumple
la condicin
b
J yl (y) dx:;;:. O.
u
En este caso el problema'de contorno (l 1) - (12) tiene solamente valo-
res propios reales A..
Podemos poner en correspondencia a este problema de valores
propios el siguiente problema variacional:
entre todas las funciones admisibles y (x) tales que
b
J r (x) y2 d.x >O. {13)
a
b
S yl (y) dx
Q
hallar aque lla para la cual b = min.
s r (X)gZ dx
a
Sea y = IJ>
1
'(x) la solucin de este problema.
Si t..
1
es el vafor mnimo, o sea, si
b b
S gl (y) dx ~ "'tl ('1>
1
) dx
l..1=mn: -,.... b-- -
llED
S r(x) yz dx ! l l ~ d x
a a
entonces f.
1
es el menor valor propio positivo y ~
1
(x) es l a funcin
propia que Je corresponde.
Si a las funciones admisibles se impone, a parte de la condicin
(13), una condicin ms
b
J npydx=O
o
(condicin de ortogonalidad), el problema
b
~ yL(y) dx
-..,...-- - =mio
~ ry2dx
CI
tendr de nuevo una solucin Wa (x).
s lli. LOS VALORES Y LAS FUNCIONES PROPIOS
177
'Si >., es el valor mnimo correspondiente, entonces h., ser e 1
siguiente, en cuanto a la magnitud ().
1
;;;:i: valor propio y 'li1a (x)
ser la funcin propia ortogonal a

(x) que fe corresponde. En gene-
ral, si se conocen ya los k primeros valores propios postivos

y el sistema ortogonal correspondiente de funciones propias
(x), 1J>ii (x), . . . 'l>.1t (x),
el valor propio siguiente. ser igual a
b
S yL(y) dx
1-k+t = mfn - ... b---
l'ED 'i ry2 dx
a
con la particularidad de que se consideran aquellas funciones admi -
sibles y (x) que, a parte de (13), satisfacen las siguientes condiciones
complementariu
b
J r (x)lj>v (x)y(x)dx=O (v=l, 2, .. .,k).
a
Si en la ecuacin (11) se tiene que la funcin r (x) >O en [a, b)
con frecuencia se emplea para estimar por arriba el menor valor
propio positivo /..
1
la $iguiente desigualdad (principio de
b
S yL (y) dx
A -< _R..,b __ _
S ry2 dx
a
EJEMPLO 5. Valindose del principio de Rayleigh, estimar A.i
en el siguiente problema de contorno
-yw = ')..y, y' (0) = O, Y (1) = O.
SOLuciN. En nuestro caso tenemos L (y) = -y, o sea, p (x) e
s 1 > O, q (x) s O y r (x) e 1 > O en [O, 1]. E.s obvio que <Xi = O,


= 1, a,= 1 y =O de modo que af + = l >O y cx.t +
+ = 1 >O. Tomemos como funcin a(imisible y (x) = 1 - x
11
;
seg(tn el principio de tendremos
1 t
yL (y) dx i 2 ( l - x2) 4
o 3
Xt t =-.:....1-----=-8- = 2.5.
S yr2 dx S (l-x2) dx 15
o o
:n2
Recordemos que el valor exacto es A.
1
=--- 2,4674.
12-0t 387
178
CAP. 111 . Marooos DIRECTOS
Estimar el menor valor propio en los problemas que
siguen:
229. -y" =A. (10 - x
2
) y; y (-1) =y (1) =O.
230. -y"= 1.y; y (O) = y ( I) = O.
En el problema de la determinacin de los valores y de las fun
dones propi9s tambin se puede emplear el mtodo de Kantorvich
(mtodo de reduccin a ecuaciones diferenciales ordinarias). Suponga-
mos, por ejemplo, que en un recinto D se tiene la ecuacin
6.z + AZ = 0
y que
Zlr=O,
donde r es la frontera del recinto D.
Busquemos la solucin en la forma
m
Zm (x,, Xz, .... Xn) = c:r.k (x) tpA (x, y) -1-q>o (x, y)
k-1
escogiendo las funciones coordenadas <flli (x, y) y las funciones a.
11
(x),
por ahora incgnitas, de modo que Zin (x, y) se anule en todos los
puntos de r. Las funciones Gt1 (x), a, (X), .... CXm (X) deben satisfa
cer el sistema de ecuaciones
J [Mm+A.zmJ<J111(x,y)dy=O (k=l,2, .. . ,m, (14)
D:t
y deben anularse en los valores extremos del argumento. Aqu D."
es la in lerseccin del recinto D y de la recta x = const.
Aquellos valores de A. para los cuales el sistema (14) tendr solu-
cin no trivial darn una aproximacin de los valores propios y las
soluclnes correspondientes darn una aproximacin de las funciones
propias.
EJEMPLO 6. Hallar aproximadamente el primer valor propio
y la primera funcin propia en el problema
6.z + A.z = O, z 1 r = O
donde el recinto D es el rectngulo: - a x a, -b .:<;;'.y b.
SOLUCIN, Buscamos la solucin del problema en la forma
z
1
(x, } = (y
2
- b) c:t
1
(x) .
La eucaci6n (14) toma en este caso la forma
b
J [2a
1
+
1
y2- b2) a. + , (y2- b2) cx
1
} y2 -b2) dy = o.
-b
15.
o sea,
LOS VALORES Y 1.AS PUNCIONES PROPIOS
..!.!!..bsa+ bS).._.! b)


15 15 3 '
a1 (-a)=at (a) = O.
La soluc:ln general de (15) es
a
1
(x)=C
1
sen V


2
!
2
x.
179
( 15)
Teniendo en cuenta la simetra del problema y tomando una soluci n
particular, obtenemos
C
1
"-0 y C
2
cos V A.-
2
:
2
a=O;
queda claro de aqu que tendremos una solucin no trivial slo si
/ 5 n
V , _ 2b2 a=t
2
k-
1
>2:
(2k-1)2 n2 5
hk = (2a)2 + 2b2
En particular, para l encont ramos
n.2 to
At ,_- (2a2 + 21>2
siendo el va lor exacto
n2 n.2
At = {2a)Z + 2b2 '
El error es menor que 1,3%.
Para la primera funcin propia obtenemos la aproximacin
nx
z
1
(x. v> = (JJ2 - b2) cos 2Q.
Hallar una aproximacin del primer valor propio en los
problemas que siguen.
231. y" + i.
2
y = O, y (O) = y (1) = O.
232. y" + i. (2 + cos x) y = O, y (O) = y (n) = O.
233. Hallar aproximadamente el primer valor propio en
problema
!!J.z + AZ = O z Ir = O,
donde D es el crculo de ra<lio uno con ctnlro m el origen
de coordenadas.
RESPUESTAS E INDICACIONES
J. a) frnsn = O en el punto (0, O); b) fmx = l en el punto (O, O);
e) no hay extremo. 2. No hay extremo. 3. fmfn= -8 en los puntos
(V2. -V2) y (-'V2, V2); en el punto (O. O) no hay extremo.
4. fmtn = O en el punto (0, O); en los puntos de la circunferencia
x2+ y2 = 1 hay mximo no estricto. 5. f mx = V3 en el punto (1, -1).
6. f mln = 4 en el punto (-} , 1, 1 ) . 7. f mfn = -1 en el punto {I, O) .
3V3 ( 2n 2rc) sv3
R. lmio--
8
- en el punto T T ; lmflx =-
8
- en el punto
n2+n+2
( ~ ; ) 9. mflx:= ( n
2
+ ~ +
2

2
para Xt = X2-== ... =X
11
=
= nz+
2
n+
2
. l t. No. 13. Los nmt!'ros ct1 y ~ h deben ser los coefi-
cientes de Fourier de la funcin f (x). 14. f mfn =-+en los puntos
(
l 1) ( 1 1) 1
V
2
, - V2' y - V
2
,
112
; lmb. =2 en los puntos
(
1 1) ( l 1) 36
v
2
. lf2 y - y'i . - v
2
. t5. t msn =13" en el punto
(
18 12 )
13, 13 . 16. f mb.=4 en los puntos (2, 2, 1), (1, 2, 2} y (2, I, 2);
4 (" 4 7) (7 4 4)
fmh.=42 en los puntos 333. 3 3 3 Y
42
4 7 4) T
(3 3 3 17. lmx =t 18. fmtn= I en el punto
( : , ! ) ; f mx = 11 en t punto ( - ~ , - ! ) . 19. f mln = -9
en el punto (- 1 2, -2); f mx =9 en el punto (1 , -- 2, 2). 20.
RESPUESTAS E INDICACIONES
181
fmx = i-enelpunto ( ~ . ~ ~ ) . 21. Jndicacfin. Hallar el
mnimo de la funcin z=+(x" +!I") con la condicin x+y=S.
'
4V5 19112
22. e . 23. -
5
-. 24.
8
. 25. El cuadrado de dimensin
a= R v2. 26. El radio de Ja base del cilindro r = ~ -,/ 2+ Vs
y la altura del cilindro h = R V 2-Vs. 27. Pr imero. 28. Proximi-
dad de cualquier orden. 29. Proximidad de cualquier orden. 30. p = e-1.
2n+3
31. P=I. 32. p=e- 1. 33. P1=e-I. 34. 1>2=
6
. 36. P1001-=e.
36. Continua. 37. Continua. 38. Discontinua ( considerar la sucesin
!In (x) = senn nx } . 39. a) Disconl nua; b) continua. 40. a) Discontinua
1-e2
b) continua. 41. Continua. 45. AJ =-
2
-
48.
a;
-0,I
0,01
49. M
.J 6.J
1,2 1
a;Z
-0,098 -0,I
M =a:+ 5; oJ =a:.
0,01002 0,01
3 (e2- l) +
6
(
3
- ) i + ~ . J - 3 (eZ-1)
4
a e a
5
, u -
4
ex.
0,1
0,01
M
4,7919
0,4792
0,0479
(J.J
6,6821
0,4963
0,0481
50. 1) Diferenciable; 2) diferenciable: 3) diferenclable; 4) no di-
ferenciables. 51. 6J2 [g (x)] =21 (g (x)J aJ.
e
63.
6.J = 3k +--. k2;
e-
3J=3k.
A M M
1 4,582 3
0,1 0,3158 0,3
0,01 0,03016 0,03
182
RESPUESTAS E I NDICACIONF.S
5-4.
5 8 5
llJ--k+- k2
61=3k
- 3 7 '
"
t:.J 6J
2,810 1,667
0,1 0,181 0,167
0,01 0,0168 0,0167
4
55. llJ=Tk?.; 6J =O.
lt 6J AJ
-1 o f,3333
0,3 o 0,1200
0,03 o 0,0012
b b
57. 61 = J 58. 6J = 2 J (yy-y'y' )dx.
a
1
69. 6J = 2y tO) 6y {O) + ) (x 6y') dx.
o
n
80. 6J = J (J/ cos y {Jy +sen 6y') cU.
o
I>
r ( ar at ' . )
61. 61 = l -
0
- yt +-
0
- 6112 + . . . +r yn dx.
Yt 112 !111
a
62. fiZJ (y, y)= 2J (y, y).
63. 6teF<11>=eF(11) {(6F)2 + 62FJ.
I> m
J
iJ2f
SS. (JZJ = L.J 6y<1t>fiycll dx.
a
11
111i ay<i>
a .Ir, t - 0
68. f>ZJ= f ... + F .. ,, (6z)2jdxdy.
J J Uic Zr,%71
G
I> m n
67. 6"'J= J [ F;
111
k6y18t111+ F;
111
.:6y16!1i +
a i, 111 i , .11-1
TI
RESPUESTAS E INDICACIONES
183
68. Considerar 11\ funcional
J [cp+a:r1J .:mil) (a)
y emplear la segunda definicin de la variacin. Exigiendo que
6J =0, llegamos a la ecuacin integral
b
J K (s, t) cp (s) ds+cp (t)-f (t) =0.
a
69. Procediendo de la misma forma que en el problema anterior.
encontramos que la !'cuacin funcional de Euler , que expresa la anula-
cin dt> la primera vnriacin, viene dada por
- ci (x+2)- q> (x-2) +<J> (x)+ f (x) = O.
Esta ltima es una ecuacin mixta con derivadas y diferencias.
70. -(pq:')'+q<p :=::: /(x). 71. y= -x3, 72. y= .73.Dos
extremales
1+<3 + 2-V2> <2x-1>2
y= 4 <lf2 1>
74. Dos extremales y=f"<x+l )Z e u=l"'l3x-1)
2
75. tt=<C+
l
+ x) sen x, donde C es una constante- arbitraria. 76. y =
2
X
7 1 13
x [e-x +O + e) lJ. 77. y=
6
x-
6
x
3
78. y=T x-
i
- tf x3+ 2. 79. y= In x. 81. La integral no depende dd camino
de Integracin; el problema variacional carece de sentido. 82. y=O
si a=O; siendo a =fr. O, no existe extremu l suave. 83. y=cosx.
84. y=cosx+Csenx. donde Ces una constante arbitraria. 86. 11=
sh X
=x+I. 86. Y=ShT 87. y=e2tt-X>. 88. No hay extrema!es; la
ecuacin de Eu ler no
90. No hay exlrcmales.
t iene soluciones. 89. y =C
1
-t-C2x- .
1
93. y=C
1
e:-: +c
2
e-x - yxex:. 94. y=2chx.
Yt sen x . 1
96. y= . 97. y=2x. 98. La clrcunferencia -=K. 99. y=
seo xi '
x3
=(1-x)shx. !OO. u=
6
(x3+6x+I). IOJ. No hay extremo. 102.
184
RESPUESTAS E INDICACIONES
El problema variacional carece de sentido porque bajo el signo de la
integral figura una diferencial exacta. 103. y=sh x.
{
a = sen 2x,
104. 11 =+x2. 105. x2 32+n2
Z= -2+ 8n X.
{
Y= -{-cx3+Sx-6l.
J06. 107. { u=sen .x,
z = x. z =sen x.
tos.
{
x2
Y=-y+I,
Z = l.
110.
(
ilz )2 iJ2z ( iJz )2 01-z
'"Fi '(jX2 + ay iJy2 = f (X, y).
= f (X1, X2, ., Xn)
113. Solucin. El planteamiento del problema es el siguiente.
las superficies z = <p (X, g) que se proyectan en el reciQ.to D del
ptano xOy y que pasan por cierta curva cerrada alabeada cuya pro-
yeceln es la curva frontera r del recinto D.'!ha llar la superficie cuyo
1ea
S == j l V 1 + cp! + cp; dx dy
!)
sea mfnlmo (problema de Plateau). La ecuacin dHerencial de Euler
para este problema es
a q>$ +..!.. 'Pv =O
-;- YI lft+qi!+IP:
o en forma desarrollada
cp= (1 + cp;) - 2q>;rvq>$<p + 'Pvv ( 1 + cp!) =O.
Esta es la ecuacin diferencial buscada de las superficies de rea
mnimo. La real izacin Hslca de la superficie de rea mlnima se puede
obtener, por ejemplo, con una pelicula de jabn tendida sobre un lato
de alambre.
114. z (x, 11) = !I El problema tiene solucin nica aunque las
de frontera no se dan en toda la frontera.
RESPUESTAS E l NOl Cl\CIONE$
115. r cos qi+Ca = C
1
In 1 r sen cp + y ,z seni q-cn
117. xi cos C2- y
2
cos Cz-2xy sen C
2
= Ct
185
lt8. Campo central. 119. a) Campo propio; b) campo cent ra l;
c) no forma campo. 120. Campo propio. 121. a) Campo central; b) no
forma campo; c) campo propio. 122. a) Campo central; b) campo pro-
pio; c} no forma campo. 123. No lorma campo porque esta familia
de curvas no cubre lodo el recinto D. 124. y =C
1
ch x forman un campo
propio de extrema y = C
2
sh x forman un campo centra 1 de ex lre-
males. 125. y=Ccos.v: forman un campo propio de exlremales; y=
= C sen x forman un campo central de extrema les. 126. La extrema l
IJ = (l-x2) puede ser incluida en el campo central de exlremales
x3
y ..c C
1
x-
6
con centro en el punto O (0, O). 127. La extrema! u=ex
se puede incluir en el campo propio de ext rernales y=ex+C. 128. Si
a< n, la extrema\ se puede incluir en el campo cent ral de ex-
tremales y=Csenx con centro en el punto 0(0. O). Si a>n, las cur
vas de la famil ia y= C sen x no forman campo. 129. La extrema! y=
=x+ l se pede incluir en el campo propio y=x+ C. 130. y=
:e - x: . 131. y ( -x) = O. 132. yi-1 = O. 133. O (1, O).
134. No hay punt o conjugado. 135. Se cumple. 136. Se cl!TTlple cual
quiera que sea a. 137. La condicin de Jacobi se cumple. La extrema!
y= O se puede inclu lr tanto en un carr.tpo central como propio de ex-
trema les. 138. La condicin de Jacobi se cumple. La extrema! y=
!..=l.. x+ l se puede incluir en un campo central de extremales con
(l
centro en el punto A (O, l}. 139. La condicin de Jacobi no se cumple .
142. Se puede. 143. Se puede. 144. Se puede. 145. Se puede aunque la
condicin de se cumple slo para ! <l. 146. Se alcanza
mrnlmo fuerte en la funcin y=t%. 147. Se alcanza mnimo fuerte
en la funcin y= 2 In (X + l). 148. Se alcanza mln mo dbil en la fun-
b
cin . . 1.49. Se alcanza mi_nimo dbil.. en la recta 11=- x.
a
. In (l+x)
160. Se alcanza mnimo en la curva y ln
2
. UH. Se al -
canza mximo fuer te en la curva g = cosx+senx. 152. No se alcanza
extremo en curvas continuas. J53. Se alcanza mlnimo dbil en la recta
11- 2x + l. No hay extremo fuerte. 154. Se alcanza min imo fuerte en
la extrema! 11=2x- l. 156. Se alcanza mlnimo fuerte en la extremal
156. Se alcanza mlnimo dbil en la extrema] y = x- l. 157.
En la extrem11l 11=.!.x se alcanza mnimo dbil si
a v2
y mhill)Q :si 'b' > y2 . No hay extremo si 1 b 1 = V2 .
186
RESPUESTAS E INDI CACIONES
158. Si p =F q, se alcanza minmo dbil en la extrema\ y=
= {[(q
3
1
2
.--p
3
1
2
1 x+p
3
1
2
)1; si p=q, la extrema! es la recta !J=P
que ofrece mnimo dbil.
X
sh y;-
!1 = 2
sh V8
159. a) Para e> O la extrema] realiza el mlnimo
1
fuerte de Ja func ional. b) Si E.< o y 1 e'> nz . la extrema! !I =
X
sen -=--
V .,, realiza cl mximo ruerte de la funcional. e) S e = O,
sen V 1 el
no existe solucin di!! problema ext rema! en la clase de funciones
x-1
continuas. Consideremos la funcin Yr. (x) =e. Ve (e> 0) que es
solucin de la <.'cuacin de Euler ey"-y=O de la funcional conside
rada. La funcin Ye (xl satisface la condicin de frontera y (1) = 1
y no satisface la segunda condicin de frontera g (O)= O. No obstante,
lim !Ir. (0} =O. Para e-.. O obtenemos de !Je (x) la csolucin limilet
e-o
!J (X)=
{

1,
O ~ x < I
x= J.
2 In li J.. X) .
160. La extrema! y= - In
2
realiza el mnimo fuerte.
161. Hay mnimo fuerte en la extrema) y(x)=I. 162. En la extrema]
( )
b 1 . . d . b., . b va , . d. b. 1
y x =-ax se a canza mm1mo e 1 si -a<-
2
- y max1mo e 1
' b 113 . b lf 3 1 . . 1 t
SI a > -
2
-; SI a=-
2
- , no se a canza n1 s1qu1era e eX remo
dbil. 163. En 111 reci a y .. ~ x se alcanza mnimo dbil si b<a
a
y mximo dbil si b >a si b ;p. a va. hay mximo fuer te mientras
que para b <a V3 no hay ni m nimo ni mximo fuertes. 164. Se
alcan:ia minimo dbil en la extrema! Y=2x, z=4x. 165. La extrema!
es la parbola { !I = ~ , que se puede incluir en el campo cen-
Z = ~ -X
tral de extremales
{ 1)
RESPUESTAS E 1 NOICACJONES 187
a y son unos parmetros) con centro en el punto (O, O, O). Es
obvio que se cumplen las condiciones reforzadas de Legendre. Demos-
tremos que en el segmento x < 1 no hay punto x conjugado del
punto x = O. Para ello bastar persuadirse de que las extremales de la
familia {I) no cortan la extremal dada si x E [O, IJ. Supongamos que
en un punto x E (O, 1) se cortan dos extremales e la familia (\).
Tendremos entonces
a
1
x=a2x , }
x
2
+ = x
2
+
De aqui resu!ta que a
1
= a.
2
y que =

Luego, no hay dos extre-


ma les distintas que se corten. Por consiguiente, la condicin reforzada
de Jacobi se cumple en e l segmento ro. 1) e incluso en cualquier seg-
mento de lonttud finita. 166. La familia de extremales es g (.x) =
= C
1
ch .xCt
2
A.. Las constantes arbitrarias C
1
y C
2
y e l parme-
tro A. se determinan de las condiciones
e
Xo-C2
!lo= t ch C1
A.,
lCi
) VT+7idx==C1 (sh x1-z.C2 h
xo-C3 }-I
5
C1 - .
X&
167. g (x) = 3x2 + 2x + J. 168. g (.x) = 2 sen rutx, donde n es un
1 ,r-
mimero entero 169. y(x)=T(2x-xZ), 170.
11
6. 171. r= R. z=C
1
+
4 , r- , r- '\IO
+Czcp. 172. VS . 173. V 20. 174. 2 V 2-1. t76. -o-. 178.
lfTI l f" ,r ( 5 -. r-)
-r-. 179. y 17+4 V 6 2-11 6 . 180. \. 181. Si cos Xt :{; 0,
el extremo se puede alcanzar solamente en la rec ta { . En
cambio, si cosx
1
= 0, o sea x
1
= ; +me, donde n es un nmero entero,
{
y=C,senx,
se tiene C siendo Ci. una constante arbitraria. 182.
Z= - 6 .sen X,
26
J (A, B) = 4 cth l. 183. J {A, 8) =T. 184. !J = 2x
2
1
3
. 185. Las lneas
quebradas formadas por los segmentos de las rectas y = x e y :::;:. 1 o por
los segmentos de las rectas g=O e u=x-1 realizan el mnimo abso-
La recta Y=+x real iza el mximo dbil. 186. !/= -x para
188
RESPUESTAS E INDICACIONES
O<x<I; y=x-2para Y=-x+6
para 3 < x < 4. En ambas quebradas la funcional alcanza su m nimo
b 1
{
o,
a so ulo. 187. No existen. 188. y= 189. Las extremales
x, x>O.
son lneas rectas. Si J Yz- Yt / < 1, existen dos soluciones discont-
X2-x1
nuas que son lneas quebradas paralel as a las bisectrices de los ngu-
los coordenados. 190. La recta y =x tg <p que pasa por los puntos ijos
realiza mximo dbil si O<tgq><n, realiza mnimo dbil si n<
< tg q < 2Jt, etc. El mnimo fuerte se alcanza en la lnea quebrada
formada por segmentos de rectas tales que la ta ngente de los ngulos
de su inclinacin es igual a 4n;-
1
n (n es un nmero entero) .
r
3
.- 16
Tx,
o '


16 .- ...... 34
191.
y (X) c, l
-- (x- 5)2.
5"'-x..,.,,5,
_ 3(x- JO)
34
+ 4

192. Las exlremales son las elipses
( 11
con centros en el eje Ox. La frontera del recinto admisible se deter-
mina por las ecuaciones y=O e y2= 2(x-C
3
) (la lti ma es la
solucin de la ecuacin 1-y2y'2=0). Los parmetros C
1
y C
2
se esco-
gen de modo que Ja elipse (1) pase por los puntos fijos A y B. La funcional
alcanza mximo en el arco de la elipse. Si el camino del punto A al
punto B se escoge segn los arcos de dos parbolas (y, posiblemente,
segn un segmento de la reda y= O), se obtiene una solucin con
puntos angulares en la que la funcional akam:a su mnimo (mn J =O).
dy x2y2 dp xZy dy p dp pZ
193
d'.t = 4p2 ' dx =2;)
194
dx = 2xg ' dx = 4xy2'
195. !!Y_ p ' dp = y . 190. dyt =
dx Vx2... y2- pz dx y xz+y2-p2 dx
P1 dp O dy2 P2 dp2 2 197. dyt =..!!2._ dyz
=2 ' d'X'""'=T' ([X' = Y
2
dx 2y
1
' -=
_ J!L clp _ Pr dp2 tl!Ji Pt d112
2y
2
' dx ' -;;-- --
198
{[X= -7' Tx =
1
= -Vpz, dp1 '-' 2x, dp2 -0: cfy1 Pt dy2 -
dx dx dx"" - 2 ' -- = V pz,
RESPUESTAS E INDI CACIONES 189
=2X, = 0. 199. g3=C1x + C2. 200. y3 =1n2x. 201. )("'
J
dy x2-x-l
=C
1
V +C
2
202. En la extrema! 11 =
2

oi C!l>-C:
alcanza el mnimo fuerte: mn J = - . 203. p (x, y ) =

.
Las exlremales son las semicircunferencias g = V q-(x-C
2
)Z con
centros en el eje Ox; g = V 2C1x --.x2 son las extrema les que pasan
por el origen 0(0, O); el campo es el semiplano superior. 204. El arco
de la circunferencia que pasa por el punto M
1
(x
1
, y
1
) y que t iene el
centro en el punto O (O, O) real iza el mnimo fuerte. 205. xP
206. Las elipses 3xZ- 8xy+ 6y2=C. 207. x3+2gS - 3xgZ-2x2y=C.
2os. /=Yt+72. 209. f = xyVy'. 21 0. f = xy1/z. 211. t=
=v(*+ ;2) (x
2
y'
2
+y2), 212. Una catenaria. 213. Indicacin.
La integral de la accin es J = J V +2h V p2-l- p'2dcp. 214.
xz 2y2 2 cos p sen2
tra:;cctotias son las elipses -e +-
2
h e - V xy = - k-
- . C{2h-C)
215. La solucin exacta es 11 = - x. 216. Las soluciones exactas
son a) y e= O, b) Y= X. 217. La solucin exacta es !I = (x2 - x). 218.
La solucin exacta es g=

- x. 219. Indicacin: la sol ucin apro-


11
xlmada debe buscarse en Ja forma lln (x) = (J-x2) <X1tx21t , La so-
11-0
1U
lucin exac:ta es 11 = cos T . 220. lndlcacln: tomar xy como funcin
b
2
-al! . ,
coordenada; entonces, Zt b?+az xy. 22l. lnd1cac16n: tomac cp
0
(x, y)=
= xi + yz, q>t(x, y)=xy ( l - x - g), 1J>2. (x, g)=x2y (l-x-y), ...
. . ., Cl>n (x, !1) =x'lg(l -x- y) como (unciones coordenadas; entonces,
2
3
(x, y)=xz+11z+xy(l-x-y)[3,0401-0,0562(x+1C2)]. 222. Indica-
ci n; hallar la primera aproximacin en la rorma z
1
(x, g) = ( y2 -
-

)a(x); entonces .<i(x, y) ... - (1 - (y2-

).
190
~ E S P U E S T S E I NDICACIONES
3 ( 1 ) ( 1 - 35 2 . 35 )
223. zt(x. y) =-- yi-
3
xi
1
_
311
x-
1
_
36
x-5-1 . 224. "-n=
= 1 + n
2
.n
2
, !In lX) = \! 2 sen nnx (n = l, 2, .. ). 225. h.,.=
- lnZ 2+ 4nztrz )- sen (To;- In x} 226 ' 25+ 2n2n2
- 4 lnZ 2 ' !In (x - - - ' "'n = 4
1
V 1nV2v x
( )
_ + \! 2 sen (nn In x) ...
1 2
)
227
,
1
2
)
!In x - _ \!x (n - . 11.,1 = -n , !In (X =
=y! sennx (n = 1, 2, .. . ).
228.
[
nn In (l+x) J.
13 ln
2
2+4nznz sen In 2
4 lnZ 2 ' Yn (x) = i r---,--= , /-
V In V 2 V 1 +x
(n = I, 2, ... ).
229. Tomando y = 1- xi, encontramos A.
1
-<
1
3
! . El valor exacto
1
es A.
1
=T. 230. Tomando y= x ( 1-x), encontramos A.
1
-< 10. El valor
exacto es ;>..
1
= :rt
2
231. A.( = 10 el valor exacto es A.f = n2. 232. A., =
= 0,493. 233. A., = 6; Zt (x, JI)= IX (xZ + gZ - 1).
A NUESTROS LECTORES:
MIR-RUBIOS 1860 edita libros soviticos tradu-
cidos al espaol. Entre ellos figuran las mejores obras
de las distintas ramas de la ciencia y la tcnica; ma-
nuales para los centros de enseanza superior y escue-
las tecnolgicas; literatura sobre ciencias naturales y
mdicas. Tambin se incluyen monografas, li bros de
divulgacin cientfica y ciencia ficcin.
Dirijan sus opiniones a MIR-RUBIOS 1860.
C/. Alcal, 98 - 28009 Madrid.
Al priocJp10 de cada capitulo
se resumen los resultadQS prin
cipales. se exponen los conoei
mientos tericos necesarios, ltls
rmulas requeridas y se estu-
dian con gran detalle ejemplos
tpicos i lu.strativos.
Este manual contiene ms de
100 ejemplos analizados y 230
problemas destinados para resol
verse independientemente. Unos
problemas se acompaan con
las respuestas, otros, con las
refereot'ias de cmo deben re-
sol verse.
La obra est destinada para los
estudiantes de centros de ense-
anza tcnica superior que se
especializan en los clculos ma
tcm lieos.
ISBN 84-604-1605- 4

Вам также может понравиться